X



英文法・語法に関する質問 Part 17
レス数が1000を超えています。これ以上書き込みはできません。
0001名無しさん@英語勉強中 (ワッチョイ f339-fBAy)
垢版 |
2020/06/15(月) 13:14:56.15ID:Se9m9GJU0
???
0002名無しさん@英語勉強中 (オッペケ Sr87-1F3g)
垢版 |
2020/06/15(月) 13:28:07.81ID:yucyRy7xr
別スレにて質問したのですが有効な返答が返って来なかったのでここでも質問させてもらいます

この添削あってますか?
https://i.imgur.com/MLxMrmK.jpg

まずa maskじゃなくて masksでしょ
those whoなんだからそれを受けてTheir keysだし
それにdid notって何で過去形なの?現在形でいいと思う
0005名無しさん@英語勉強中 (オッペケ Sr87-1F3g)
垢版 |
2020/06/15(月) 14:44:03.86ID:OXNisZaHr
いやネタじゃないからw
結局ドヤ顔で添削してるけど添削自体も間違いだって事でOK?
0009名無しさん@英語勉強中 (ロソーン FF27-10Ae)
垢版 |
2020/06/15(月) 19:01:25.14ID:MF7QjgbAF
556 名無しさん@英語勉強中 (ワッチョイ c906-HP3t) 2019/08/19 12:40:37 ID:tBiXpCoL0
> >訳が意味不明になる原因の三割くらいは「多義語の読み間違い・解釈違い」で説明がつきますからね
> >読んでいて意味不明だな、と思ったら「ひょっとしてどれかの単語に私の知らない意味があるのかな」と考えて、調べなおすのが重要ですね
>
>
> この作業を一つもやらないで意味不明な訳文を
> 「原文が難解なので意味は取れませんでしたが訳す事は出来ました!」
> って提出する子、先生は本当に困っています
0012三年英太郎 ◆3CZBjOt3.Y (ワッチョイ ff86-EVMN)
垢版 |
2020/06/15(月) 20:56:35.46ID:BxgjDrNG0
The following explanations apply particularly to
British English. Americans often use "have (got)"
to where British people use "must" (75). However,
this use of "have (got)" to is becoming more common
in British English under American influence. And
note that "must" is becoming less common in both
British and American English. PEU§73

理由が外的であれ内的であれ、とりあえず have to
をつかっとけば間違いない
0013名無しさん@英語勉強中 (ワッチョイW 6307-//tw)
垢版 |
2020/06/15(月) 21:19:29.75ID:dr2Ta+6c0
教えて欲しいです。
Any handsome men in your life?
と書かれてましたが、

これは、
Is ther any handsome men in your life?
ということでいいのでしょうか?
0014名無しさん@英語勉強中 (ワッチョイ ff32-NjyN)
垢版 |
2020/06/15(月) 22:10:45.82ID:UTffzicK0
>>13
前後がイマイチ不明だが
Have you had とかHave you got (any handsome men in your life?)
だと思うよ

今までハンサムな彼氏いたことある? 今までハンサムな彼氏ゲットできた? みたいな

there is 構文はこういう恋愛関係の言い方にはあんまり出てこないような
0015名無しさん@英語勉強中 (ワッチョイ 63f6-+Do1)
垢版 |
2020/06/15(月) 22:32:11.06ID:Hek9ZyZM0
良い子のみんなは、たくさん英文を読まないと、ただの文法オタクで英語が全然読めない
吾輩は猫先生であーる ◆MLdq8Ig59zES 別名 English geekのようになってしまうぞ。

気を付けよう二ャーーン

(^◇^)  (=^・^=)  (^◇^) (=^・^=)  (^◇^)
0016名無しさん@英語勉強中 (ワッチョイ f339-fBAy)
垢版 |
2020/06/16(火) 05:32:41.30ID:EZHljncV0
>>13
●Are there● any handsome men in your life?
("Out of the Frying Pan, Straight Into the Fire" by Tina Clinger)
.

https://books.google.co.jp/books?id=UG_w4m_2XNIC&;pg=PA35&lpg=PA35&dq=%22any+handsome+men+in+your+life%22&source=bl&ots=L2CNZ-Dz4W&sig=ACfU3U3-BMZ4hibw9HswW
7wgkyujNicgAg&hl=en&sa=X&ved=2ahUKEwj7vKXZ04TqAhVkyYsBHeUfDvYQ6AEwAHoECAQQAQ#v=onepage&q=%22any%20handsome%20men%20in%20your%20life%22&f=false

.

直接の参考になるかどうかわからんが、次のような例文もある。
“Have you ever been engaged? Any beautiful women in your life?”
His face hardened. “●There is● no one you need be jealous about.”
("The Heart Gem" by Isabella Macotte - 2012 - ‎Fiction)
0017名無しさん@英語勉強中 (ワッチョイ f339-fBAy)
垢版 |
2020/06/16(火) 05:43:45.70ID:EZHljncV0
"there any handsome men in your life"
を検索すると、小説などでも一応は出てくるみたいだな。
https://www.google.com/search?client=firefox-b-d&;q=%22there+any+men+in+your+life%22


"have any men in your life"
の用例の方がはるかに多いのかな?それは俺にはよくわからんけど。
https://www.google.com/search?client=firefox-b-d&;q=%22have+any+men+in+your+life%22
0018名無しさん@英語勉強中 (ワッチョイ fff0-NjyN)
垢版 |
2020/06/16(火) 12:34:13.48ID:nV+Pzvt00
「松濤の東側の地域は危険です。」をgoogle翻訳にかけると
"The area east of Shoto is dangerous."
となりますが、これだと松濤地域の東側の部分と混同される恐れはありませんか。

「松濤の一部ではなく、松濤より東側に位置する」ことが明確に分かるようにする表現があれば教えてほしいです。
0019名無しさん@英語勉強中 (ワッチョイ fff0-EVMN)
垢版 |
2020/06/16(火) 13:03:58.16ID:jWaPEqYv0
to the east of
0027名無しさん@英語勉強中 (ワッチョイ 5f39-LS63)
垢版 |
2020/06/20(土) 05:10:59.29ID:IFR9tpsK0
b
0030三年英太郎 ◆3CZBjOt3.Y (ワッチョイ 9a86-1/Nl)
垢版 |
2020/06/21(日) 16:31:14.64ID:govjifII0
たびん、抽象的な数は単複の概念を逃れてるのでわ?
んで、英語のルールでは単数が無標なんで、単数扱いされると

Two is a number.
(cf. Two are better than one.)

and の場合、複数扱いも可らしい

Three and four make(s)/is/are seven.
0031名無しさん@英語勉強中 (ワッチョイ 1af0-1/Nl)
垢版 |
2020/06/21(日) 17:13:59.23ID:v7YKmXbt0
One and one and one is three ("Come Together" by the Beatles)
0032名無しさん@英語勉強中 (ワッチョイ b3f6-Epcz)
垢版 |
2020/06/22(月) 01:36:33.48ID:w/n9k1VP0
The pandemic has devastated economies around the globe, shutting businesses
and slowing spending. However, the fallout has been much more severe in some
countries than others, and nowhere is that more apparent than in unemployment
figures in the United States and Japan.

and nowhere is で始まる文の構造を教えてください。倒置になっているのでしょうか。
意味的には米国と日本の失業率よりも明らかなものはほかにない、という感じだと
思うのですが。that が何なのか、どんな役割なのか、もよくわかりません。

よろしくお願いします。
0033名無しさん@英語勉強中 (ワッチョイ 4e7b-IXeA)
垢版 |
2020/06/22(月) 01:54:36.19ID:emR8NioY0
倒置だね
thatは主語
0034名無しさん@英語勉強中 (ワッチョイ b3f6-Epcz)
垢版 |
2020/06/22(月) 02:03:14.44ID:w/n9k1VP0
>>33
回答ありがとうございます。主語ですか。

では当該文
and nowhere is that more apparent than in unemployment
figures in the United States and Japan.

and that is not more apparent than anywhere in unemployment
figures in the United States and Japan.
とでも書き換えられますか。元の文だと理解しにくいので書き換えていただければ
幸いです。
0037名無しさん@英語勉強中 (ワッチョイ 4e39-LS63)
垢版 |
2020/06/22(月) 05:17:18.66ID:oPP32rhg0
>>32
"Nowhere is that more apparent than in"
これはどうやら決まり文句のようだな。

Nowhere is that more apparent than in
そのことがどこよりも明らかなのは〜である
(英辞郎より)

少なくとも10年くらい前までは英辞郎には間違いがいくらか含まれているので、
少なくとも俺は使わないようにしていたんだけど、いいことも書いてあるんだな。
ネット上でこのフレーズを検索してみた。

(1) ●Nowhere is that more apparent than in● this historic town nestled by
Chesapeake Bay, which spilled waist-high water into the streets during
the hurricane.

(2) ... and administrative strategies, and ●nowhere is that more apparent than in●
reform efforts designed to encourage ethics and curb corruption in government.
(ネット上の書籍より)

(3) We wanted Ceviche to reflect this 'buzz ' and
●nowhere is that more apparent than in● our cocktails.
(ネット上の書籍より)

(続く)
0038名無しさん@英語勉強中 (ワッチョイ 4e39-LS63)
垢版 |
2020/06/22(月) 05:18:07.85ID:oPP32rhg0
>>32
(続きだよん)

他にもたくさんたくさん使われている。きちんとした書籍の中でも使われている
から、きちんとした言い回しなのだろう。さて、これを倒置しない形で
検索してみると、次の通り。

(4) And Bamford's love of Rolex ●is more apparent nowhere than in● this room.

(5) This ●is more apparent nowhere than in● the kitchen, where large and
small appliances save time and effort for people every day.

(6) This ●is more apparent nowhere than in● the well-timed online pictures
showing the perfect moment.

倒置しない形で見つかったのは、上のものだけ。だから倒置した形が定着していて、
大いに使われているらしい。
0040名無しさん@英語勉強中 (ILW 0Hca-WJsQ)
垢版 |
2020/06/22(月) 15:36:14.97ID:OSoC32UuH
>>32
このthatは代名詞で文は倒置ってことになったみたいだけど
Nowhere is more apparent thanっていう文章にくっついてる副詞では?
意味としてはto that degree的な

Nowhere is more apparent thanという文章自体が山ほど検索で出てくるし
0041名無しさん@英語勉強中 (ワッチョイ 8a8a-l/FX)
垢版 |
2020/06/22(月) 22:54:48.08ID:JpolcngK0
Would you like tea or coffee ?
という質問に対して
A: No, I don't want either.
B: Neither.

と答えた時の意味ですが、
B は紅茶もコーヒーも入りません。
でわかりますが、
A はどちらか片方でかまいません。
という意味であってます?
日本語で考えると返し方としておかしい気がして。

紅茶とコーヒーのどちら入りますか?
と聞かれてどちらか一つでいいです。

って答えられても困りますよね?
Aの答えの解釈をどうしたら良いでしょうか
0043名無しさん@英語勉強中 (ワッチョイ b3f6-Epcz)
垢版 |
2020/06/23(火) 02:40:56.40ID:CMQa3NVq0
>>37-38
非常に詳しい解説ありがとうございます。倒置の形のほうが一般的なんですね。
>>38で引用していただいた倒置でない形を読むことで理解が深まりました。
たくさんの用例ありがとうございます。頭にすっと入って来るようになりました。

>>35でnowhereをthanの後ろに書きましたが、前に置くのが正しいですね。
本当にありがとうございます。

>>40
新しい視点をありがとうございます。
thatを副詞として解釈するなら、日本語では「それほど」という意味になりますよね。
(英語ではto that degreeであると説明していただきました)
そうすると、moreが邪魔になるような気がします。that apparent (それほど明確)
という言い方はあっても、that more apparent となると、moreがそこにあることに
自分は違和感を覚えます。

>Nowhere is more apparent thanという文章自体が山ほど検索で出てくるし
自分の推測では、こういう例ではNowhereを名詞的に捉えるという誤用から広まった
のではないでしょうか。Nothing is more apparent than...という言い方はNothingが
代名詞だから正しい言い方です。Nowhereは本来副詞なので、主語にはなれないけど、
Nothingと同じく否定語なので主語の位置に来ることに違和感を覚えない人がいて、そういう
用法が広がってきているのではないでしょうか。

>>36
NY Times からです。
0046名無しさん@英語勉強中 (ワイーワ2W FF12-TxGj)
垢版 |
2020/06/23(火) 21:27:21.85ID:5quS8/k9F
The sea today is placid and the weather sunny and bright.
(今日の海は穏やかで、天気は晴れて明るい)

and 以下の文で動詞が省略できるのは何故でしょうか?
英検一級用の単語帳の例題です・
0047名無しさん@英語勉強中 (ワッチョイ 3ee3-VNxU)
垢版 |
2020/06/23(火) 22:04:39.38ID:jb2u3lVb0
>>46
語句の重複を避けるための省略ですね。
The sea today is placid and the weather (is) sunny and bright.

Some fish live in rivers and others (live) in the sea.
川に住んでいる魚もいれば、海に住んでいる魚もいる。
江川『英文法解説』 p.492
0048名無しさん@英語勉強中 (ワッチョイ b3f6-Epcz)
垢版 |
2020/06/23(火) 23:19:14.05ID:CMQa3NVq0
>>45
リンク先での回答、非常に勉強になりました。ありがとうございます。

どちらの回答者もthatが直前の the fallout has been much more severe in some countries than others
を指すということで一致していますね。

2番目の人は倒置された形のほうが自然に思えると書いており、勉強になります。

説明の中で使われていたこの文
Nowhere is it more apparent that the fallout has been much more severe in some countries than others,
than in unemployment figures in the United States and Japan.

仮主語itをthat以下で説明する文、一読では理解できずめまいがしました。
こういう文を書けるのはさすがだ。
0050名無しさん@英語勉強中 (アークセー Sxa3-FkTT)
垢版 |
2020/06/24(水) 00:27:56.81ID:yntUULDzx
>>47
なるほど。省略可能な動詞に共通点などあるのでしょうか?
再度述べなくても理解可能なくらい簡単な動詞に限って省略は起こるのでしょうか?
0052名無しさん@英語勉強中 (ワッチョイ 7fe3-H7K1)
垢版 |
2020/06/24(水) 17:45:06.90ID:Q+HJtAcQ0
以下『ロイヤル英文法』からの類例:

He studied music in Vienna and (studied) painting in Paris.
He was reading something but I don't know what (he was reading).
Mr.Mill teaches us mathematics and Mrs.Smith (teaches us) English.
The sun shines in the daytime and the moon (shines) at night.

[述語動詞の共通部分の省略] p.777
0053名無しさん@英語勉強中 (ワッチョイW 7f32-ukj8)
垢版 |
2020/06/26(金) 13:50:52.11ID:iCkbeYxG0
文法の質問と言うか

The other summer activity which I enjoy is watching fireworks.

と言う文なのですがこれを日本語に訳すのはなんとなくできても日本語から英語を作り出す事ができないです。
which見つかってI enjoyまでを主語して作ったのか、それをどうやって作れるのか英語の考え方ですか?教えてほしいです。スレチでしょうか?
0054名無しさん@英語勉強中 (ワッチョイW 4fe3-k3AD)
垢版 |
2020/06/26(金) 14:34:30.58ID:DHLpi9H80
>>53
これは中学レベルだよね
和文英訳も英文和訳も

関係詞周辺がわからないの?
もしそうだとしたら、文法書でも何でもいいから関係詞を使ったいろんな文をたくさん見るのがいいよ

自分が英語の勉強で1番効果があったと思う勉強方法は、ノートの右に英文、左に和文を書いて、和文を英訳する方法
0055名無しさん@英語勉強中 (ワッチョイ 7f7b-NJGG)
垢版 |
2020/06/26(金) 14:40:12.06ID:fctyIxpZ0
質問の意味が分からんw
0056名無しさん@英語勉強中 (ワッチョイW 7f32-ukj8)
垢版 |
2020/06/26(金) 18:05:30.90ID:iCkbeYxG0
>>54
はいNHK基礎3からです
関係詞はわかるのですがなぜ使うのかその英語の理論といか理解をしたいんです。
実際whichの接続詞とか必要なのか喋る場合はどう喋るのかこれは問題だからこういう作りなのか
0058名無しさん@英語勉強中 (ワッチョイ 3ff0-3xvz)
垢版 |
2020/06/26(金) 18:14:00.10ID:oajW6liX0
関係代名詞の訳しかたで質問です。
In response, relatives of two of the mutineers began a campaign that succeeded in turning public opinion against Bligh.
それに反応する形で、2人の反乱者の親戚たちが組織的な運動を始め、世間を反ブライに変えることに成功した。
that は主格の関係代名詞だと思うのですが、thatの前にコンマがないのに、なぜ叙述用法的な訳になっているのでしょうか?
0063名無しさん@英語勉強中 (ワキゲー MM8f-23NG)
垢版 |
2020/06/26(金) 19:22:35.37ID:mZg9XOiuM
>>57
自分で英作文してみないとなかなか必要性は分からないだろうね

     「これはケンが昨日買った本です」

これを英語で言ってみて
簡単な単語しか使わないから

そしたら、なぜ関係代名詞が必要か分かるんじゃね?
0064名無しさん@英語勉強中 (ワッチョイ 3f3f-DlpM)
垢版 |
2020/06/27(土) 12:47:42.44ID:J+23VhwE0
他動詞と目的語の間に前置詞が必要でないのは何故なのでしょうか?
それとも、古英語では必要だったのでしょうか?

変な質問ですがよろしくお願いします(他掲示板でも質問したのですが無回答でしたので)。
0065三年英太郎 ◆3CZBjOt3.Y (ワッチョイ 3f86-5fXH)
垢版 |
2020/06/27(土) 13:58:38.02ID:1mjhYKD20
それぞれの動詞は、文の中で原則的に必ず名詞(項)を
取らなければいけないが、その数が決まってる。

I left. 1項
I eat pizza. 2項
I made her pizza. 3項

このうち、2項以上のものを他動詞という。

次のような自動詞+前置詞/副詞の句動詞も
他動詞として再分析することが可能。

I [stopped by] him.

なので、他動詞だからといって前置詞がない
とも言い切れない。
0067名無しさん@英語勉強中 (ワッチョイ 3f9d-wFiy)
垢版 |
2020/06/27(土) 17:25:01.79ID:EV69ZPo40
He could get something more than ramen to eat.という文のmore than はどういう使い方なのでしょうか?
日本語訳は、彼はラーメンよりマシなものを買って食べれる です 
0068名無しさん@英語勉強中 (ワッチョイ 3f3f-DlpM)
垢版 |
2020/06/27(土) 21:09:04.43ID:J+23VhwE0
>>65

レスポンスありがとうございます。

ただ、当方の疑問は、他動詞と目的語の間に、
日本語の「を」に相当する前置詞が必要でないのはなぜか?
そしてそれは古英語からずっとそうか?ということでした。

あらためてこの点についてはどうなのでしょうか?
0069名無しさん@英語勉強中 (ワッチョイ 7fe3-H7K1)
垢版 |
2020/06/27(土) 21:26:33.08ID:J44QCeK70
>>68
膠着語と屈折語について調べて見るとよいのでは?

膠着語 https://ja.wikipedia.org/wiki/%E8%86%A0%E7%9D%80%E8%AA%9E
屈折語 https://ja.wikipedia.org/wiki/%E5%B1%88%E6%8A%98%E8%AA%9E

日本語は膠着語。
英語は屈折語だが、屈折語尾はほとんど失われている。

ついでながら、中国語などは孤立語。
孤立語 https://ja.wikipedia.org/wiki/%E5%AD%A4%E7%AB%8B%E8%AA%9E
0070三年英太郎 ◆3CZBjOt3.Y (ワッチョイ 3f86-5fXH)
垢版 |
2020/06/27(土) 21:55:54.72ID:1mjhYKD20
>>68
言語の違いとしか言いようがないネ

たとえば属格なら
日本語は「ノ」、英語は「of」、仏語は「de」が必要だけど
ドイツ語はいらない(言い方もある)

日英独仏では前/後置詞が必要な言い方でも
スラブ語でいらなかったりする

> 古英語からずっとそうか

ずっとそうだろうけど、対格に屈折はしてた
0071三年英太郎 ◆3CZBjOt3.Y (ワッチョイ 3f86-5fXH)
垢版 |
2020/06/27(土) 22:02:50.17ID:1mjhYKD20
カレー食べたい
I want to eat curry.

なんで日本語は「〜たい」って欲求を表す法(desiderative)
があるのに、英語では動詞の変化で表せないの?

言語の違いという以外にこたえられる?
0072名無しさん@英語勉強中 (ワッチョイ 3f3f-DlpM)
垢版 |
2020/06/28(日) 18:15:40.84ID:6+c5r6I00
>>69

膠着語、屈折語、孤立語についてあらためて見て思ったのですが、
これらは言語の特徴や違いをあまり捉えたものではないようですね。
このような特徴づけや区別は今も評価されているのでしょうか?

もう一点。
世界の言語は互いに違うというより、むしろほとんど同じにも見えます。
そのような観点の研究はなにかあるのでしょうか?
0074三年英太郎 ◆3CZBjOt3.Y (ワッチョイ 3f86-5fXH)
垢版 |
2020/06/28(日) 19:00:08.01ID:SX7pSqMV0
印欧語の長い長い歴史から言えば、むしろ発想は逆で
なんて対格(と一部与格)だけは屈折⇒前置詞で代用
にならなかったんだろうと思うが (よく使うから?)
0075名無しさん@英語勉強中 (ワッチョイ 8f8f-wMgy)
垢版 |
2020/06/28(日) 19:04:01.87ID:gp9c9oJ30
教えてください。

Do you think that people today should take
more of an interest in political issues?
(今日の人々は政治問題にもっと関心を持つべきだと思いますか?)
これは英検準1二次試験の設問なのですが、
take more of interestではなく
take more interestが正しいように思うのですが。
つまり、なせ of が必要なのかわかりません。
0079名無しさん@英語勉強中 (ワッチョイ 3f3f-DlpM)
垢版 |
2020/06/28(日) 20:34:35.71ID:6+c5r6I00
>>73

チョムスキーの普遍文法は、いわば「あらゆる言語に共通する基本ルールがある」
という主張であり、「あらゆる言語はほとんど同じ」とまでは言っていないと思います。
そこで示される共通の基本ルールはごく抽象的なレベルのものであり、
その程度のものでは、外国語学習のような場面ではほとんど有用性をもたないように思います。
0081名無しさん@英語勉強中 (ワッチョイ 3f3f-DlpM)
垢版 |
2020/06/29(月) 08:59:41.39ID:9OYEJorR0
>>80

そんなことはどうでもいいので、なにか知ってることがあれば教えてください
0083名無しさん@英語勉強中 (ワッチョイ 0f5a-FPvj)
垢版 |
2020/06/29(月) 11:16:04.57ID:QHnDU3Sd0
「同じ」だからみんな「言語」という分類に入れられ、
「違う」から「X語」「Y語」みたいな個別の名称が割り当てられるんだろう。
その中でも比較的類似点が多い言語は「〜語族」にまとめられるんだろう。

漠然と「同じ」だのと言われても意味不明。
比較言語学か何かの入門書でも読めばいいんじゃないの?
0084名無しさん@英語勉強中 (ニククエW 3f5d-uvpZ)
垢版 |
2020/06/29(月) 20:00:46.94ID:rSCLx3AH0NIKU
>>75
結局これなんなん
ofでan interestが微妙に強調されてる感じ?
0085名無しさん@英語勉強中 (ワッチョイW 0f07-V+IW)
垢版 |
2020/06/30(火) 19:55:25.97ID:3A5TCR390
It makes me laugh.
It makes me laughing.
について、

以下サイトで説明があったんですけど、「常に」と言ったら、It always makes me laughing.になるの でしょうか?
It always makes me laugh.ではダメなのかということです。

サイトです。
https://www.usingenglish.com/forum/threads/245600-It-makes-me-laugh-laughing
0089名無しさん@英語勉強中 (ワッチョイ 6b60-ERT+)
垢版 |
2020/07/01(水) 04:11:17.56ID:+LsNyA190
i'll meat up with you guys kater i gotta read a book
god your being annoying just go read a book

これどういう意味?

君とやりたくなったら、後でね、本をよむよ。
ああ、悩みがある時は本をよむことさ。

でいい?
go read a bookの意味をさがしてたらUrbanDictionaryの例文で見つけたんだが意味不明。
go readって複合動詞ってことだろうか?
0090名無しさん@英語勉強中 (ワッチョイ 6b60-ERT+)
垢版 |
2020/07/01(水) 04:21:01.35ID:+LsNyA190
シチュエーションがよくわからないんだが、たとえば
僕  「泊っていく?」
彼女 「貴方としたくなったら、そうね後で、本をよむわ。」
僕  「ああ、悩みがある時は本をよむのが一番さ。」
みたいな感じなんだろうか?

guys kater て間投詞で「後で、、」みたいな感じなんでしょ。
0091名無しさん@英語勉強中 (ワッチョイ 6b60-ERT+)
垢版 |
2020/07/01(水) 04:30:27.54ID:+LsNyA190
わかった。文の切り方が問題、たぶん例文はこうだね。
僕  i'll meat up with you guys.
彼女 kater i gotta read a book.
僕  god, your being annoying just go read a book.

僕   しようよ。ねえ。
彼女  また今度ね、 本を読みたいの。
僕   そうだね。悩みがあるなら本を読むのが一番だね。
0092名無しさん@英語勉強中 (ワッチョイ 6b60-ERT+)
垢版 |
2020/07/01(水) 06:40:15.09ID:+LsNyA190
>>84
take more capital than
〜よりも多くの資本を要する[必要とする]
take more cases to trial
より多くのケースを裁判に持ち込む、裁判に持ち込むケースを増やす
take more concentration than
〜よりも多くの集中力を要する[必要とする]
take more courage
さらなる[いっそうの]度胸[勇気]を要する[必要とする]
take more effort on someone's part
〔主語のために〕(人)がもっと努力する必要がある
take more notice of ~ than
…より〜に注目する[を注意して見る]
take more than the prescribed dose of
処方された量より多く〜を服用する
take more time in ~ than any of one's competitors do
どの競争相手よりも〜に時間をかける

take more of a commitment than
〜よりもさらに深い関わり合いを必要とする
take more of an interest in scientific matters
科学的な事柄にもっと興味[関心]を持つ

調べたところofがつくのはこの2例しかない。つまり関心、関係に関するときにはofがつくってことで
理解したらいいのではあるまいか?
0093名無しさん@英語勉強中 (ワッチョイ bb39-dE/J)
垢版 |
2020/07/01(水) 07:12:09.09ID:WLwxUTDP0
more of a surprise
more of an effort
more of a book
more of a book person
more of a conversationalist
more of a crisis
more of a dictionary than a story
more of a study
0094名無しさん@英語勉強中 (ワッチョイ bb39-dE/J)
垢版 |
2020/07/01(水) 07:19:49.57ID:WLwxUTDP0
(1) The bug, which stands 58 feet tall and weighs two tons, is perhaps
even ●more of a sight● to behold during the holiday season, when it is dressed ...

(2) This is ●more of a report● by accountants for accountants for near-term purposes,

(3) But as a result, they were able to convict Frazee, who will
serve much ●more of a sentence● than she will, which is appropriate, since he was the ...

(4) “I don’t expect CVS to be making any changes,” Cantu said.
“They are ●more of a company● that needs to be forced into change. ..."

(5) Or would you think he might be ●more of a project● because of
some of the flaws you mentioned earlier?

(6) Larson said that without providing ●more of a program●, the university
isn't acknowledging the Deaf community.
0095名無しさん@英語勉強中 (ワッチョイ bb39-dE/J)
垢版 |
2020/07/01(水) 07:24:41.16ID:WLwxUTDP0
(7) Then I moved to New York and was working at MAC when I got a call
from a friend to do a photoshoot with these girls no one had heard of.
It ended up being Icona Pop, and later that month, “I Love It” exploded.
Their management called me: “They look like pop stars! Can you keep doing it?”
Slowly and surely it became ●more of a job●.
0096名無しさん@英語勉強中 (ワッチョイ bb39-dE/J)
垢版 |
2020/07/01(水) 07:29:31.17ID:WLwxUTDP0
(8) His art on Bernie Wrightson’s Frankenstein is the apex of a brilliant career,
but that’s ●more of a novel with pictures●, and this is a straight-up stand-alone
comic-book issue.

(9) Not a lot of people are ordering fine dining for delivery, as it is also
●more of an experience●, people pay for the ambiance, the music and the level ...
0097名無しさん@英語勉強中 (ワッチョイ bb39-dE/J)
垢版 |
2020/07/01(水) 07:33:29.74ID:WLwxUTDP0
(10) She later said “Like how different we are when it comes to marriage,
when it comes to faith, when it comes to lifestyle. I mean I’m just being
honest and I just got ●more of a sight● into life outside of this and just yeah
how different it all is.”
0098名無しさん@英語勉強中 (ワッチョイ 6b60-ERT+)
垢版 |
2020/07/01(水) 07:33:46.80ID:+LsNyA190
more ofはあってもTakeがついてない例はだめですよ。
0099名無しさん@英語勉強中 (ワッチョイ 6b60-ERT+)
垢版 |
2020/07/01(水) 07:37:33.25ID:+LsNyA190
take more of a surprise
take more of an effort
take more of a book
take more of a book person
take more of a conversationalist
take more of a crisis
take more of a dictionary than a story
take more of a study

こんな例あるんか www
0100名無しさん@英語勉強中 (ワッチョイ 6b60-ERT+)
垢版 |
2020/07/01(水) 07:39:17.13ID:+LsNyA190
>>93
ご苦労様。該当しません。
0101名無しさん@英語勉強中 (ワッチョイ bb39-dE/J)
垢版 |
2020/07/01(水) 09:50:33.81ID:WLwxUTDP0
(11) So, I think your question can be paraphrased as an internal sense of
questioning "Do we really have to use so many classes to implement patterns?"
and the answer is "sometimes", and of course, the seemingly ever-present
"it-depends". But it ●takes more of a book● to answer you, so I am
recommending that you get this one if you can.
0102名無しさん@英語勉強中 (ワッチョイ bb39-dE/J)
垢版 |
2020/07/01(水) 09:53:13.41ID:WLwxUTDP0
(12) For instance, he believed it ●took more of a person● to walk away from
a fight.
0103名無しさん@英語勉強中 (ワッチョイ bb39-dE/J)
垢版 |
2020/07/01(水) 09:54:41.38ID:WLwxUTDP0
(13) Elizabeth A. Reynolds - 2010 - ‎Biography & Autobiography
I replied that it ●took more of a man● to refuse drink than to give way to
desires and get drunk.
0104名無しさん@英語勉強中 (ワッチョイ bb39-dE/J)
垢版 |
2020/07/01(水) 10:00:04.80ID:WLwxUTDP0
(14) Dylan Perry - 2014 - ‎Fiction
He knew where to find Vulcan in the Underworld, but the other person
he wanted see in Uffern might ●take more of an effort● to locate.
0105名無しさん@英語勉強中 (ワッチョイ 6b60-ERT+)
垢版 |
2020/07/01(水) 10:17:29.88ID:+LsNyA190
わわわ、たくさん例文が、、、
すすすいません。w
これら有志により発掘された例文から考えるとofはあってもなくてもかまわない。意味もそんなに変わらない。
take more of a book.
take more books.

take more interest.
take more of an interest.

take more of a person.
take more persons.

take more of an effort.
take more effort.

いずれもOK、意味もニュアンスもほとんど同じと結論できます。
以上
0106名無しさん@英語勉強中 (ワッチョイ 6b60-ERT+)
垢版 |
2020/07/01(水) 10:30:50.63ID:+LsNyA190
今気が付いたんだがof aと必ずaがくるのでa lot ofと同じ感じでひとかたまりを表すニュアンスがあるのではあるまいか?
take more of an effort. もうひと踏ん張りせよ
take more effort. さらなる努力をせよ

take more of a person. もう一人つれてこい
take more persons.   もっとたくさんの人つれてきな。

take more of an interest. 興味の一つも持ちなさい。
take more interest.    もっと興味を持ちなさい。
0107名無しさん@英語勉強中 (ワッチョイ 6b60-ERT+)
垢版 |
2020/07/01(水) 10:55:28.60ID:+LsNyA190
(12) For instance, he believed it ●took more of a person● to walk away from a fight.

歩いて逃げるには一人の力じゃ無理だろうと彼は信じてる。

この例文から判断すると一人以上ってことだな。もう一人じゃないな。
take more of a person. もう一人以上はつれてこい
0108名無しさん@英語勉強中 (ワッチョイ bb39-dE/J)
垢版 |
2020/07/01(水) 11:18:15.35ID:WLwxUTDP0
>> (12) For instance, he believed it ●took more of a person● to walk away from a fight.

この (12) の意味は、俺が思うには
「喧嘩を買わないでそこを立ち去るには、喧嘩を買うよりももっと勇気が必要だ」
という意味だと思う。この原文の前後関係を見たら、もっとよくわかると思うけど。

A man of few words, just sitting, having a cup of tea or just being with him
was so comfortable. He lived in the now – not the past or the future.
When you were ready to listen, he had some very profound things to say
about the way to live your life. For instance, he believed
it ●took more of a person● to walk away from a fight. He helped his
grandchildren to seek peace. He counseled that sometimes when we are
in moments of upset with those we love, whether we are right or wrong,
we can choose to just let things go and that not everything is worth fighting
over. He helped us understand that sometimes acceptance, even when
the other person is wrong, is necessary, because there is never a real winner
in a fight with someone you love.
https://www.sunburyfuneralhome.com/obituary/Robert-Wade/guest-book
0109名無しさん@英語勉強中 (ワッチョイ bb39-dE/J)
垢版 |
2020/07/01(水) 11:20:35.73ID:WLwxUTDP0
>>(13) Elizabeth A. Reynolds - 2010 - ‎Biography & Autobiography
>>I replied that it ●took more of a man● to refuse drink than to give way to
desires and get drunk.

この場合もやはり、
「酒への誘惑に負けて酔っぱらうよりも、酒を断る方がもっと勇気がいる」
という意味だと思う。
0110名無しさん@英語勉強中 (ワッチョイ bb39-dE/J)
垢版 |
2020/07/01(水) 11:23:37.45ID:WLwxUTDP0
>>(11) So, I think your question can be paraphrased as an internal sense of
questioning "Do we really have to use so many classes to implement patterns?"
and the answer is "sometimes", and of course, the seemingly ever-present
"it-depends". But it ●takes more of a book● to answer you, so I am
recommending that you get this one if you can.

この場合の takes more of a book の意味は、俺にとっては難しいのだが、たぶん
次のような意味だと思っている。

「君の質問に答えるには、一冊の本のうちのもっとたくさんの部分
(言い換えると「もっとたくさんのページに相当する内容量」)が必要だ」
0111名無しさん@英語勉強中 (ワッチョイ bb39-dE/J)
垢版 |
2020/07/01(水) 17:03:29.45ID:WLwxUTDP0
>>108-110
ここでの俺の解釈に疑問を感じる人は、どうぞ遠慮なくネイティブに尋ねてみて
ほしい。俺自身も確信があるわけじゃないからね。
0112名無しさん@英語勉強中 (ワッチョイW 8f96-vm5e)
垢版 |
2020/07/01(水) 22:32:28.42ID:O5GU+R2E0
I would set no limits to what gentlemen might consider shipshape, or might not, as the case were.
(スティーブンソン「宝島」より)

<質問>
as the case were.とあるが何でwere?
wasではないのか?

推測するに主文にorがある。
caseはあくまでひとつの状況であり単数、
しかしbe動詞は主文のorをひきずり
どっちにしたってということを示す意味でwasでなくwereを使うことで
いささか教養に欠ける人物の発言としてチグハグさ・発言者自身の迷いを含意して表した。
wasでも間違えではない。


<参考>
(翻訳例1)
「わしは紳士がきちんとしてると思うことや、思わないことなんかに、この場合とらわれないことにしますや」

(翻訳例2)
「多分あっしは紳士って方々がその時の場合によってどんなことを適当と考えるか考えねえかってことの区別をつけなかったんでしょう」
0114名無しさん@英語勉強中 (ワッチョイ bbf6-ERT+)
垢版 |
2020/07/02(木) 02:30:12.96ID:UMGMJppf0
コロナの感染者数に関する文です。

The daily number in Kanagawa was the largest since May 14. Twenty-eight of the 31
cases were reported in Yokohama, with the lion’s share of those — 26 — male employees
of a host club in the city.

withはいわゆる付帯状況のwithだと思いますが、(being) make employeesのように考えれば
よいでしょうか。male employeesといきなり名詞句が来ているのに違和感を覚えるので、
文法的な理屈が知りたくて質問しました。よろしくお願いします。
0115名無しさん@英語勉強中 (ワッチョイW 8f96-vm5e)
垢版 |
2020/07/02(木) 02:36:17.35ID:BvCcINqB0
>>113
回答ありがとうございます。

つまりこういうこと?(仮定法)

(前提シチュエーション例として)
銭湯の脱衣場から服も着ずタオルもはおらず裸で出て来るって?
例えば浴室から火事とか気違いが刃物取り出して暴れだしたとするだろ?

(先の英文)
俺ならジェントルマンシップに敵うとか敵わないとかそんなことには構っちゃいられないね(裸で脱衣場から飛び出してくるわな)、そんな場合にゃあね。

asよりifだと思うが、
asの「様態」だとか「条件」
そもそもの「〜のような」「同じような」の意から
ifでなくasでも可、asを用いることで話者間の共通理解の話・ケースであることを含意、
wereを用いることで仮定・突飛なシチュエーションである・あったことをハッキリさせた、ということですかね?
0118名無しさん@英語勉強中 (ワッチョイ bb39-dE/J)
垢版 |
2020/07/02(木) 04:50:19.91ID:Wl+2Gn8Y0
>>115
as the case were の as は、if にしたらまずいんじゃないかな?as にしてあるのは、
if とか because とか since というような単なる接続詞ではなくて、関係代名詞
としての which みたいな意味合いを兼ねているからだろう?

The only thing preventing Kmet from being the starting tight end over
an aging Graham is his lack of experience, ★as is the case★ for most rookies.

ネット上で見つけた上の文では、as the case were でもないし as the case may be
でもなくて as is the case だけど、この場合の as は which の意味合いを兼ねていて、
その前にある文章を受けているんだろう?

それと同じように、Stevenson の文章に出てくる as the case were でも、
as がその前の文章を受けていると思うんだけど、違うかな?
0119名無しさん@英語勉強中 (ワッチョイ bb39-dE/J)
垢版 |
2020/07/02(木) 04:53:09.69ID:Wl+2Gn8Y0
>>118 のように言ってはみたけど、自分でもよくわからんようになってきた。すまん、
俺にはこれについて説明する資格はなさそうだな。
0121名無しさん@英語勉強中 (ワッチョイ bb39-dE/J)
垢版 |
2020/07/02(木) 09:50:32.11ID:Wl+2Gn8Y0
>>114
その英文は、Japan Times に記載されている現時点での次のような一節にかなり
似ているな。もしも質問者がもともとこの記事から引用したのなら、質問者が
質問を提起した後に Japan Times が書き換えたんだろうな。もともとの
文章は、少しわかりにくかったもんな。

The daily number in Kanagawa was the largest since May 14, when 32 people
tested positive for the novel coronavirus.

Of the 31, 28 cases were reported in Yokohama, public broadcaster NHK
reported. Among the 28, the lion’s share — 26 — were male employees
of a host club in the city.

出典:
https://www.japantimes.co.jp/news/2020/06/30/national/tokyo-54-coronavirus-yokohama/#.Xv0ufudS_IU
0122名無しさん@英語勉強中 (スプッッ Sd4a-vm5e)
垢版 |
2020/07/02(木) 12:44:13.28ID:/Uep3BW/d
>>113
>>117
ありがとうございます。

やはり仮定法ですかね。しかも古い表現として。これで腑に落ちた気がします。

(参考)
個人的にも最初に仮定法と思ったのですが、下記のリンク先では
ネイティブからの返答として
古い表現、古い文学的な表現、誤用という意見が見られました。

そしてそのフォーラムで
検索結果として示された「as the case were」が用いられた数少ない例では(いずれも)主文に「or」があったということが気になったところでここで質問した次第でした。

https://forum.wordreference.com/threads/as-the-case-were.3704909/
0123三年英太郎 ◆3CZBjOt3.Y (ワッチョイ 8a86-Wiu6)
垢版 |
2020/07/02(木) 12:54:00.72ID:ZARg8Ti50
as the case were = as the case maybe

The Pirate Primer: Mastering the Language of Swashbucklers and Rogues

> A comprehensive course in pirate vocabulary, pronunciation, grammar, and syntax,
> The Pirate Primer contains three centuries of distinctive terms
> and usages uttered by (and attributed to) pirates in film, TV,
> literature, and history.

別に海賊用語とゆーもんでもなくて、まあ海賊モノにでてくる
(現在では読みがたい)語法を集めたもんでしょ。
宝島読みたければ、これ買えばいいんでない?
012475 (ワッチョイ a38f-xC9a)
垢版 |
2020/07/02(木) 22:57:27.39ID:Efo9uvxc0
>>92=104
色々調べていただきありがとうございました。
結局、はっきりした理由はわかりませんでしたが、
そのような表現があるという風に覚えていきます。
0125名無しさん@英語勉強中 (ワッチョイ bbf6-ERT+)
垢版 |
2020/07/02(木) 23:51:44.22ID:UMGMJppf0
>>121
質問文はジャパンタイムズのメルマガ「TAKE 5」からの引用でした。
ttps://www.japantimes.co.jp/email-newsletters

このメルマガ「TAKE 5」では、5本の記事の要約が届きます。それぞれの要約から元記事にリンクで
飛ぶことができます。>121さんのリンクした記事に飛ぶようになっていました。

>もともとの文章は、少しわかりにくかったもんな。

元記事が書き換えられたという訳ではなく、TAKE 5の要約記事では
あのような書き方がされていました。
0126名無しさん@英語勉強中 (ワッチョイ 6b60-ERT+)
垢版 |
2020/07/03(金) 16:22:04.06ID:SHj/FPJN0
>>114
The daily number in Kanagawa was the largest since May 14.
神奈川では5月14日以来の日レベルでは最大の感染数だった。
Twenty-eight of the 31cases were reported in Yokohama,
横浜では31のケースの24人が報告されているが
with the lion’s share of those — 26 — male employees of a host club in the city.
市のホストクラブの26人の男性従業員がシェアとしては群をぬいていた。

なんか問題あるんか?
24人に対して26人の関係がわからんという曖昧な文章であるという問題はあるが
文章そのものとしては問題ないような気がするけど。
24人全員がホストクラブの人間だったかどうかは分からないが、本当は実はホストクラブ
からは3人だけで「ホストクラブが悪い奴らだ、、、、」みたいな、そういう誤読を誘うために
態と曖昧にしているのではあるまいか?
0127名無しさん@英語勉強中 (ワッチョイ 6b60-ERT+)
垢版 |
2020/07/03(金) 16:25:01.41ID:SHj/FPJN0
あっすまん。28人だった。28人中26人なら何も問題ないな。そのとおりだな。
0128名無しさん@英語勉強中 (ワッチョイ 7539-kYYI)
垢版 |
2020/07/09(木) 03:49:59.71ID:gMWrTkts0
t
0131名無しさん@英語勉強中 (ワッチョイW cde3-GwjW)
垢版 |
2020/07/12(日) 19:24:57.65ID:3pFTOtfi0
TOEICの公式問題集の会話式のリスニング問題からです
文の意味はだいたいわかるのですが、文法的にわからないことが多く、もやもやするので質問させてください。

I just received a letter from you explaining that Dr.Patel is retiring inMarch, but I already had an appointment scheduled with him for April.
Is there another doctor being assigned to his patients?

【1】
「a letter from you explaining that〜」
thatはthat以下の同格ですか?
explainingはa letterを修飾してるのかなと思うのですが、from youが間にあり、よくわかりません
修飾部分が長いからfrom youが後ろに倒置されているのでしょうか?
倒置や省略がなければ以下のような感じでよろしいでしょうか?
a letter (which is) explaing〜from you


【2】
an appointment scheduled with〜の部分は
an appointment (which is) schedules with
ですか?

【3】
another doctor being assigned toの部分は
another doctor (who is) being assigned to
ですか?

よろしくお願いいたします
0132名無しさん@英語勉強中 (ワッチョイ 7539-kYYI)
垢版 |
2020/07/13(月) 09:37:19.86ID:vrdKYl9+0
>>131
(1) a letter from you explaining that ...
>>thatはthat以下の同格ですか?

文法用語を俺はあまり知らないんだけど、これは「同格」と呼ばれているのではなく、
explaining that S V というときの、「〜が〜するということを説明する」という
意味を作るにあたって、that を接続詞として使っているのだ。

>> a letter (which is) explaing〜from you

あなたは大筋のことは理解していると俺は察するけど、これは正確に言うと
a letter which is explaining
ではなくて
a letter which explains
ということだ。そして、a letter には from you が挿入されているのだと
解釈すればいいと思う。

(2) an appointment scheduled with〜の部分は
>>an appointment (which is) schedules with
ですか?

単なるあなたのタイプミスだとは思うけど、schedules ではなくて scheduled だな。
an appointment which is scheduled with ... で間違いない。

(3) (Is there) another doctor being assigned to his patients? の部分は
>>another doctor (who is) being assigned to
ですか?

その通りじゃ。
0134名無しさん@英語勉強中 (ワッチョイ 2ddc-k+PU)
垢版 |
2020/07/14(火) 18:19:53.72ID:fBOi0dOc0
政府が経済対策で推進しているgo to travelキャンペーンだけど
語法になんか違和感を感じる。
go travelが正しいんじゃないの?
ネイティブはgo to travelの使い方はどう感じる?
0136名無しさん@英語勉強中 (ワッチョイ 23f0-mXGD)
垢版 |
2020/07/14(火) 18:27:11.92ID:zxI8Ljct0
go 女 trip
0138名無しさん@英語勉強中 (ワッチョイ 7539-kYYI)
垢版 |
2020/07/14(火) 21:09:06.96ID:NaGv/C5g0
>>134
And cocaine is not only popular in New York. “When I
●go to travel● somewhere else, people think I do it and they’re
so eager to shove it up my nose,” said Roxy Summers,
a party promoter and D.J. who goes by the name Oxy Cottontail.

(New York Times)
https://www.nytimes.com/2007/06/10/fashion/10cocaine.html
0139名無しさん@英語勉強中 (ワッチョイ 7539-kYYI)
垢版 |
2020/07/14(火) 21:13:00.45ID:NaGv/C5g0
(2) Navigating the world map is rather simple, as you just point the cursor
to where you want to ●go to travel● there.
http://diehardgamefan.com/2013/07/29/review-mystic-chronicles-sony-playstation-portable/

(3) The state of California had law AB 1887 go into effect on January 1st
that prohibits state funds or state-sponsored legislation to
●go to travel● to states that have laws that discriminate
against the LGBT community.
https://www.californiagoldenblogs.com/2017/2/6/14526938/california-golden-bears-ucla-bruins-schedule-ku-kstate-unc-duke-ole-miss-tennessee-discrimination
0140名無しさん@英語勉強中 (ワッチョイ 7539-kYYI)
垢版 |
2020/07/14(火) 21:16:45.39ID:NaGv/C5g0
(4) “I think our industry has been unfairly singled out. People aren’t saying,
previously at least, ‘don’t go to a hotel, don’t ●go to travel● in all these other areas.'”
https://www.citynews1130.com/2020/03/13/cruise-ship-ban-virus/

(5) State organizations such as the Michigan State Department are working
to get the word out about this upcoming switchover well ahead of time to
ensure people don’t find themselves grounded when they ●go to travel●
after Oct. 1, 2020.
https://www.candgnews.com/news/michigan-secretary-of-state-prepares-for-real-id-regulations-in-2020-113023
0141名無しさん@英語勉強中 (ワッチョイ 7539-kYYI)
垢版 |
2020/07/14(火) 21:27:57.38ID:NaGv/C5g0
(6) She is a year younger than him and to ●go to travel● with him was difficult.
https://bleacherreport.com/articles/2783582-futsal-family-friends-the-making-of-brazils-creative-king-philippe-coutinho

(7) Though the safest thing to do may be to put your travel plans on hold,
for those who are ●going to travel●, experts agree that traveling by car is
a safer option when it comes to Covid-19 transmission.
https://www.cnbc.com/2020/07/02/how-to-travel-safely-by-car-during-coronavirus-pandemic.html

(8) We had ●gone to travel● to Brazil for a month.
https://www.biznews.com/thought-leaders/2018/12/14/matthew-booth-former-bafana-tall-guy
0144名無しさん@英語勉強中 (ワッチョイW 21af-01jJ)
垢版 |
2020/07/17(金) 15:46:24.90ID:ibNRBEGC0
Is that a lion? No,it´s not. It is a cat.という文のIt is a cat.のItは具体的に何を指しているのですか?itは人称代名詞で、具体的に名詞一語を指すとネットでみましたが、a lionじゃおかしいですよね?

わかるかた回答お願いいたします。
0145名無しさん@英語勉強中 (ワッチョイ 5a32-gTBF)
垢版 |
2020/07/17(金) 15:55:49.05ID:2PETQe3Z0
>itは人称代名詞

どこでそんなデマを…
0149名無しさん@英語勉強中 (ブーイモ MMa1-CtfD)
垢版 |
2020/07/17(金) 16:28:53.48ID:x8FxoGspM
itが指しているものはitを言った人にしかわからない
thatはライオンですか?と聞かれて
itは猫です。
状況的にネコ科の動物が2匹いると言うことだから
ライオンに見えるthatと猫であるitがいると言えるので
itが指しているものは猫が正解
0150名無しさん@英語勉強中 (ワッチョイ 7d32-xjR4)
垢版 |
2020/07/17(金) 16:49:36.95ID:RuYIPqW70
下記の文は文法的に正しいでしょうか。また他に自然な表現があれば教えてください。

The drought left many people in want of food.
0151名無しさん@英語勉強中 (ワッチョイ 7d39-PDgx)
垢版 |
2020/07/17(金) 16:58:50.91ID:emEF8s1l0
>>144
>>148 による回答と重なるけど、俺は次のように考える。

(1) Is that a lion?
(2) No,it´s not. --- しいて言えば、これは No, that is not. という意味。
(3) It is a cat. --- しいて言えば、これは That is a cat. という意味。

しかし実際には、that を何度も繰り返さないで、that は一回目だけ使って、
二回目以降は it を代わりに使うことになっている。
0152名無しさん@英語勉強中 (ワッチョイ 46e3-R8z5)
垢版 |
2020/07/17(金) 20:03:28.61ID:IcPP7NHm0
148, 151が正しい。

>>145
itは人称代名詞。そうでないとしたらいったい何なのか?
人称とは人のことだと思ってたり、3人称っていうのは3人いることだと思ってる中学生はときどきいる。
0154名無しさん@英語勉強中 (ワッチョイ 7d39-PDgx)
垢版 |
2020/07/18(土) 05:07:41.38ID:kKQk2HTw0
文法用語なんて適当に流せばいいとは思うが、一応。

Use of Subjective Personal Pronouns in the Sentence:

Following are sentences with the proper use of subjective ●personal pronouns●:

I love to learn English and other languages.
You are my best friend.
You are looking very happy.
She is a good singer.
He never hurts his good friends.
I like your uniform very much.
You have been very late.
He is not my friend
★ It is a nice and healthy fruit.
We have to go there.
They are playing football together.
They successfully won the competition.
★ It is raining very heavily.
https://www.teachingbanyan.com/grammar/personal-pronoun/

上に挙げたリンク先の一節では、無生物の it をも "personal pronoun" と
呼んでいるみたいだな。他の文法書ではどう書いているか知らんが。
0155名無しさん@英語勉強中 (ワッチョイ b66c-dDBt)
垢版 |
2020/07/18(土) 17:38:04.81ID:cnL6juii0
Thou this to hazard needs must intimate
Skill infinite or monstrous desperate.
0156三年英太郎 ◆3CZBjOt3.Y (ワッチョイ da86-hHzd)
垢版 |
2020/07/18(土) 17:43:29.21ID:43bNtkJp0
一人称 話し手
二人称 聞き手
三人称 その他

人称とは、本質的に人間のことであり、それ以外の拡張用法はおまけ
三人称とゆーのも、ほぼおまけ
(三人称がない言語多し。印欧祖語にも三人称はない)
0158名無しさん@英語勉強中 (ワッチョイ b66c-dDBt)
垢版 |
2020/07/18(土) 17:51:40.89ID:cnL6juii0
いちおうこう解釈した

Thou this to hazard needs must intimate
Skill infinite or monstrous desperate

thou たぶん主格
this hazardの目的語
Thou this to hazard ⇠ 主部(主語)
needs 副詞
intimate 動詞
monstrous 副詞
0159名無しさん@英語勉強中 (スッップ Sdfa-p4HR)
垢版 |
2020/07/18(土) 18:33:27.92ID:A9sYvhwRd
>>150
文法的にも表現的にも完璧です
0162名無しさん@英語勉強中 (ワッチョイ 7d39-PDgx)
垢版 |
2020/07/18(土) 19:32:04.76ID:kKQk2HTw0
原文
Thou this to hazard needs must intimate
Skill infinite or monstrous desperate.

現代語訳
If you're willing to risk all of this,
then that must mean you have either infinite skill or horrible desperation.

原文と現代語訳とを突き合わせて考えてみると、原文のそれぞれの語句は次のような
意味になるだろうと推測できると思う。

原文1行目 Thou this to hazard needs must intimate
これは、If thou must needs intimate to hazard this, という意味だろう。
なんでこんな変な語順になっているかというと、最後の intimate と2行目の
最後の desperate とを rhyme させたかったし、さらには1行目の
syllables が10個で2行目の syllables も 10 個を使って弱い音と強い音とをうまく
並べて iambic pentameter を形成しようとしてこのように並べたのだろうと思う。

さて、If thou must needs intimate to hazard this, の needs は副詞であり、
これはジーニアス英和(第5版)にも載っているが、must needs で
「ぜひ(どうしても)〜になければならない」という意味になる。

intimate は古い英語では suggest という意味になると
Alexander Schmidt の Shakespeare Lexicon and Quotation Dictionary
には書いてあるので、If thou must needs intimate to hazard this の全体は
「もし君が危険を冒しながらこのことを行うことになりそうだと
どうしても言うのなら」とかいうような意味になると思う。

続く
0163名無しさん@英語勉強中 (ワッチョイ 7d39-PDgx)
垢版 |
2020/07/18(土) 19:33:02.01ID:kKQk2HTw0
(続き)

そして次の
原文2行目 Skill infinite or monstrous desperate.
現代語訳2行目 then that must mean you have either infinite skill
or horrible desperation.  

原文2行目の順序を並べ替えると infinite skill or monstrous desperate になる。
infinite が形容詞で skill が名詞。だから infinite skill なら「無限の技能」つまり
「限りなく優れた技能」という意味だろう。

monstrous desperate の desperate はあくまで形容詞なので、monstrously desperate
というのが本来の英語なんだろうと思う。だから horribly desperate という
意味になるというわけだ。

全体では infinite skill or monstrous desperate となると、"名詞句 or 形容詞句"
となって変なので、本来なら "名詞句 or 名詞句" にするか、あるいは
"形容詞句 or 形容詞句" にすべきなのだが、韻文を作るためにはこのような
変則的な英文を書かざるを得なかったのだろう。ここでは poetic license (詩的許容)
つまり「詩を書くにあたってはリズムを最優先するために、少しくらいは文法的に
変でも許そうではないか」という原則を採用しているのだろう。
0164名無しさん@英語勉強中 (ワッチョイ b66c-sTnA)
垢版 |
2020/07/18(土) 19:48:37.50ID:cnL6juii0
これはたすかるわー

If you're willing to risk all of this,
then that must mean you have either infinite skill or horrible desperation.

thouは主格でto hazardの意味上の主語らしい(大塚って人の本の解説)
>then that mustからすると
intimateはやっぱ動詞か
ほのめかす、(・・だと)知らせる 公表する(ジーニアス)

これを一かばちかやってみようとするからには、
限りなきいのちがけの技量がお前にはあるに相違ない(前掲書、大塚の訳)
0165名無しさん@英語勉強中 (ワッチョイ 7d32-xjR4)
垢版 |
2020/07/18(土) 22:18:17.21ID:WQyObYfm0
>>159
ありがとうございました!
0166名無しさん@英語勉強中 (ワッチョイ 761d-97kB)
垢版 |
2020/07/20(月) 11:50:30.41ID:Pn7Xz9Zo0
略なんですけど、添削お願いいたします。
It always seems impossible until it's done. 何事も成功するまでは不可能に思えるものだ。
until it's doneのところなんですけど、it'sはit hasなんでしょうか?それともit wasなんで
しょうか?わかんなくなっちゃっています。どなたか教えてください。
0167名無しさん@英語勉強中 (ワッチョイ 76b5-G2ah)
垢版 |
2020/07/20(月) 12:18:44.58ID:7PpFFYSp0
>>166
it is の省略形
done は受け身というより「終わった」という意味の形容詞と捉えるとよい

通常 it was を省略してit's と書くことはない
時・条件の副詞節until節中に強調の意味で現在完了形がくることもあるが
その場合は、it has been done となる
0168名無しさん@英語勉強中 (ワッチョイ 761d-97kB)
垢版 |
2020/07/20(月) 12:52:21.01ID:Pn7Xz9Zo0
>>167
isですね!ありがとうございます。
0169名無しさん@英語勉強中 (ワッチョイ 9131-R8z5)
垢版 |
2020/07/20(月) 16:24:18.34ID:4PC0/e070
文法の本に仮定法について下のような例文がありました。

#仮定法的な「未来の弱い義務」
You should have finished your essay within 10 days.
「あなたたちは10日以内にレポートを仕上げておかねばなりません。」

質問-1 
実際このような言い方するんでしょうか?
質問-2
普通に仮定法の文章だとすると
「10日以内にレポートを仕上げておかねばならなかった。」
(実際はやらなかった)
という意味になりませんか?
質問-3
You should finish your essay within 10 days.
それなら、シンプルにこれでよくないですか?
0170名無しさん@英語勉強中 (ワッチョイ 7d39-PDgx)
垢版 |
2020/07/20(月) 16:43:53.38ID:Gwt1Q3W30
>>169
そういう表現が、どういう文脈の中で使われるかを、実際の読書の中で出会ったときに
確かめていけばいいのだ。しかし差し当たってそういう例文に読書の中で出会ったことが
ないのであれば、ネット上で検索し、その前後を読み取ってみるとよいだろうな。

(1) To make the most compost in the shortest amount of time, try some of
these proven recipes. For each recipe, mix the ingredients thoroughly
and follow the directions in the next section, “Keeping your pile happy.”
Depending on weather and compost ingredients,
you ●should have finished● compost within one to two months.
https://www.dummies.com/home-garden/gardening/making-compost-black-gold-for-your-organic-garden/

(2) Eligibility
●You should have:
achieved● or be expected to achieve at least a 2:2 degree classification
in any discipline by August 2019 (if you have already graduated,
you ★should have done★ so within the last 3 years)
https://www.gov.uk/government/publications/beis-graduate-and-experienced-entrants-programmes/join-the-graduate-or-experienced-entrants-programme
0171名無しさん@英語勉強中 (ワッチョイ b66c-sTnA)
垢版 |
2020/07/20(月) 16:46:51.45ID:XFX9prNX0
ジーニアス英和大辞典だとこういう説明がある

「・・してしまっているべきである」(単なる完了)の意味にもなる
You should have finished your homework by the next class.
次の授業までには宿題を済ませておくべきです
ただし you should finish your homework by the next classのほうがふつう
0172名無しさん@英語勉強中 (ワッチョイ b66c-sTnA)
垢版 |
2020/07/20(月) 17:01:00.85ID:XFX9prNX0
一冊の文法の本にすべての語法が載ってるわけじゃないし
学校英文法の知識がかえって固定観念になるケースもある

It seems to me most strange that men should fear:
Seeing that death, a necessary end,
Will come when it will come.
人が死を恐れるのが私には不思議でならぬ
死は必然の終わりで、
来るときには必ず来るものなんだから

seeing that = because
when it will come.  現代だと when it comes
0173名無しさん@英語勉強中 (ワッチョイW 9dbe-dBQw)
垢版 |
2020/07/21(火) 22:10:12.50ID:T66fkQ8r0
ここの人が一冊、文法書を進めるなら何ですか?
0174名無しさん@英語勉強中 (ワッチョイ 767b-YsWi)
垢版 |
2020/07/21(火) 22:13:10.06ID:AsD9Rc600
>>173
江川泰一郎『英文法解説』
0178名無しさん@英語勉強中 (ワッチョイ 0b6c-WQpp)
垢版 |
2020/07/22(水) 03:56:54.04ID:4PLYW3a80
ロックの統治二論の冒頭(second treatise of government)

Chap. I. Sect. 1. It having been shewn in the foregoing discourse,
1. That Adam had not, either by natural right of fatherhood, or by positive donation from God,
any such authority over his children, or dominion over the world, as is pretended:
2. That if he had, his heirs, yet, had no right to it:
3. That if his heirs had, there being no law of nature nor positive law of God that determines
which is the right heir in all cases that may arise, the right of succession,
and consequently of bearing rule, could not have been certainly determined:
4. That if even that had been determined, yet the knowledge of
which is the eldest line of Adam's posterity, being so long since utterly lost,
that in the races of mankind and families of the world,
there remains not to one above another, the least pretence to be the eldest house, and to have the right of inheritance:
以下略
https://history.hanover.edu/texts/Locke/J-L2-003.html

there remains not to one above another, the least pretence to be・・
ここの解釈
there remains not
<to one above another,> one 人 above 優越 ・・より上で、すぐれて
the least pretence pretence たぶん主語 「口実、言い訳」、not the least 成句 「少しも・・ない」
<to be・・・> pretenceにかかる
と解釈したんだが

<to one above another,>のtoは前置詞でremainとともに
remain to・・ 「結局・・のものとなる」「・・の手に帰する」「(事が)・・の手中にある」 でいんだろうかと考えている
remainを「・・のまま」ととってもいけそうな感じもするけどよくわからない

It remains to you to assist your friend. 友人を助けるのは君の役目だ
0179名無しさん@英語勉強中 (ワッチョイ 135a-8jMQ)
垢版 |
2020/07/22(水) 05:02:28.62ID:1Ozcuarv0
>>178
単に「残っている」ってことじゃないの?

A daughter remains to him.
なお存するは一人の息女 - 斎藤和英大辞典
0180名無しさん@英語勉強中 (ワッチョイ 2939-0S/1)
垢版 |
2020/07/22(水) 06:16:43.02ID:Gm7cAhfZ0
>>178
>>there remains not to one above another,
the least pretence to be the eldest house, and to have the right of inheritance

remains に関しては、>>179 の言う通りだと思う。pretense に関しては、
「口実、言い訳」と解釈しても意味が通じそうな気もするけど、もっと
適切なのは "a pretense to [be something]" で「〜であるという主張」
"a pretense to [have something]" で「〜を所有する権利を持っているという主張」
という意味合いではないかと俺は思う。

というのも、「口実」という意味で pretense を使う時には、そのあとに of などが
ついて名詞句が続くとしか辞書に書いていないのだ。この John Locke の文章に
おいては、pretense のあとに "to 不定詞" が続いているので、あくまでも
"pretense to [verb]" という形を辞書の中で確かめた方がよいと思う。
ただしそういう意味合いで使うのは、この下に示すように、現代では rare だそうだ。
以下の記述は、OED Online (OED Third Edition, 2007 より)

A. n.
1. a. An assertion of a right, title, etc.; the putting forth of a claim;
a claim. Now rare.
(この後の例文はたくさんあるが、そのうちの1つを取り上げる。)
1707 tr. P. Le Lorrain de Vallemont Curiosities in Husbandry & Gardening 186
No Man has..more ●Pretence to speak● of Nitre, than M. Boyle.

(続く)
0181名無しさん@英語勉強中 (ワッチョイ 2939-0S/1)
垢版 |
2020/07/22(水) 06:16:58.68ID:Gm7cAhfZ0
(続き)
ただし、これだけでは俺にとってはわかりにくい。だから、pretense という
名詞ではなくて、その本来の形(動詞)で "pretend to be, pretend to have"
という形ではどのような意味になるのかを、同じ OED にて確かめてみる。

4. b. transitive. With infinitive as object: to claim the right (to do something).
Also occasionally with clause as object. Obsolete.
(たくさんの例文のうち、"pretend to [verb]" という形を紹介したものは
次の2つだけ。)

(1) 1686 F. Spence tr. A. Varillas Ἀνεκδοτα Ἑτερουιακα 36
The deputy of the Ruffians ●pretended to receive● the full sum which
his accomplices had agreed upon.

(2) 1761 D. Hume Hist. Eng. I. ix. 204 As both the archbishops
●pretended to sit● on his right hand, this question of precedency begat
a controversy between them.

以上、古い用例ばかりだし、現代ではそういう意味で使うことは rare だとか
obsolete だと書いてあるが、それもそのはずで、John Locke は 17 世紀の後半に
この文章を書いたはずなので、こういう用例や定義文は適切だろうと思う。
0182名無しさん@英語勉強中 (ワッチョイ 2939-0S/1)
垢版 |
2020/07/22(水) 06:41:21.52ID:Gm7cAhfZ0
There remain to [someone] [something]. については、質問者と回答者との
お二人の意見に賛同する。一応、そのことをさらに確認するために、簡単そうな
例文をネットから拾い出してみる。前後関係を読み取れば、その意味合いは
明らかだろう。

(1) Without organization, ●there remain to you two solutions●. Either you
negotiate an exchange of services in various form with the person who
has competences which interest you, or you take time to acquire these
competences.

(2) ●There remain to you two brothers● of whom my son Ali is the elder.

(3) There remains to you the 3 and 5 action cards.

(4) And ●there remains to you the problem of persuasion● -- in your initial
building of your ladder; and in your argument.

(5) Nevertheless, aim at a size slightly too large: steal an empire and
●there will remain to you perhaps a kingdom●.

"There remains to [someone] [something]." は、わかりやすい語順にすれば
"There remains [something] to [someone]." だけど、そういうわかりやすい
語順になっている例文も、ついでに探してみる。

(6) ... so that ●there remains nothing to him● but his poor miserable person
already sufficiently burdened with age and infirmity, to discharge all
personal execution ...

(7) ... my dear friend, but I cannot bring myself to think you guilty of
infidelity to me: for ●there would remain nothing to me● in this world,
not even the affection, the love ...
0183名無しさん@英語勉強中 (ワッチョイ 2939-0S/1)
垢版 |
2020/07/22(水) 06:41:32.93ID:Gm7cAhfZ0
いずれにしても、"There remains to you two solutions." は少し堅い言い回しで、
"You have solutions (left)." と言えばわかりやすいんだろうな。だから
上の例文はすべて、学術関係とか宗教書など少し堅めの文章とか人生論めいた
文章の中でよく出てきそうな気がする。
0184名無しさん@英語勉強中 (ワッチョイ 2939-0S/1)
垢版 |
2020/07/22(水) 06:42:40.13ID:Gm7cAhfZ0
間違い: "There ●remains● to you two solutions."

remain と書くべきだった。
0185名無しさん@英語勉強中 (ワッチョイ 8b32-keh3)
垢版 |
2020/07/22(水) 10:49:50.66ID:aHPDEHg50
時制がよくわかっていないのですが、


I met her before I have talked to her son in my office.

彼女に会う前に彼女の息子と事務所で話したと両方過去の
話しなのですけど、息子にあった方が先です。この文章で
あってますか?
0187名無しさん@英語勉強中 (ワッチョイ 0b6c-keh3)
垢版 |
2020/07/22(水) 12:40:10.30ID:4PLYW3a80
178へのレスありがとう
参考になりました
0188名無しさん@英語勉強中 (ワッチョイ 0b6c-keh3)
垢版 |
2020/07/22(水) 12:54:20.88ID:4PLYW3a80
角田って人の訳らしい。わかりやすさのために若干意訳されてる。
われこそはアダムの直系の子孫であるとか、相続の権利を持っているとか言い立てることは、
いささかもできない。その点で万人は平等である。

pretence
ジーニアスに乗ってる一番近い訳語書いたんだけどより正確には指摘されたもののほうが正しい

A. n.
1. a. An assertion of a right, title, etc.; the putting forth of a claim;
a claim. Now rare.
0191名無しさん@英語勉強中 (ワッチョイ 135a-8jMQ)
垢版 |
2020/07/22(水) 15:37:32.73ID:1Ozcuarv0
ロックの市民政府二論はアメリカ独立宣言との絡みもあって
アメリカの大学の一般教養でも普通に読んだりする古典の定番。
日本でもホッブス、ルソーとセットで公民や倫理政経の授業で
お馴染みではある。

日本の古文だと文法だの語彙だの勉強しないと読むのはきついけど、
英語の古典はこのくらいの時代のものはそのまま頑張れば読めたりする
…まぁ日本の古文も江戸時代くらいのものはまだマシか。
0193名無しさん@英語勉強中 (ワッチョイ 8b89-0S/1)
垢版 |
2020/07/22(水) 17:18:24.77ID:2Ge8nd1e0
>>192
横から返事をするけど、
https://global.oup.com/academic/product/the-oxford-english-dictionary-9780198605553?q=oxford%20english%20dictionary&;cc=jp&lang=en
上のリンク先の右側の中央の白い箱の中を見たらわかるように、
英米向けなら安いけど、日本で使うとなるとけっこう高くて、
3か月単位か12か月単位かのどちらかだ。確かこれに 10% の消費税が上乗せされる。

ただし、今は特別に安い料金で提供されている。そのことは、次のような文章で
示されている。

We are pleased to offer annual individual OED subscriptions at a reduced rate
of $90 in the US (usually $295) or £90 for the Rest of the World
(usually £215) until 31st March 2021. Customers outside North and South
America, please used the promotional code OED90.
0194名無しさん@英語勉強中 (ワッチョイ 8b89-0S/1)
垢版 |
2020/07/22(水) 17:22:43.11ID:2Ge8nd1e0
>>190
OED Online を使う仲間ができて、実にうれしい。他にも少なくとも1人が
これを使っているらしい。確か半年くらい前に英英辞典スレに
OED Online を使い始めたばかりだという人に出会った。

ところであなたは、ちゃんと特別料金で OED Online を使ってる?
>>193 に書いてある通り、今年の3月までは普段はないような
特別の低価格で提供されてるからね。その特別価格を申請するための
code を入れないと、高い料金を請求されてしまうからね。
0196名無しさん@英語勉強中 (ワッチョイ d1dc-WQpp)
垢版 |
2020/07/24(金) 17:11:30.16ID:ZSdeKMNl0
>>134
会議通訳者の意見

>Go to travelと言ったら、「移動に行く」と言うのと同じで、言いたいことはわかるけれど、おかしな響きがあります。

>Go travelはI went traveling.と言えますから、正しい表現ではあります。ただ、まず聞かない表現です。
>正しいからと言って、ネイティブスピーカーが聞いて「自然な表現」と判断するとは限りません。

「GoToキャンペーン」は英語としてヘン。プロ通訳が苦笑
https://news.nicovideo.jp/watch/nw7689900
0197名無しさん@英語勉強中 (ワッチョイ 7bc5-It4W)
垢版 |
2020/07/25(土) 14:03:43.91ID:iAzotxv/0
Nobody she knew spoke English well, and neither her parents nor anyone she knew
really used English or math in their jobs.

この文章でshe knew spoke と主語+動詞過去形+動詞過去形と続いてる文法がわかりません。

nobody knew she spoke 〜の間違いなのでしょうか?
高認過去問なので間違いの可能性は低いと思ってはいますが。
Nobody knew she spoke 〜の場合との意味的な違いはあるのでしょうか?
0202名無しさん@英語勉強中 (ワッチョイ 8b89-0S/1)
垢版 |
2020/07/27(月) 17:28:52.08ID:+CqS3M5l0
abscd
0203名無しさん@英語勉強中 (ワッチョイ 8b32-keh3)
垢版 |
2020/07/28(火) 14:54:08.33ID:w/s5I/iA0
>>189

ネットが半分故障してしまって遅くなりました。
有難うございます。知人がbefore の後の文章は
現在形だとおしえてくれたのですが、やっぱり
両方過去形なんですね。THANK YOU
>>189
0204名無しさん@英語勉強中 (ニククエ d632-QWST)
垢版 |
2020/07/29(水) 12:05:54.59ID:S4Qa7L1C0NIKU
There are trees all down my street.

という文章があったのですが、streets

ではないのでしょうか?全ての通りなので
複数じゃないですか?
0207名無しさん@英語勉強中 (ニククエ d632-QWST)
垢版 |
2020/07/29(水) 12:50:10.63ID:S4Qa7L1C0NIKU
>>205

彼女の住んでいる町のイメージの説明
だったのでてっきり複数化と思いました。
有難う御座います。
0208名無しさん@英語勉強中 (ニククエ d632-QWST)
垢版 |
2020/07/29(水) 12:50:51.41ID:S4Qa7L1C0NIKU
>>206

そういう意味だったのですね。
有難う御座いました。
0209名無しさん@英語勉強中 (ワッチョイW 7ae3-m1rq)
垢版 |
2020/07/30(木) 12:35:13.21ID:YmYT6hW10
There is a necessary conflict of interests in our present civilization between the prosperous and employing classes of people and the employed mass.

訳:現在の文明においては、富裕な使用者階級と使用されている大衆との間に必然的に利害の衝突が存在する。

参考書に載っている上の英文を英文校正ツールGrammarlyにかけたところ、interestsを単数形に直せと指摘されましたが、どちらが正しいか分かりません。
どうやらagreement mistake(数の一致のミス?)を犯しているようです。
0213名無しさん@英語勉強中 (ワッチョイW 7ae3-m1rq)
垢版 |
2020/07/30(木) 13:53:02.34ID:YmYT6hW10
>>210
>>211
ご返信ありがとうございます

利害関係のinterestはC,Uどちらも可能で、Cの場合は相互複数(e.g. I am friends with him.)を念頭に複数形で使われるということですね。Uの場合は両者間で生じうる利害関係一般、、、みたいなニュアンスでしょうか

とにかくどちらでもよいと分かって安心しました
0214名無しさん@英語勉強中 (ワッチョイ fa60-UdrD)
垢版 |
2020/08/02(日) 02:53:35.22ID:g6z8EJUT0
"YOU ARE THE CLOSEST I WILL EVER COME TO MAGIC." —SUZANNE FINNAMORE
google 「あなたは私がマジックに来る最も近い場所です。」 —スザンヌフィナモレ

これはどういう意味?
「私にいつか奇跡が起こるとしたらあなたがもっとも可能性がたかい。」が直訳とかんがえて
すこし意訳すると
「私に奇跡を起こすとしたらあなたしかいない。」 でよいか?
0215名無しさん@英語勉強中 (ワッチョイW 8e6c-TWrq)
垢版 |
2020/08/02(日) 06:37:20.71ID:hqJkEHe20
I marvel why she should marry such a man.
彼女がなぜそんな男と結婚したのか不思議でならない

というのはオーレックスのmarry の項目の例文です。
should は感情を表す時に使い、今の場合、なぜこんな奴と結婚してるのか、
に驚く、というのが上の訳になったものと思いますが、
状態動詞の意味を持たせず、結婚するという動作動詞の意味を持たせるとすれば
marry じゃダメで、get marriedにするのが妥当でしょうか?

それとshouldの意味ですが、仮定法なので、万が一、という意味が込められていると思う
のですが、感情をshould を伴うthat 節で表す理由は、思いもかけないことがthat節内で来て、
それが万が一という意味のshould で表せるからということでしょうか?

最後に、こんな奴と結婚するべきなのか、驚く

ということを言いたい時にも上の英文になるでしょうか?
0216名無しさん@英語勉強中 (ワッチョイ d689-ANno)
垢版 |
2020/08/02(日) 08:00:07.02ID:kcO8K2cS0
>>215
(1) I marvel why she should marry such a man.
>>なぜこんな奴と結婚してるのか、に驚く
>>状態動詞の意味

marry は、状態動詞じゃなくて動作動詞だと思う。
「結婚している」と言いたいのなら、she is married with ... になる。
I marvel why she should ●be married with● such a man.

(2) should の意味は、「万が一」という意味はないと思う。
辞書のどこかに書いてあるはずだけど、why などの wh- 疑問詞に should とか
would をつけると、「意外だ」という感情だったか何だったか忘れたが、
ともかく話者のそういう気分を表すのだ。

(3) こんな奴と結婚するべきなのか、驚く

そのような意味の英文を書きたいときには、もしももともとの英文の
表現をできる限り生かすとしたら、

I marvel why she should have to marry such a man.
とでも書けばいいのだろうと思う。
0217名無しさん@英語勉強中 (ワッチョイ d689-ANno)
垢版 |
2020/08/02(日) 08:05:23.28ID:kcO8K2cS0
>>216 の (3) で俺は "why ... should have to" と書いたが、これを使った用例をネットから
拾ってみる。

(1) ●Why everyone should have to● work in food service.

(2) ●Why everyone should have to● learn computer programming.

(3) Many clients wonder ●why they should have to● pay such a fee, particularly
when they are sometimes non-refundable and they have not yet received any benefit ...

(4) ●Why Plaintiffs Should Have to● Prove Irreparable Harm in Copyright
Preliminary Injunction Cases.
0218名無しさん@英語勉強中 (ワッチョイ fa32-8P+6)
垢版 |
2020/08/02(日) 15:11:06.55ID:xEiYGUWn0
>>216
be married withは間違い
0219名無しさん@英語勉強中 (アウアウウー Sa09-eoeL)
垢版 |
2020/08/02(日) 16:15:39.43ID:37g3bhnRa
子連れか一夫多妻になるね
0220名無しさん@英語勉強中 (ワッチョイ d689-ANno)
垢版 |
2020/08/02(日) 16:38:44.68ID:kcO8K2cS0
>>218
あっ。そうか、また間違ってしまった。俺、いつもここで間違うんだよね。
I'm married. だけですでに「結婚している」という意味になってしまうよね。
I'm married with a child. なら、「配偶者がいて、子供が1人いる」というわけだな。

>>216
>> I marvel why she should ●be married with● such a man.

というわけでこの上に書いたものは間違いで、「そんな男と結婚しているなんて驚き」
と言いたいときには、次のようになる。
I marvel why she should ●have married● such a man.

だから、たとえば「私は馬鹿な男と結婚している」とか「私の旦那は馬鹿だ」と
言いたいときに、次のように言う。
I married a fool.

この I married a fool. というのは、Charles Dickens の "Oliver Twist" の
(白黒の古い)映画化作品の中で、馬鹿な旦那としゃべっていて「ほんとに私は馬鹿な男と
結婚してしまったもんだ」という時に言っているセリフ。
0221名無しさん@英語勉強中 (ワッチョイW 8e6c-TWrq)
垢版 |
2020/08/03(月) 03:56:45.91ID:9GNdTTLe0
>>216
およびその他の方々

よく分かりました。be married toという書き方もありますね。
過去分詞のmarriedで状態を表し、能動態では動作ですね。


万一〜ならば」という意味で,そういうことが起きるか起きないかはわからないが,話し手は可能性が少ないと思っている場合に用いる。絶対に起こり得ないことの仮定には使わない。
帰結節には,過去形の助動詞ばかりでなく,意味内容によっては,現在形の助動詞や命令形を用いることもある。
If I should lose you, the stars would fall from the sky.
(もしもあなたに死なれたら,星が空から落ちてくる)

とはロイヤル黄色に書いてありました。

教えていただいたように、なになにするなんて意外だという説明がもとの英文の
解釈としてしっくりきます。
他方で万が一というのも起きる可能性の少なさを念頭に置いていると
考えるとすれば、どの程度に意味の拡大を認めるかということかなと思いました。
教えてくれてありがとう
0222名無しさん@英語勉強中 (ワッチョイ d689-ANno)
垢版 |
2020/08/03(月) 14:49:39.06ID:afMGMKJb0
abc
0223名無しさん@英語勉強中 (ワッチョイ fa32-UdrD)
垢版 |
2020/08/03(月) 22:26:44.88ID:ZLkmZNyq0
I have been wanting to go to Nagano.

カナダの人に、want に ing はつけたら
だめなんですけどーといわれたのですが、
そうなんでしょうか!?
0225名無しさん@英語勉強中 (ワッチョイ fa32-UdrD)
垢版 |
2020/08/03(月) 23:29:47.61ID:ZLkmZNyq0
>>224

有難うございます。テストではダメですね。
0226名無しさん@英語勉強中 (アウアウウー Sa09-gj0H)
垢版 |
2020/08/03(月) 23:44:28.11ID:tr1YwAHRa
記述文法を学べる文法書や問題集はありますか?
0227名無しさん@英語勉強中 (ワッチョイ d689-ANno)
垢版 |
2020/08/04(火) 10:43:09.02ID:P5ybzY+c0
want 歯状態動詞(stative verb)だから進行形にするな、と言っている英語ネイティブも
いる。しかし現実には大いに使われているわな。でも英語の下手な俺たち日本人は
使わない方がいいわな。

I have wanted と I have been wanting との微妙な使い分けについて
https://ell.stackexchange.com/questions/39279/i-wanted-have-wanted-was-wanting-to-be-a-doctor-since-i-was-ten
0228名無しさん@英語勉強中 (ワッチョイ d689-ANno)
垢版 |
2020/08/04(火) 10:51:52.79ID:P5ybzY+c0
want ●歯● 状態動詞だから

訂正
want ●は● 状態動詞だから
0229名無しさん@英語勉強中 (ワッチョイ d689-ANno)
垢版 |
2020/08/04(火) 10:57:04.34ID:P5ybzY+c0
「want の進行形の使い方が正しいかどうかについては、文脈によるのだ」と
言っている英語ネイティブらしき Forero という人が、次のように言っている。
まあ俺たちは無理に進行形を使わず、want は進行形にしないで使えばいいだけだが。

The appropriate tense really depends on context, both on what else is being said and on what you want to express.

I see nothing at all wrong with using "I was wanting" in a context such as:

At three o'clock yesterday, I was wanting a cup of coffee, but all they were
able to offer me was decaf.

The idea here is that, come three o'clock yesterday, I was feeling the need
for real coffee but they were not finding any for me. Because the meaning
is "was feeling the need" rather than something like "had an urge" or
"ordered", "was wanting" is perfectly appropriate.

俺自身は、小説や英米のドラマや映画の中で、状態動詞 (want, love, like,
guess など)の進行形に気づくことが最近は増えてきた。昔から大いに
使われていたはずだが、昔は俺の英語の聴き取り能力(理解能力)が低かったから、
まだ「状態動詞の進行形」を聴き取って理解してはいなかったに過ぎないのだろうと
思うけどね。
0230名無しさん@英語勉強中 (ワッチョイ d689-ANno)
垢版 |
2020/08/04(火) 11:10:58.48ID:P5ybzY+c0
"I think" という意味で通常は "I guess" というところを、Friends という
例の有名な sitcom の中で Ross が "I'm guessing" と言っていたので、
面白かった。

Ross: So, ●I’m guessing● you had an extra ticket and couldn’t decide
which one of you got to bring a date?

Chandler: Well, aren’t we Mr. “The glass is half empty.”

上のセリフは Episode 4 からのものらしいが、数か月前に YouTube ではっきりと
その場面を見たのに、今はなくなってしまっている。
0231名無しさん@英語勉強中 (ワッチョイ d689-ANno)
垢版 |
2020/08/04(火) 11:13:53.48ID:P5ybzY+c0
"be wanting" よりも、
(1) I'm loving
(2) I was hoping
(3) I was wondering
(4) You are forgetting
の方なら、もっと頻繁に見聞きするような気がする。
0232名無しさん@英語勉強中 (ワッチョイ d689-ANno)
垢版 |
2020/08/04(火) 11:21:20.60ID:P5ybzY+c0
forget も状態動詞だが、you are forgetting はときどき見聞きする。現代英語において
のみならず、「罪と罰」(Crime and Punishment)を1914年にイギリス女性が
翻訳したときの文章にさえ出てくる。100年も前のイギリス英語に出てきているのだ。

"Well, then, I’ll tell you, your fiance is a scoundrel.”

“Excuse me, Mr. Razumihin, ●you are forgetting●...” Pulcheria Alexandrovna
was beginning.

“Yes, yes, you are right, I did forget myself, I am ashamed of it,”
Razumihin made haste to apologise.

上の一節では、酔っぱらってしまっている Razumikhin が(主人公 Raskolnikov の
妹である)Dounya の fiance である Luzhin の悪口を言っているので、
Dounya の母親である Pulkherya Alexandrovna Raskolnikova が
「ちょっと、言葉がひどすぎないでしょうか?」という意味で、
Excuse me, you are forgetting.... と言っている。
0234名無しさん@英語勉強中 (ワッチョイ d689-ANno)
垢版 |
2020/08/04(火) 13:50:58.39ID:P5ybzY+c0
>>233
そうそう、その通りだと俺は思っている。
(1) You've forgotten yourself.
(2) You forget yourself.
上の2つの言い方なら、「あなたは完全に自制心を失ってしまって、
言ってはいけないことを言っている」という、いわば断定口調になってしまうから、
それを避けてあえて現在進行形にすることにより、「完全に自制心を失っている
わけじゃないけど、今だけたまたま自制心を失いかけている」というふうに
口調を和らげているのだろうと思う。

I was wondering も同じで、「ちょっと少しだけ(これこれこうじゃないかな)
と考えていたんだけどね」というふうに相手に対して柔らかく話を持ち掛けている
言い回しだと思う。

I was hoping もやはり、I hoped that you would help me. とか
言ってしまうと、相手にどうしてもそうしてほしいと押しつけがましく聞こえてしまう
かもしれないと恐れて、「いやね、ちょっとだけね、あなたがそうしてくれないかな、
なんて思ってたんだけどね、いやほんの一時的にそう思っただけだったんだけどね」
という口調になると思う。

なお、今回の You're forgetting... はおそらく、You're forgetting yourself.
と言いたいところを、最後まで言わないで終わっているのだろうと思う。
最後まで言わないからこそ、余計に控えめな言い回しになっているのだろう。

https://forum.wordreference.com/threads/youre-forgetting-yourself.3683140/
上記のサイトでは、有名な例の Shawshank Redemption (ショーシャンクの空の下で)
という映画の中で出てくる You're forgetting yourself. という台詞がどういう意味
なのかを説明してくれている。
0235名無しさん@英語勉強中 (ワッチョイ d689-ANno)
垢版 |
2020/08/04(火) 13:59:53.20ID:P5ybzY+c0
>>230 で取り上げた Friends という sitcom の中での Ross のセリフ
Ross: So, ●I’m guessing● you had an extra ticket and couldn’t decide
which one of you got to bring a date?

これも同じ理屈であり、I guess you had an extra ticket... ならば普通の言い方であり、
「これこれこうだろうと思う」つまり「これこれこうなんだろう?」と、少しばかり
決めつけた言い方になってしまうが、それをさらに和らげて I'm guessing と
言うことにより、「君たちがこれこれこういうことを考えてるん
じゃないかと、僕は今のところ考え始めてるんだけどね、いや、間違ってるかもしれ
ないけど〜」という感じになると、俺は今のところ思っている。
I guess ではなくて I'm guessing という表現は、Friends において何度も使われている。


Friends, S.1 Ep.1 The One Where Monica Gets a New Roommate (Part 2)

Monica: Now ●I’m guessing● that he bought her the big pipe organ, and she’s
really not happy about it.

Rachel: Daddy, I just... I can’t marry him! I’m sorry. I just don’t love him.
Well, it matters to me!

上の "I'm guessing" は、みんなでスペイン語によるメロドラマを見ていて、
スペイン語だからあまりよくわからず、「いま二人が言っているのはこれこれこういう
ことだと思う、間違ってるかもしれないけど」というようなことを言っているのだと
思う。つまり Monica はスペイン語がわからずドラマの筋がわからないが、
たぶんこんなことを言ってるんじゃないかな、間違ってるかもしれないけどね、
というニュアンスで I'm guessing と言っているのだと思う。I guess よりも
さらに自分の言っていることに自信がない時に使う表現なんだろうと俺は思っている。
0237名無しさん@英語勉強中 (ワッチョイ d689-ANno)
垢版 |
2020/08/04(火) 14:14:49.79ID:P5ybzY+c0
>>230 のセリフが出てくる場面を、やっと見つけた。

https://www.youtube.com/watch?v=ggZkZK-9Pm4&;t=19m9s

上のリンク先の、19m9s のところ。
0238名無しさん@英語勉強中 (ワッチョイ d689-ANno)
垢版 |
2020/08/04(火) 14:23:33.66ID:P5ybzY+c0
Grace Casselman - 2005 - ‎Juvenile Fiction
“●So, I was thinking you might● like to pick the colour for your room.”
She shrugged. “I don't care.” “Come on. You must have some thoughts...pink?” “No!

上のような、"So I was thinking you might..." なんていう表現も、俺は特に意識して
いなかったので聞き逃してしまっていたかもしれないが、英米のドラマや映画の
中で大いに出てきそうだわな。
0239名無しさん@英語勉強中 (ワッチョイ d689-ANno)
垢版 |
2020/08/04(火) 14:26:49.33ID:P5ybzY+c0
Won't you tell me to my face
If I'm bound to be replaced
★I've been thinking you might★ be the one
Won't you show me where you're coming from

上の一節は歌詞の中の文句だが、これによく似た言い回しはあちこちで
使われているだろうな。
0240名無しさん@英語勉強中 (ワッチョイ d689-ANno)
垢版 |
2020/08/04(火) 14:33:25.05ID:P5ybzY+c0
"I've been wanting to (動詞)." がまったく正しい表現だと言っている英語ネイティブも
いる。
https://nic-english.com/phrase/%E6%9C%80%E8%BF%91%E3%80%81%E8%A1%8C%E3%81%A3%E3%81%A6%E3%81%BF%E3%81%9F%E3%81%84%E3%81%A8%E6%80%9D%E3%81%A3%E3%81%A6%E3%81%84%E3%81%9F%E3%80%82-ive-been-wanting-to-check-it-out/

この上のリンク先の解説は、Nick という名前のオーストラリア人で、この人は
YouTube 上で映画の聴き取りのコツを教え続けてくれている人だ。
俺自身も I've been wanting to do... だけじゃなくてこれに似た言い回しに
たくさん触れたことがあるので、この人の解説はたぶん正しいと思っている。
まあ、正しいと思うかどうかは、人それぞれ好きなように判断すればいいと思うけど。
0241名無しさん@英語勉強中 (ワッチョイ d689-ANno)
垢版 |
2020/08/04(火) 14:38:51.16ID:P5ybzY+c0
(1) I was wondering if...
(2) I've been wondering if...

上の二つの表現の微妙な意味の違いについては、次のリンク先に解説がある。
このような "be wondering" について理解すると、次に "be wanting" の用例の
意味合いも理解しやすくなるだろうと俺は思っている。

https://forum.wordreference.com/threads/i-was-wondering-ive-been-wondering.3507474/
0242名無しさん@英語勉強中 (ワッチョイ d689-ANno)
垢版 |
2020/08/04(火) 14:46:24.35ID:P5ybzY+c0
https://forum.wordreference.com/threads/i-have-been-wanting.1386491/

この上のリンク先にいる英語ネイティブたちもやはり、
I've been wanting to [do]
が正しいと言っている。それだけでなく、口語においては次のようなものも
(正しくないかもしれないがときどき)使われていると言っている。

(1) "Were you wanting to use this?"
(2) "Are you wanting to use this at some point today?"
(3) "I hope you're not wanting to use the photocopier today."
0243名無しさん@英語勉強中 (ワッチョイ cd32-IsMZ)
垢版 |
2020/08/04(火) 15:33:41.37ID:pwbcE1Ph0
そういう個人感想レベルの話じゃなくてちゃんとしたサイトから引用しようよ

ケンブリッジの英英から
https://dictionary.cambridge.org/grammar/british-grammar/want

■Want in the continuous form(進行形におけるwant)
(1)We can use want in the continuous form to show indirectness or politeness:
  (婉曲的もしくは丁寧さを示したい場合にwantを進行形で用いることが可能)

Customer:
We’re wanting to buy a new TV, but we’re not sure what to get.
(新しいテレビを買おうかなと思っているところなんですけど… どれを買っていいものか決めかねておりまして)
Assistant:
Okay, sir. Let me show you some of them.
(それでしたらいくつかお見せしますよ)

I was wanting to ask you something. Are you free right now?
(ちょっとお尋ねしたいのですが… 今、お時間あります?)

(2)We can also use the continuous form to emphasise an ongoing or repeated process:
  (目下進行中もしくは反復される過程を強調したいときにもこの形を用いることが可能)

We’d been wanting to go to New Zealand for years, so his sixtieth birthday was a good excuse.
(ここ何年かずっとニュージーランドへ行こうと思ってるんだ。彼の60歳の誕生日はいい口実だった)

Now that she’s a teenager she’s wanting expensive things, you know, computers, clothes, sports stuff.
(もう、あの娘も高価なものに憧れるティーンエイジャーなのよ。PCとか洋服とかスポーツ用品とかね。わかるでしょう)
0244名無しさん@英語勉強中 (ワッチョイ d689-ANno)
垢版 |
2020/08/04(火) 16:55:45.09ID:P5ybzY+c0
>>243
偉そうに他のサイトでのいろんな人の意見にケチをつけておきながら、
お前が引用している Cambridge のサイトに書いてあることは、
お前が軽蔑している「個人感想レベル」のコメントで言いつくされていること
のうちの一部でしかないだろうが。
0246名無しさん@英語勉強中 (ワッチョイ d689-ANno)
垢版 |
2020/08/04(火) 18:36:09.41ID:P5ybzY+c0
xwa
0248名無しさん@英語勉強中 (ワッチョイ d689-ANno)
垢版 |
2020/08/04(火) 18:36:29.09ID:P5ybzY+c0
xyz
0249名無しさん@英語勉強中 (ワッチョイ d689-ANno)
垢版 |
2020/08/04(火) 18:36:54.47ID:P5ybzY+c0
a

a
a

gh
0250名無しさん@英語勉強中 (ワッチョイ d689-ANno)
垢版 |
2020/08/04(火) 18:42:20.73ID:P5ybzY+c0
(1) I've been wanting to [do].
これに似た形をしていて、しかも意味も似ている言い回しを一つ思い出した。
確かテレビドラマか何かの中に出てきた台詞だったと思う。

(2) I've been meaning to do [do].

この言葉の意味合いについては、この下のリンク先でネイティブたちが説明してくれている。

https://forum.wordreference.com/threads/ive-been-meaning-to.682074/
0251名無しさん@英語勉強中 (ワッチョイ 93f0-mvG/)
垢版 |
2020/08/05(水) 04:19:05.88ID:ig8S71qV0
次のlook upon の語法が分かりません。

Such dishonesty is looked upon strictly both in journalism and academics.
そのような不正行為はジャーナリズムにおいても、学問的環境においても厳しく見なされている。
look upon A as C の受け身かと思ったのですが、as C の部分がないし、strictly も副詞なので違う語法のようです。
となれば、look upon (on) A で一つの熟語かと思ったのですが、辞書には載っていなかったです。
分かりやすい解説をお願いします。
0252名無しさん@英語勉強中 (ワッチョイ eb89-HICl)
垢版 |
2020/08/05(水) 09:25:09.23ID:B1mNtP0v0
>>251
look upon [something]
look upon そのものが見つからなかったら、look on [something] を見ればいいんだよ。
そもそも upon は on という意味で昔はよく使われることが多かったんだが、
現代では upon をあまり使わず、その代わりに on を使うようになっただけなんだ。
0253名無しさん@英語勉強中 (ワッチョイ eb89-HICl)
垢版 |
2020/08/05(水) 09:27:20.97ID:B1mNtP0v0
>>251
>>look upon (on) A で一つの熟語かと思ったのですが、辞書には載っていなかった

すまん、あなたは look on A もきちんと調べようとしていたんだな。
それなら、別の辞書を見てみたらいい。少なくとも「ジーニアス英和、第5」には
載っているから、その他の有力なオンライン辞書や紙の辞書に大いに載っているだろう。
0254名無しさん@英語勉強中 (ワッチョイ eb89-HICl)
垢版 |
2020/08/05(水) 09:57:42.59ID:B1mNtP0v0
>>251
>> Such dishonesty is looked upon strictly both in journalism and academics.

この英文は、どこに出ていたのかな?俺にはこれが奇妙な英文に見える。
通常は、このようなフレーズが並んでいたら次のようにでも書くのが普通ではないかと
思うのだが、それは俺の気のせいかな?

Such dishonesty is looked upon strictly ●as a taboo●
both in journalism and academics.

まあ、英語ネイティブにでも尋ねてみてほしい。俺は英語ネイティブに尋ねる
つもりはない。

もし俺の直感が正しければ、この原文よりも長いものがもともとあって、
予備校の教師か大学教授か何かがその英文を適当に短くして、それを問題集の中に
入れたんだろうという気がする。
0256名無しさん@英語勉強中 (ワッチョイ eb89-HICl)
垢版 |
2020/08/05(水) 11:22:49.43ID:B1mNtP0v0
>>254 にて、俺は原文が奇妙で、通常なら次のようにでも書くものだろうと言った。

>> Such dishonesty is looked upon strictly both in journalism and academics.

Such dishonesty is looked upon strictly ●as a taboo●
both in journalism and academics.

ここで一つ見逃していた。"journalism and academics" というのも変だ。
というのも、journalism とは「ジャーナリズムの世界」とか何とかいう意味で、
抽象名詞だわな。それに and をつけて academics というのは変だと思う。
academics とは "an academic" の複数形であり、「学者たち(大学教授たち)」という
意味での具体的な意味を表す言葉であって、抽象名詞ではない。このように
"抽象名詞 and 具象名詞の複数形" というふうな並列は変だと俺は思うのだが、
違うかな?俺はここの部分も、次のように書き換えたいところだ。

(1) Such dishonesty is looked upon strictly ●as a taboo●
both in journalism and ★academia★.

または

(2) Such dishonesty is looked upon strictly ●as a taboo●
both ★among journalists and academics★.
0257名無しさん@英語勉強中 (ワッチョイ 93f0-mvG/)
垢版 |
2020/08/05(水) 20:30:01.73ID:ig8S71qV0
ジーニアスを見てきましたが、なんか納得できる記述はなかったです。
look on A Aを傍観する、見物する。
(ある感情で)みる。regard (with) と書いてあり、一瞬下の記述かなと思ったのですが、
「dishonesty を見る」って変な感じがします。
regard A with B のような形なら、Aにdishonestyは来ないだろうし…。
0259名無しさん@英語勉強中 (ワッチョイ a960-Ea0s)
垢版 |
2020/08/06(木) 07:22:46.91ID:GgPbL1mP0
If you have a specific process for preparing to read,
then your brain will know when you’re about to read
and you’ll be more focused before you even start.

whenの訳し方がわからない
「いつ」じゃ意味が通らない。 「いま」で訳したほういいのだが、、、
つまり
「脳はいまが読む時だと知るだろう。」でいいだろうか? 意味的にはこうだろう。
この時whenは関係代名詞? そんな文法あり?
0261名無しさん@英語勉強中 (ワッチョイW 9bc5-Pr7b)
垢版 |
2020/08/06(木) 08:31:51.68ID:pLnOkJPs0
いつなになにか、だよ。名詞節を作る接続詞。
that に時のニュアンスが加わったみたいなものですよ

他人を揶揄するだけのチンケな人を見た時に、
無視するのが大人の態度なのだろうが、
チンケにはチンケと言ってあげるのも親切なことだからね。
あえて書いたんだよ
0264名無しさん@英語勉強中 (ワッチョイ 0be3-2X+j)
垢版 |
2020/08/06(木) 10:35:02.49ID:7Xf+SLbt0
以下Genius第5版より引用:

[whenの品詞の違い]
Tell me when you're ready.とTell me when you'll be ready.は意味が違う。
前者のwhenは接続詞で「用意ができたら教えてください」、
後者のwhenは疑問詞で「いつ用意ができるか教えてください」の意。
willの有無に注意。
0265名無しさん@英語勉強中 (ワッチョイ 0be3-2X+j)
垢版 |
2020/08/06(木) 10:39:05.55ID:7Xf+SLbt0
ランダムハウス英語辞典でも、間接疑問のwhenは「疑問副詞」に分類されている。

I ((疑問副詞))
【3】((間接疑問の名詞句[節]を導いて)) いつ:
I know when to be silent. いつ沈黙を守るべきかを知っている
Do you remember when and where you saw the picture? いつ,どこでその絵を見たか覚えていますか.
0266名無しさん@英語勉強中 (ワッチョイ 0be3-2X+j)
垢版 |
2020/08/06(木) 10:46:26.96ID:7Xf+SLbt0
COBUILDだと接続詞扱いになってた

5 [CONJ]
You use when after certain words, especially verbs and adjectives, to introduce a clause where you mention the time at which something happens.
 ・ I asked him when he'd be back to pick me up...
 ・ I don't know when the decision was made...
0268名無しさん@英語勉強中 (ワッチョイ a960-Ea0s)
垢版 |
2020/08/06(木) 11:38:46.86ID:GgPbL1mP0
>>261
>>264
なるほど、接続詞なんだね。
ありがとう。
0269名無しさん@英語勉強中 (ワッチョイ a960-Ea0s)
垢版 |
2020/08/06(木) 11:43:20.65ID:GgPbL1mP0
>>260
それ苦手って言葉ちがうんじゃない? 強いて言えば下手、超初心者とかだね。
とにかくできるできないは別にして全然苦手じゃあないよ。 
言語感覚が鋭い人は色々疑問が湧くんじゃあるまいか? 鈍い人は疑問はわかないと思う。
普通の人はまあ鈍いのだろうかね。それだと疑問には思わないと思う。
0271名無しさん@英語勉強中 (ワッチョイ ebb5-f2iR)
垢版 |
2020/08/06(木) 12:40:20.57ID:N/FzZhQt0
通常、間接疑問のwhenは(「いつ」という意味から)疑問副詞扱いだと思うが、
COBUILDが接続詞扱いなのは、例えば次の接続詞if(whether)の働きと同じと考えてるからなのかな。

I asked her if(whether) she could heip me.
I asked her when she could help me.
0272名無しさん@英語勉強中 (ワッチョイ 0be3-2X+j)
垢版 |
2020/08/06(木) 12:50:14.48ID:7Xf+SLbt0
COBUILD以外で、間接疑問のwhenその他を「接続詞」としている文献・辞書があったら御教示願います。

OALDでは間接疑問のwhenの用例が出ていない(!)のですが、whereには次の用例がありました。
I wonder where they will take us to.
分類はadverbです。
0273名無しさん@英語勉強中 (ワッチョイ a960-Ea0s)
垢版 |
2020/08/06(木) 13:31:09.71ID:GgPbL1mP0
https://dictionary.cambridge.org/grammar/british-grammar/when

Whenは接続詞のときは
‘at the time that’.
‘considering that’

When as a conjunction
We use when as a conjunction meaning ‘at the time that’.
The clause with when is a subordinate clause (sc) and needs a main clause (mc) to complete its meaning.
If the when-clause comes before the main clause, we use a comma.


Talking about the future
In references to the future with when, we use the present simple or the present perfect in the when-clause, not the future with shall and will:

When the new park opens, I’ll go there every day.
Not: When the new park will open, I’ll go there every day. これは間違い

When I’ve finished my homework, I’m going to phone Marita.
Not: When I’ll finish my homework, I’m going to phone Marita. これも間違い

We can use when as a conjunction to mean ‘considering that’:
What’s the point in going out when we have to be home by eleven o’clock?

総合的に判断すると
If you have a specific process for preparing to read,
then your brain will know when you’re about to read
and you’ll be more focused before you even start.

脳はいつ読む準備ができたかを知ることになるだろう。そして読書を開始する前にはもっとフォーカスするだろう。
 でやはり疑問詞かな。
もし接続詞だとするとknowの目的語がない。
0275名無しさん@英語勉強中 (プチプチ eb89-HICl)
垢版 |
2020/08/08(土) 14:45:49.82ID:YvUg6J8M00808
>>230-243
上のレスにて出てきたような、通常なら単純現在形あるいはそれに似たような
形を使っていたはずなのに、最近では "I'm ...ing" と現在進行形にした例を
最近は盛んに俺は見るような気がするのだが、今日もやっぱり出てきた。
ネットでの個人の文章だ。

Actually saw this car in person at Richard's warehouse during a private tour
last April. Highly unlikely he's having an estate sale, he's very savvy with
his collection and ●I'm betting● he sold it recently because he received an offer
he could not pass up.
https://forum.amcorner.com/threads/richards-desoto-for-sale.17573/#post-206063
0276名無しさん@英語勉強中 (プチプチ eb89-HICl)
垢版 |
2020/08/08(土) 14:54:48.66ID:YvUg6J8M00808
>>275 では、I bet S V とか I'll bet S V の形で見ることが昔は多かったように
思うが、俺としては I'm betting S V の形を初めてみたので、面白く思った。
毎日のように英語ネイティブと気楽に会話している人にとっては、こんなものには
毎日接しているだろうけどね。

上に出てくる投稿のすぐ10行ほど後には、次のようなものが出てくる。

●I was hoping that● he could've opened up the place during the 50th
Anniversary celebration in April 2019 but that wasn't going to happen!

この I was hoping that S V. なら、大昔からよく見るものなので珍しくも何とも
ないけどね。この上に出てきた I'm betting だけは、俺としては珍しく感じたのだ。
昔よりもはるかに現在進行形を使う人が増えてきたのかもしれないと思っている。

ただ、俺はもともとは会話文じゃなくて正式な学術書とか文学書を読むことがはるかに
多いので、そういう気がしているだけかもしれない。
0277名無しさん@英語勉強中 (ワッチョイ eb89-HICl)
垢版 |
2020/08/10(月) 14:44:32.10ID:SIdzNBe60
uuu
0278名無しさん@英語勉強中 (ワッチョイW 13f0-7U9h)
垢版 |
2020/08/10(月) 18:15:45.50ID:7GYqJmit0
the shimamami kaido is a very beautiful
areas of japan that i highly suggest you visit on your next trip here if possible.

アメリカ人のYouTuberが話してた一文ですが、分からないことがあります

文中にあるthatは接続詞ですよね?that以降の文をみても関係代名詞では無さそうですし…

このthatの用法を教えてください宜しくお願いします
0279名無しさん@英語勉強中 (ワッチョイ 0be3-2X+j)
垢版 |
2020/08/10(月) 18:27:28.56ID:6jUR4uEF0
>>278
関係代名詞でしょう。that以下で、visitの目的語が欠けてます。

2つの文に分けて書くと
The Shimanami Kaido is a very beautiful area of Japan.
I highly suggest you visit *it* on your next trip here if possible.
0283名無しさん@英語勉強中 (ワッチョイ 9bc5-VtC7)
垢版 |
2020/08/10(月) 20:47:56.43ID:viV3yz5Z0
>>281
教えてくれてありがとう

この橋ができたから、本州側から島への渡し船に乗らなくてよくなったんだけど
それでもみんな乗ってるよな。自転車が乗るとか車が乗るとか、人が乗るとか
0284名無しさん@英語勉強中 (ワッチョイ 9bc5-VtC7)
垢版 |
2020/08/10(月) 20:49:41.77ID:viV3yz5Z0
尾道は林芙美子が住んでたところなのだ。
それから、尾道三部作という大林宣彦の作品群の舞台となったのだ。
時をかける少女の映画はいい映画だよね。尾道は山の途中にも町がある。
0285名無しさん@英語勉強中 (アウアウエーT Sa23-Ea0s)
垢版 |
2020/08/10(月) 21:27:59.73ID:4SEea/JAa
first of all we've shut off the pathways for Iran getting nuclear weapon which was priority number one.

これはオバマ元大統領のインタビュー動画のスクリプトです。
聴いていて意味が分からず、スクリプトを見ると、文の構造がよく分かりませんでした。
日本語翻訳がある動画なので意味は理解はできたのですが。

この文の構造は
first of all we've shut off the pathways for Iran getting nuclear weapon
which was priority number one.
の二つから構成されているようです。
(1)whichは関係代名詞でしょうか。
(2)whichの意味は「それが」「以上が」「前述した内容が」ということですが、
このようなwhichの使い方はあるものですか。
0287名無しさん@英語勉強中 (ワッチョイW d19d-/Jt2)
垢版 |
2020/08/10(月) 22:00:24.17ID:IL9iinLi0
The time it takes to shopping is different between going shopping and shopping online.
買い物に行くのとオンラインでの買い物では、買い物にかかる時間が異なる。

これって合ってますか?
0288名無しさん@英語勉強中 (ワッチョイ 9bc5-VtC7)
垢版 |
2020/08/10(月) 22:31:02.38ID:viV3yz5Z0
>>287
あってるけど、上から下に訳し下ろすんだよね。
なぜかというと、英語と日本語を全く違う風に
やってしまうと(順序でね)、理解が大変になるんよ。
聞いててもそうだし。英文通りに理解するのが文章を理解するための
コツと言えますね。

で、僕の直訳は、
ショッピングにかかる時間はショッピングに出かけていくのと、
オンラインでのショッピングで異なる。
0294名無しさん@英語勉強中 (ワッチョイW 13f0-7U9h)
垢版 |
2020/08/10(月) 23:34:29.72ID:7GYqJmit0
>>279
>>282
ありがとうございました!
とても勉強になりました
0297名無しさん@英語勉強中 (ワッチョイ eb89-HICl)
垢版 |
2020/08/11(火) 03:28:08.39ID:X/Ea4Ybd0
>>285
別のサイトでは、次のように書いてあったよ。冠詞の a とかコンマなどもちゃんと
ついている。

Well, keep in mind, first of all, we've shut off the pathways for Iran getting
a nuclear weapon, which was priority number one.
https://www.bbc.com/news/world-us-canada-33646542
0298名無しさん@英語勉強中 (ワッチョイ eb89-HICl)
垢版 |
2020/08/11(火) 03:50:34.51ID:X/Ea4Ybd0
>>287
>> The time it takes to shopping is different between going shopping
and shopping online.

これって、自分で書いた英文を添削してくれっていう依頼かな?それなら、

(1) 「買い物のために」という意味なら、"to shopping" はここではありえない。
"for shopping" ならありえなくもない。

(1-a) the time for shopping
(1-b) the time it takes to shop (to shop は「買い物するために」という意味の
不定詞)
(1-c) the time (that) shopping takes
(1-d) the time required for shopping
(1-e) the time it takes to do the shopping
(1-f) the time it takes to go to the shops and back

簡潔に書くにはどうしたらいいかな?

(2-a) Online shopping is usually quicker than offline.
(2-b) It saves you time by shopping online rather than offline.
(2-c) The time you take in shopping varies between online and offline.
(2-d) Time for shopping varies between online and offline.
(2-e) Going to the shops and back (usually) takes more time
than doing the shopping online.
0299名無しさん@英語勉強中 (ワッチョイ eb89-HICl)
垢版 |
2020/08/11(火) 03:55:51.61ID:X/Ea4Ybd0
>>287
ネット上で見つけた英文。ただしここでは、オンラインの方が時間が節約できる
と想定している。「かかる時間が異なる」という趣旨をそのまま訳すと、どうしても
言い回しが堅くなる。

(1) Online shopping is quicker than shopping by visiting stores.
(2) Online shopping is quicker than shopping in-store.
(3) Online shopping is quicker than traditional shopping.
(4) Online shopping is quicker than face to face shopping.
0300名無しさん@英語勉強中 (ワッチョイ eb89-HICl)
垢版 |
2020/08/11(火) 04:04:18.71ID:X/Ea4Ybd0
>>287
"the time it takes varies between" という言い回しは、ネットでみればわかるように
よく使われている。これを使ってみると、

The time it takes varies between online and traditional shopping.

「オンラインの方が早い」とは言わないであくまで「かかる時間が違う」と言いたければ、
この上の言い回しが一番いいかなと俺は思う。
0302名無しさん@英語勉強中 (ワッチョイ 6b6c-B5sl)
垢版 |
2020/08/11(火) 18:18:26.31ID:YEakeUAN0
ネットとかで調べてだいたい分かった気がするけど
シェイクスピアのソネット1むずかしい
特に以下の語句の解釈にあまり自信がない
なんか良い注解書ないかな 岩波のやつ評価いいけどどうなんだろ

But as the riper should by time decease のbutの用法
to thy sweet self too cruel の文法的機能(thouの叙述っぽい)
To eat the world's due toの用法

From fairest creatures we desire increase,
That thereby beauty’s rose might never die,
But as the riper should by time decease
His tender heir might bear his memory:
But thou, contracted to thine own bright eyes,
Feed’st thy light’s flame with self-substantial fuel,
Making a famine where abundance lies,
Thyself thy foe, to thy sweet self too cruel.
Thou that are now the world’s fresh ornament,
And only herald to the gaudy spring,
Within thine own bud buriest thy content,
And, tender churl, mak’st waste in niggarding.
 Pity the world, or else this glutton be,
 To eat the world's due, by the grave and thee.
0304名無しさん@英語勉強中 (ワッチョイ eb89-HICl)
垢版 |
2020/08/11(火) 19:19:24.01ID:X/Ea4Ybd0
Shakespeare の注釈書なら、日本語によるものより英語によるものの方が絶対に
よいと思う。種類が豊富だから、何種類もの中から選べるしね。さらには、
何年か経ったら改訂版が出る。

たとえば、Arden Shakespeare というシリーズの Shakespeare 注釈書は、
Shakespeare の脚本または詩集1作品ごとに一冊の注釈書を刊行しているから、
全部で少なくとも40冊くらいある。それがこの50年ほどのあいだに3回も
刊行された。つまり20年くらいに1度、すべてを改訂するのだ。

改訂と言っても一部を変更するだけじゃなくて、すべてのページを全面的に書き換える。
まったく違った著者が、1ページ目からすべてまったく新しく書くのだ。

それで、First Series から Second Series へ、そして今は Third Series と
なっているけど、最初は1作品につき200ページくらいの薄っぺらい注釈書
だったけど、今では1作品ごとに400ページから600ページくらいあって、
値段も 2,000 円くらいで手ごろだ。

Sonnets については、次のものがある。

Shakespeare's Sonnets (The Arden Shakespeare: Third Series) Paperback
– June 1, 2010
0305名無しさん@英語勉強中 (ワッチョイ 6b6c-B5sl)
垢版 |
2020/08/11(火) 19:41:11.94ID:YEakeUAN0
さっそく中身チラ見してきた
自分の英語力だと、もっと解説ほしい

This glutton
demonstrative: the following line defines the kind of glutton he will be

これでTo eat the world's due が前の行の具体的説明になってることはわかるんだけど
toの用法の特定に確信が持てない
0306名無しさん@英語勉強中 (ワッチョイ eb89-HICl)
垢版 |
2020/08/11(火) 20:41:39.50ID:X/Ea4Ybd0
確かに、ネイティブスピーカー用の本だから、英語ネイティブにとってすぐにわかり
そうなことはどんどん省いてるんだろうね。俺自身も、この Arden Shakespeare とか
Cambridge とか Oxford などいろんな注釈本を並べて勉強することもあるんだけど、
背中に手が届かなくてかゆい思いをすることが多い。

でも、日本語の注釈本の世界って、
俺が想像するには競争が少なすぎて、誰か1人か2人の有名な学者が薄っぺらい本を
出したら、そのあとはみんな恐縮してしまって、誰もそれを超えるような本を
出そうとはしなくて、古い解釈の本が何十年も経っていても改善されないままで
残るんじゃないかという気がしている。だから、こういう本の日本人による
注釈書を俺は読んだことがないのだ。

だから Shakespeare と言えばイギリスが
本場だから、やっぱりイギリス国内のいろんな大学や教授たちが競争しあって
いろんな本を出し続けているから、それらを何冊か買って、あれこれ
比べながら読んでるんだ。
0307名無しさん@英語勉強中 (ワッチョイ 6e6c-wb83)
垢版 |
2020/08/12(水) 04:35:42.92ID:5VK2EdyM0
英語話者向けの者は文法解釈より、内容解釈のものが多いんだろうか
ラテン語の学習者向けの本とかは文法解釈が充実してるらしいけど

日本だと訳本は出てるけど、注釈書はあんまり流行ってないなあ
リーディングが軽視されるご時勢も関係してるかもしれないが
まあ、細部まで完璧に解釈できなくても、ある程度の理解で読み進むしかないか
ソネットかギタンジャリとか読んだら、次の英文解釈のステージいけそうだし
0308名無しさん@英語勉強中 (ワッチョイ 2e89-QlkZ)
垢版 |
2020/08/12(水) 05:15:00.05ID:kRXOHlG00
>>305
>>This glutton
demonstrative: the following line defines the kind of glutton he will be

ということは、

>>302
>> Pity the world, or else this glutton be,
>> To eat the world's due, by the grave and thee.

この verse の意味合いは、次のようになると思うんだけど、違うかな?

Pity the world, or else be greedy enough
●to eat● what the world deserves by the grave and thee.

だから "to eat" という不定詞が副詞的用法であって、「これこれこういうものを
食べてしまうくらいに貪欲になれ」と言っているんだと思うんだが。
0309名無しさん@英語勉強中 (ワッチョイ 2e89-QlkZ)
垢版 |
2020/08/12(水) 05:26:13.83ID:kRXOHlG00
Shakespeare's Sonnets であれそれ以外の Shakespeare の作品であれ、あるいは
Christopher Marlowe でも何でもいいから、もしそういう文学作品を読み続けて
いるんなら、ネット上のどこかにサイトでも作って(あるいは5ちゃんねる上で
スレッドを作って)コメントを書き続けてくれたら、俺は必ず参加するよ。

ただし俺は、Shakespeare's Sonnets に関しては、まだ10時間くらいしか
勉強したことがないんだけどね。Romeo and Juliet なら、5年ほど前に2か月くらい毎日
勉強したことがある。"A Midsummer Night's Dream" もそのあと2か月ほど、
さらに "Much Ado About Nothing" の前半だけを1か月ほど勉強した。
すべて歳を取ってからまったくの独学だから大したことないけどね。

なんせ Shakespeare を独学するわけだから、すぐに挫折しそうなので、
誰かと一緒にネット上で勉強できればいいな、といつも思ってる。
YouTube 上で Shakespeare の Romeo and Juliet の英文解釈ビデオを
20時間分くらいは投稿したんだけど、ほとんど誰も見ようとはしない。
Skype を使って何人かで一緒に Shakespeare を読まないか、と
何年か前からいろんな人に呼び掛けたが、誰も乗ってこない。
0310名無しさん@英語勉強中 (ワッチョイ 6e6c-wb83)
垢版 |
2020/08/12(水) 05:36:23.46ID:5VK2EdyM0
全体の趣旨として、青年に結婚して子供をつくりなさいとすすめてる詩らしい
上にあげた現代語訳(意訳)だとこうなってる
Take pity on the world, or else be regarded as a selfish glutton,
By the laws of God and nature you must create a child, so that the grave
does not devour the memory of your loveliness.

なので、意味的には
さっさと結婚して子をつくりなさい、
でないと君の美しさが死によって引き継がれない てきな意味だとおもう

どっかのだれかが現代語訳したもの
https://www.youtube.com/watch?v=a7hjgM3LwrM
ヒンドゥー語の解釈 なにいってるかさっぱりわからん
https://www.youtube.com/watch?v=kIgZz3Dmei4
0311名無しさん@英語勉強中 (ワッチョイ 6e6c-wb83)
垢版 |
2020/08/12(水) 05:39:14.53ID:5VK2EdyM0
シェイクスピアならどっかに読書会とかありそうだけどなあ
あるいは大学の市民講座とか
0312名無しさん@英語勉強中 (ワッチョイ 2e89-QlkZ)
垢版 |
2020/08/12(水) 06:31:01.93ID:kRXOHlG00
>>310
現代語訳らしきものがあちこちで発表されているとは思うけど、

>>302
>> Pity the world, or else this glutton be,
>> ●To eat● the world's due, by the grave and thee.

肝心のこの "to eat" という「to 不定詞」をどのように解釈してそのような
現代語訳(意訳)になったのかが、さっぱりわからない。
0313名無しさん@英語勉強中 (ワッチョイ 2e89-QlkZ)
垢版 |
2020/08/12(水) 07:43:42.43ID:kRXOHlG00
>>302  >> Pity the world, or else this glutton be,
      >> To eat the world's due, by the grave and thee.

この部分についていろんなサイトや書籍をちらほらと読んでみたが、これをきちんと
文法などに沿って解釈してくれているものは少ない。その中で、ネット上にあった
次の解説だけは、少し納得できるような気がする。

the world's due (14): what you owe to the world, i.e. the perpetuation of your
beauty. The grave, which will already consume the young man's body,
will also eat any chance of his beauty living on, if the young man helps
the grave by himself being gluttonous (in his refusal to have children).
★Steevens conjectures that the line should read "'To eat the world's due,
be thy grave and thee;' i.e. be at once thyself, and thy grave" (Alden, p. 19)★

この最後に書いてあるのは、Steevens という研究者の解釈によると
"To eat the world's due, be thy grave and thee" つまり
"To eat the word's due, to be at once thyself, and to be thy grave" という
意味だというわけだ。

この部分とその前の行とを合わせると

Pity the world, or else be glutton enough
to eat the world's due, and to be both yourself and your grave.

つまり
「世の中を憐れんでくれ。もしそうでないなら、
世の中が当然のことながら受けてしかるべきものを自分で食べてしまって、
自分らしく生きると同時に自分の墓になってしまえ」

という意味になるのではないかな?文字通りの意味はそういうことであって、
そこから思いっきり意訳すると、いろんなサイトや書籍に出ているような
解釈が生まれるというわけだと、今の俺は感じている。
0314名無しさん@英語勉強中 (ワッチョイ 2e89-QlkZ)
垢版 |
2020/08/12(水) 12:55:34.69ID:kRXOHlG00
>>313
by the grave and thee
と原文に書いてあるはずなのに、なぜ
be thy grave and thee
というふうに Steevens という人が解釈しているかというと、たぶん
by と be とが、400年前の Shakespeare の時代には(少なくとも軽く発音したときには)
「ビ」のように発音され、さらには thy が「thi」つまり舌を噛んだ上で「イ」と
発音していたからであるらしい。

(以上のことは、俺が手元に持っている
David Crystal, "The Oxford Dictionary of Original Shakespearean Pronunciation"
による。)ただし、the はどのように発音していたかについては、この辞書には
俺にわかるような書き方をしていないので、よくわからない。

ともかく、be と by だけは同じような発音をしていたらしいからこそ、
混同されて表記されてもまったく不思議ではないということだと思う。
0315名無しさん@英語勉強中 (ワッチョイ 712f-s47K)
垢版 |
2020/08/12(水) 14:26:38.37ID:q1W0+l2Z0
some people are concerned that [look] at the bright screens may affect sleep patterns.
のlookを正せという問題なんですけど
答えがlookingなのは主語だから動名詞はわかります
しかしなぜto lookじゃダメなんでしょうか?
0317三年英太郎 ◆3CZBjOt3.Y (ワッチョイ 4286-++7W)
垢版 |
2020/08/12(水) 20:26:43.88ID:tZd7Wk340
手許のソネットの翻訳

> この世を憐れみなさい。でなければ大食の罪に落ちます。
> 世間の分け前をきみと墓とでくらいつくすのだから。

to は本来、着点を指すのだから to eat .... は,
「結果の to」とか言われてるものと近いだろう。

翻訳を見れば、これは「七つの大罪」の大食をいましめて、
世の取り分を誰にも分け与えず墓場まで持ってくようなことはするな!
と言っているのがわかるわけである。

「日本の学者は〜」とか(しかも想像で!)ディスって
「貪欲であれ!」とか「自分らしく!」(ロハス女かよ)とか
トンチンカンな解釈をするより、
素直に巨人の肩の上にのって世の中を見渡したほうがよい例(苦笑)
0318三年英太郎 ◆3CZBjOt3.Y (ワッチョイ 4286-++7W)
垢版 |
2020/08/12(水) 22:07:47.90ID:tZd7Wk340
> 墓場まで持ってくようなことはするな!

アーデン版3では、ちと違うな

14 awkwardly expressed: what is owned to the world, the young
man's wealth of beauty, is instead consumed by himself, first,
and will be finally devoured by the grave. Dying is commonly
referred to as "paying one's debt to nature". For the sterile
body as itself a grave.

いずれにしても、発想がいかにもルネサンス人ですな
0320名無しさん@英語勉強中 (ワッチョイ 2e89-QlkZ)
垢版 |
2020/08/13(木) 06:08:01.85ID:MnBUoSwB0
英文の絵本さえまともに読んだことがなくて、文法書と辞書だけで英文が読めると
勘違いしていて、ひたすら日本語での解説だけにかじりついて、それを誤読しながら
鵜呑みにする馬鹿を相手にするのはやめよう。
0321名無しさん@英語勉強中 (ワッチョイ 2e89-QlkZ)
垢版 |
2020/08/13(木) 06:27:18.10ID:MnBUoSwB0
10年以上にわたる偽装がバレたからと言って、必死だな、この糞コテ。
コテハンを名乗っていなかったら、「なんだお前、そこにいたのかよ?
いるんならいるとはっきりしろよ。ほんと特徴のまったくない奴なんだから」
と言われ続けるのが落ちなんだからな。

平凡きわまりない扁平な鼻と目をしていながら、
長年にわたってイケメンだと大嘘をほざき続けているしな。
他人のことを "armchair なんとか" と罵倒し続けながら、自分が
その "armchair なんとか" だしな。

さらには他人を「内弁慶」と言いながら、自分が本物の内弁慶。
ギリシャ語もラテン語もフランス語もドイツ語も勉強します、と言っていながら、
最初の5ページもやらずに3日坊主が続く。

um ... zu ... については俺の勘違いでも何でもなくて、ただ俺の用例の挙げ方が
まずかっただけでその意味を知らなかったわけでも何でもなかったのに、
俺の真意を完全に誤読して、「基本的なことが何もわかっていない」と罵倒し続ける。
600時間もドイツ語を勉強した俺が、um... zu... に関してそんな馬鹿なことを
言うはずがなかろうとは考えもしない。所詮は、ほんの20時間くらい
ドイツ語をかじっただけでわかったような気になってしまっているという、
いつものやり口だ。

あほらしくてずっと黙っていたら、それをいいことにまったく頓珍漢な
negative campaign を続ける。俺が日本であり、この糞コテハンは
朝鮮とか中国共産党みたいなやり口を続ける。

ところで、肉便器の相手を探し求めている西洋人から自分に都合のいい
コメントをもらって披露するという例の卑劣な手口は、もうやめたのかい?
0322三年英太郎 ◆3CZBjOt3.Y (ワッチョイ 4286-++7W)
垢版 |
2020/08/13(木) 09:37:25.88ID:Yh9plglq0
アーデンアーデン言いながら、当の本人が参照しておらず
どこの誰が書いたとも知れぬネットの「書き換え」をうのみにして
挙句の果てに「自分らしく!」て(笑) 

それは、ルネサンス的発想じゃありません(笑)

若者はバカだ〜、アメリカ人はバカだ〜、女は肉便器だ〜
そう世を呪って、いいことありましたかね?
0323三年英太郎 ◆3CZBjOt3.Y (ワッチョイ 4286-++7W)
垢版 |
2020/08/13(木) 09:40:39.89ID:Yh9plglq0
毎回ふしぎに思うんだけど、「〇〇語を××時間やった!」って、
それ何の自慢になんの?
シェイクスピアを〇ヶ月やりました!って何アピール?

その結果、できるようになって、初めて自慢できると思うんだけど?

ピアノを〇〇年やりました!←だからなんだよ
ショパンの練習曲弾けます!←すごいっ!

こーゆー自慢する人、このおじいちゃん以外に
見たことないんだけど、いったいなんなん?
ドイツ語何年やってようが(本人談)、
結果として初歩の知識さえ知らなかったじゃん
0324三年英太郎 ◆3CZBjOt3.Y (ワッチョイ 4286-++7W)
垢版 |
2020/08/13(木) 09:50:00.17ID:Yh9plglq0
もひとつ、某OEDさんにアドバイスしますね(笑)

https://pbs.twimg.com/profile_banners/537227071/1587631348/600x200
世の中の誰が、こんな小汚いじいさんが暗がりでモゴモゴ
しゃべってる動画を見たいと思う?

「自分がこ〜んなに〇〇してやってるに、バカどもは見ない」
じゃなくてさ〜、世にアピールしたいなら、も少し視聴する側の
気持ちを研究した方がいいと思いますね(笑)
0325名無しさん@英語勉強中 (ワッチョイ 2e89-QlkZ)
垢版 |
2020/08/13(木) 10:11:44.58ID:MnBUoSwB0
xyz
0326名無しさん@英語勉強中 (ワッチョイ 2e89-QlkZ)
垢版 |
2020/08/13(木) 10:11:58.92ID:MnBUoSwB0
wzx
0327名無しさん@英語勉強中 (ワッチョイ 2e89-QlkZ)
垢版 |
2020/08/13(木) 10:12:18.55ID:MnBUoSwB0
uuu
0328三年英太郎 ◆3CZBjOt3.Y (ワッチョイ 4286-++7W)
垢版 |
2020/08/13(木) 11:07:33.57ID:Yh9plglq0
Steevens conjectures that the line should read
"'To eat the world's due, be thy grave and thee;'
i.e. be at once thyself, and thy grave" (Alden, p. 19)

Alden: Sonnets, from the quarto of 1609, with variorum readings and commentary.
該当ページ
https://archive.org/details/sonnetsfromquart00shakuoft/page/18/mode/2up

この Steevens は、Twenty of the Plays of Shakespeare (1766) の
発行者のようで、スキャンしたものがアマゾンでも売ってる。

> To eat the world's due, by the grave and thee
> To eat the world's due, be thy grave and thee

e ⇔ y の誤植じゃね?程度の思い付きだろう。

とまれ、素直に翻訳を参照してれば、こんなトンチキ解釈に
まんまと引っかからずにすんだのではあるまいか(笑)
0329三年英太郎 ◆3CZBjOt3.Y (ワッチョイ 4286-++7W)
垢版 |
2020/08/13(木) 11:19:13.15ID:Yh9plglq0
> でも、日本語の注釈本の世界って、
> 俺が想像するには競争が少なすぎて、誰か1人か2人の有名な学者が薄っぺらい本を
> 出したら、そのあとはみんな恐縮してしまって、誰もそれを超えるような本を
> 出そうとはしなくて、古い解釈の本が何十年も経っていても改善されないままで
> 残るんじゃないかという気がしている。(>>306)

そうして、英語資料こそ正しいんだと妄信して
250年前のたわいない思い付きを信じちゃうと(笑)
0330名無しさん@英語勉強中 (ワッチョイ 2e89-QlkZ)
垢版 |
2020/08/13(木) 11:26:55.11ID:MnBUoSwB0
xcv
0331名無しさん@英語勉強中 (ワッチョイ 2e89-QlkZ)
垢版 |
2020/08/13(木) 11:27:08.36ID:MnBUoSwB0
abc
0332名無しさん@英語勉強中 (ワッチョイ 2e89-QlkZ)
垢版 |
2020/08/13(木) 11:27:27.75ID:MnBUoSwB0
cbs
0333名無しさん@英語勉強中 (アウアウウー Saa5-/zfN)
垢版 |
2020/08/13(木) 12:04:47.28ID:IbYr19nBa
このwouldの用法はなんですか?

Five acres were reserved for the building, which would not be constructed for many years.
0338名無しさん@英語勉強中 (ワッチョイ 41e2-FRDO)
垢版 |
2020/08/14(金) 11:05:44.39ID:6lxUbnmZ0
>>333
単に時制の一致のwillの過去形。
0339三年英太郎 ◆3CZBjOt3.Y (ワッチョイ 4286-++7W)
垢版 |
2020/08/14(金) 11:16:03.01ID:67vNU/Qi0
1600年頃     1750年頃
by [bəi]/[bɪ]    [bʌi]
be [be(ː)]※    [bi(ː)]     ※上寄り

the [ðə]      [ðə]
thy [ðəi]/[ðɪ]    [ðʌi]

出来た! 
二つ並んでる場合は、後ろが強勢なし。

もちろん、これは一つのモデルであって
これとまったく同じように発音されていた
というわけではない。

「ここだけ活字のタイポなんじゃね?」なら分かるが、
「発音が似ていたから、本来はこう読むべき!」というのは激しく嘘くさい。
それなら、その版のすべての the ⇔ thy を交換できるのか?

アーデン3他を見る限り、そんなとぼけた主張は却下されてるようである(^^)
0340名無しさん@英語勉強中 (ワッチョイ 2e89-QlkZ)
垢版 |
2020/08/14(金) 20:26:44.79ID:vW2jZ3GT0
>>335 の「過去から見た未来」に一票。
0341名無しさん@英語勉強中 (ワッチョイ 2e89-QlkZ)
垢版 |
2020/08/14(金) 20:27:13.71ID:vW2jZ3GT0
acb
0342名無しさん@英語勉強中 (ワッチョイ 82a7-jUmA)
垢版 |
2020/08/14(金) 20:41:40.10ID:r8llUKsG0
>>335
He said that it would not be rain tomorrow?
なんていうのかな?
過去から見た未来なんてものあるのかな?
話した時点が基準なんではないの?

He said that it was not rain the next day.
としたら間違い?
0343名無しさん@英語勉強中 (ワッチョイ ed60-XQXO)
垢版 |
2020/08/14(金) 20:51:01.95ID:bjFkEVeG0
というかbe rainという表現が変と感じないのか?
君は雨が降っていることをit is rainというのか?
0345名無しさん@英語勉強中 (ワッチョイ 82f0-7DKX)
垢版 |
2020/08/15(土) 08:26:10.02ID:4gp4ywxD0
次の文の関係代名詞what の解説をいくつかお願いします。

Science can be defined as the reduction
of multiplicity to unity.
It seeks to explain the endlessly diverse
phenomena of nature by ignoring the
uniqueness of particular events,
concentrating on what they have in
common and finally abstracting some
kind of“law”, in terms of which they make
sense and can be effectively dealt with.
科学は多様性を単一性へ還元することであると定義づけできる。
科学が説明しようとしているのは、自然界の果てしなく多様な現象であるが、
それはある特定の事象の持つ特殊性を無視し、個々の現象の共通点だけに注意を向け、
そして最後に抽象化によってある種の「法則」を導き出すことによってなされ、
またそうした法則に照らして諸事情は意味を持ち、効果的に扱うことができるのである。
in terms of which 以降がよく分からないです。
関係代名詞の後ろの文は、主語か目的語が欠けていると聞いたことがあり、
which は関係代名詞だと思うのですが、後ろに欠けている部分はあるのでしょうか?
they make sense は、主語も目的語も欠けていないし、and can be effectively dealt with.は、主語がないけど、前のtheyとつながっているので主語は欠けているとは言わない気がします。
知恵袋でも質問したのですがよく分かりませんでした。
0348名無しさん@英語勉強中 (ワッチョイ bee3-s47K)
垢版 |
2020/08/15(土) 09:40:59.76ID:YgaKIOTS0
前置詞+関係代名詞
もっと簡単な文で考えてみると分かりやすいかも

This is the park in which I met her.
which以降は、「主語も目的語も欠けていない」ですよね。

二つの文に分けてみると
1) This is the park..
2) I met her in it.

whichは(2)のitと同様に、前置詞inの目的語となっていて、in it (in which)で副詞句を形成しています。

元の文を二つの文に分けてみると、
1) It seeks 〜 abstracting some kind of "law".
2) They make sense and can be effectively dealt with in terms of it(=the law).

一見複雑ですが、in terms of it (which)で副詞句を形成している点は前例と同じです。
0349名無しさん@英語勉強中 (ワッチョイ bee3-s47K)
垢版 |
2020/08/15(土) 09:45:08.34ID:YgaKIOTS0
関係代名詞は、関係詞の節の中で代名詞と同じ働きをします。
その働きが主語や目的語である場合は、当然関係代名詞の後に主語や目的語が欠如することになります。
前置詞+関係代名詞のように、前置詞の目的語になる場合は、関係詞の後に主語や目的語が欠けることはありません。

つまり、
「関係代名詞の後ろの文は、主語か目的語が欠けている」が常に正しいとは限らないということです。
0351名無しさん@英語勉強中 (ワッチョイ 2e89-++7W)
垢版 |
2020/08/15(土) 17:46:12.10ID:Fzvd5hLe0
>>333
文法についての議論というものは、たいして英語がわかっているわけでもなく、ろくに英文を読んできた
経験もないのに、へ理屈だけこね回せば英語がわかっているかのようなふりができるので、
せっかくの正解も多数派によって間違いとされてしまうことも多い。

今回の would が「過去から見た未来」を示すということを間接的に証拠づける例が、俺の今たまたま
読んでいる本の中で少なくとも10か所くらい立て続けに出てきた。


Kurt Vonnegut というドイツ系のアメリカ人作家の年表より

(1) 1941: Disappointed in Butler (University), transfers to Cornell University....
(上の transfers は、"he (= Kurt Vonnegut) transfers" という意味。)
Following his father's orders ..., shuns "frivolous" classes in the arts and humanities,
focusing instead on chemistry and physics.
("I had no talent for science." Vonnegut ●will● remember. "I did badly.")

これは著者の略歴を綴っているので、すべて現在形を基本としている。通常ならば
1941 に起こったことだから過去形で書きたくなるが、ここでは1941年のことを
現在形で述べている。そしてそのあと ( ) の中で、「著者は後になってから
これこれこういうことを思い出すことになる」というわけで未来形を登場させている。

この例では、現在形プラス未来形となっているが、今回の質問者の用例では
「過去形プラス過去未来」となっているのだ。

さて、上のような例文が少なくとも10個ほど一冊の本の中で見つかった。上の用例だけでは
物足りなければ、他にいくらでも用例を見せてあげよう。

というわけで、たかが文法ごときでも、文法書と辞書だけを頼りにして質問者たちから提示された
例文の文法的意味合いを解説しきることができるわけではないのだ。普段からの読書体験によって
蓄積されてきた英文データベースがどれくらいに豊かであるかによって、一つひとつの文法事項が
立体的に理解できる。そのデータベースが貧弱な人は、へ理屈によってごり押しするしかなくなるということになる。
0352名無しさん@英語勉強中 (ワッチョイ 2e89-++7W)
垢版 |
2020/08/15(土) 17:51:35.73ID:Fzvd5hLe0
>>333
ここで俺が使っている参考文献は、次の通り。

Kurt Vonnegut: Novels & Stories 1963-1973
The Library of America, p. 813 以降の
Chronology (略歴) のページより

今回のこの著者略歴のみならず、一般的に
文学者などの略歴を英文でたくさん読んでいると、
今回のような「過去の時点から見た未来」が
たくさん出てくることが多いので、今回のことが
立体的に理解できるようになる。
0353名無しさん@英語勉強中 (ワッチョイ 2e89-++7W)
垢版 |
2020/08/15(土) 18:18:10.02ID:Fzvd5hLe0
>>333
いま俺が読んでいる本の著者略歴の中で、さっきよりもはっきりと今回のことを
よく理解させてくれる用例を見つけた。

1966: ... Vonnegut ... hires entertainment lawyer ....
(In 1977 Farber ●will become● Vonnegut's sole agent and attorney.)

つまり上の例では、1977年という、2020 年の今から見れば過去のことを
言うときに未来形を使っている。これは2020年の今を基準にしているのではなくて、
あくまで 1966 年という過去の時点を基準にしているので、1966年のことを
述べるときにはあくまで現在形であり、それ以降のことをここで述べるときには
未来形を使っているわけだ。
0356307 (ラクッペペ MM26-YT0q)
垢版 |
2020/08/15(土) 22:55:56.16ID:goZ8ExWjM
最初に出てくるwillは、どんな役割のものですか?

ロングマン英英で調べましたが、いまいちどれが当てはまるのか、しっくりきてません。

BELIEF�used to say that you think something is true

↑コレ?

https://i.imgur.com/vn8CRoC.jpg
0357名無しさん@英語勉強中 (ワッチョイW 4128-BjFG)
垢版 |
2020/08/16(日) 14:18:15.61ID:J0oH+ZTS0
>>351
ほうでは、その「過去から見た未来」を使って、どう意味解釈するのですかね? 訳してみてくださいよ
0358名無しさん@英語勉強中 (ワッチョイ 2e7b-p5K4)
垢版 |
2020/08/16(日) 14:21:13.59ID:4xYD3IQR0
●● の言ってることはあてにならない
0360三年英太郎 ◆3CZBjOt3.Y (ワッチョイ 4286-++7W)
垢版 |
2020/08/16(日) 14:52:09.52ID:j0G39Sud0
351-353は、354が指摘してる通り歴史的現在、
もしくは年代記の現在とゆーもので、
発話時と基準時が一緒とゆーか、
おそらくより正確には基準時という概念が不在のために、
デフォルトの現在形で埋められているという例

> Chronicles of history
> Past events can also be expressed in this
> *timeless present tense* when they are seen as
> part of a chronicle forming a permanent record of
> history.
CaGEL p.130

どーゆー意図で>>333を説明するのにこれを持ち出したのか
「普段からの読書体験によって蓄積されてきた英文データベース」
が調子がくるってるのか、ちょっとわかりませんね(汗)
0361三年英太郎 ◆3CZBjOt3.Y (ワッチョイ 4286-++7W)
垢版 |
2020/08/16(日) 15:10:07.16ID:j0G39Sud0
間接的になにを証明してるのか、よーわからん

未来を表現する will が、
「時制の一致」で would になるとゆーことをなら、
>>33と同じく、関係節内が will → would になる例を挙げる。

> This was the place where I would work.
> (これが私の勤めることになる店だった)
リーダーズ2(一部省略)
0363名無しさん@英語勉強中 (ワッチョイ 2e89-++7W)
垢版 |
2020/08/16(日) 16:14:22.13ID:7hWegtav0
夏休みとか冬休みなどの休暇が来るたびに、1か月か2か月ほどしつこく俺に絡みついてくる超初心者たちが
必ず現れる。すまんけど、そいつらの相手はしない。ともかく、必要最小限度の英文読書(どんなに少なくても
3万ページくらい)をクリアしたことのない人間は、俺に絡みつかないでくれ。必要最小限度の読書が
クリアできていない人間は、そこらへんの馬鹿ネイティブとメール交換でもして喜んでいればいい。

3万ページと言ったけど、それはあくまで「どんなに少なくとも」という意味であって、それで十分なわけがない。

「過去から見た未来」と解釈するならどのように訳せるか、なんていう質問もやめてくれ。
最小限度の日本語の読書をしている奴なら、簡単に訳せる。英文読書はおろか、
日本語読書さえしたことのない人間に、俺は空しい指導をするのはやめた。

雑談スレを本拠地とする糞コテハンが7年前から俺に絡み続けているが、この2年前からは常に
あぼーんしている。たまに偶然に糞コテハンの言っていることが見えてしまうこともある。そういうときに
うっかりと俺は返答してしまって、大変に時間と労力を浪費し、後悔することになる。
0365名無しさん@英語勉強中 (ワッチョイ 82a7-jUmA)
垢版 |
2020/08/16(日) 16:22:42.17ID:KNkUpVUI0
>>338
時制の一致という考えは知らない
と知り合いのあフロリダ在住だった
人はいった。今から見るとき過去だから
that 以下の動詞を過去にするだけだ
とも続いて説明してくれた
0367名無しさん@英語勉強中 (ワッチョイ 82a7-jUmA)
垢版 |
2020/08/16(日) 16:35:28.78ID:KNkUpVUI0
英語に未来系なんてないんじゃないの?
willって法の助動詞だから、不確かなことについて
不安な気分を表すものなのでは?

未来過去というんじゃなくて、
不確かなことについて不安な気持ちを持ってる、その時、は
今から見ると過去なんだけど、その過去についていう時に
例えばhe said that it would be built in the next year.
とかいうんじゃないの?
0369名無しさん@英語勉強中 (ワッチョイ 715a-Cejt)
垢版 |
2020/08/16(日) 17:04:59.90ID:coVr1E/z0
英語で「未来時制が無い」というのは、動詞の活用だけで
未来を専門に表現することができないからそう言われているだけで、
willなどが純粋に単純未来を表現できないわけじゃない。
willは意志など他の意味でも使われるから純粋な「未来形」認定
されないというだけ。
0370三年英太郎 ◆3CZBjOt3.Y (ワッチョイ 4286-++7W)
垢版 |
2020/08/16(日) 17:30:30.61ID:j0G39Sud0
でもさー、英語に未来時制があると考える人たちは
時制は形態論的特徴のみじゃないんだ!って考えるわけでしょ。
それなら、なおさら「未来形」っつーコトバは避けるじゃん?
「〜形」でとらえきれない時制なんだからさ。

ま、そんなこたー一般人にはどーでもいいんですけど、
某OEDさんって比較言語学を研究なさってるそうでしょ?
(現代英語以外出来ない人が、どうやって研究されてるのか謎だけど)

だったら、そこんとこ慎重にならなきゃいけなくない?
0371335 (ワッチョイ bee3-s47K)
垢版 |
2020/08/16(日) 17:42:34.04ID:pNDuoTx+0
>>357
横から口を出すようですが、「過去から見た未来」を持ち出したのは私なのでレスしておきます。
日本語訳は>335で既に示しました。

「過去の時点から見た未来」という表現はランダムハウス英語辞典から引用したものです。
*****
would
━auxil.v.will1 の過去形
【1】((従節における時制の一致))((過去の時点からみた未来(完了) ))…だろう:
He said he would go tomorrow.[=He said,“I will go tomorrow.”] 彼は明日行くと言った
*****

ジーニアス大英和では、このようなwouldを単に「時制の一致」として扱っているようです。
しかし、ジーニアス第5版では、wouldの項に特別な語法コラムがあって、以下のように記述されています。
*****
語法(1)[過去から見た未来] 物語では過去から見た未来を表すことがある:
The climbers had reached 3,000 meters. Soon they would see the summit.
登山者たちは3000メートルまで登っていた。もうすぐ頂上が見えてくるだろう。
*****
0372名無しさん@英語勉強中 (ワッチョイ bee3-s47K)
垢版 |
2020/08/16(日) 17:56:32.00ID:pNDuoTx+0
>>351-353
歴史的現在を基調に語られる略歴の中でwillが用いられる実例。
大変興味深かったです。ありがとうございました。

過去形を基調にした場合、次のようになるということですよね。

Following his father's orders ..., shuned "frivolous" classes in the arts and humanities,
focusing instead on chemistry and physics.
("I had no talent for science." Vonnegut ●would● remember. "I did badly.")

1966: ... Vonnegut ... hired entertainment lawyer ....
(In 1977 Farber ●would become● Vonnegut's sole agent and attorney.)
0374名無しさん@英語勉強中 (ワッチョイ bee3-s47K)
垢版 |
2020/08/16(日) 18:27:06.66ID:pNDuoTx+0
ついでながら、このwouldを「意志のwill」とする解釈(>>334)にも反論しておきます。

The PC would not work. パソコンが動こうとしなかった。
これは、PCが*あたかも意志を持っているかのように*workすることに反抗している、と読んで良いかと。

Five acres were reserved for the building, which would not be constructed for many years.
この文ではwhich(=the building)が*あたかも意志を持っているかのように*建設されることに反抗しているのでしょうか?

それは無理でしょう。
このbuildingはまだ存在さえしていないわけで、存在していないものが意志を持って何かに反抗できるはずがありません。
0375三年英太郎 ◆3CZBjOt3.Y (ワッチョイ 4286-++7W)
垢版 |
2020/08/16(日) 19:33:03.17ID:j0G39Sud0
「時制の一致」は、主節の時制が従属節の時制に影響を及ぼすこと。
>>333はこれで理解できるので、>>361, >>371のRH大の引用だけで十分。

一方

> The climbers had reached 3,000 meters. Soon they would see the summit.

は、文をこえて「一致」が起こってる特殊例で
書き言葉の*物語スタイル*で見られるという(『講義』 p.320)。

さて、簡潔さが売りの「年代記の現在」を使ってる年表で、
過去を基準時に設定しなおしたい場合、
一様に時制を一個もどして、will は would になるから
In 1977 Farber would become .... だ! はい、出来ました!

これでいいの? ちょっと奇異に聞こえるということはない?
手元の文法書みても、書いてなかった。
0376三年英太郎 ◆3CZBjOt3.Y (ワッチョイ 4286-++7W)
垢版 |
2020/08/16(日) 19:34:35.94ID:j0G39Sud0
簡潔に答えられる問題に対して(PEUにも出てる。§43)、
3レスも使って、しかもその延長には一般学習者が
なかなかアクセスしづらい問題にぶちあたる回答って
ピントずれてません?

的確な答えをはきだしてくれない英文データベース、
ポンコツじゃないですかね?
0377名無しさん@英語勉強中 (ワッチョイ 2e89-++7W)
垢版 |
2020/08/16(日) 20:49:31.29ID:7hWegtav0
>>372 >>374
僕が言いたかったことを雄弁に語ってくださって、ありがとうございます。
1,000 人のうちの 999 人は理解を拒絶していても、1人だけは
僕の示す英文用例を読んで何らかの参考にしてくれればそれでいいと思いましたが、
やはりあなただけはきちんと理解してくれていたのですね。うれしいです。

僕は最終的には間違っていたとしてもよかったのです。ただ、こういう
面白い用例があるから、今回の場合には仮に適合しなかったとしても、
また別の時に役立つ時が来ればそれでいいと思っていました。

あなたは見事に、僕と同じようにこの著者略歴を面白く読んでくれたのですね。
それだけでも僕は、(大げさに言えば)喜んで死んで行けます。

僕は参考書や文法書や辞書をうまく活用したり文法用語を駆使して解説するのが
下手で、仕方ないから洋書や映画やドラマからやたらに用例を引き出そうとしますが、
うまく整理できていないため、こういうふうにすぐに周囲から攻撃されます。
でも、あなたは見事にこの件を解説してくれました。
0378名無しさん@英語勉強中 (ワッチョイ 82a7-jUmA)
垢版 |
2020/08/17(月) 16:52:06.42ID:vvtHDRhf0
>>377
なんか>>377さんって都合がいいんだよね。
>あなただけはちゃんと理解してくれていた、
とかね。おまけに
>喜んで死んで行けます、
だと??いつ死ぬの?
文法弱いと思ったのなら、文法勉強しろよ
0379名無しさん@英語勉強中 (ワッチョイ 8160-XQXO)
垢版 |
2020/08/17(月) 17:05:36.07ID:IAGHtIYD0
Nor will I reveal either the name or the location of her Ashram,
thereby sparing that fine institution publicity which it may have neither the interest in nor the resources for managing.
意味わからない頼む。
interest in publicity
the resourses for pubuliciy
managing はこの両方に形容詞的にかかるんだろうか?
構造が全然わからない。
0380名無しさん@英語勉強中 (ワッチョイW 4128-BjFG)
垢版 |
2020/08/17(月) 17:07:06.96ID:1kPjBM310
>>374
>5エーカーの土地がその建物のために確保されたのだが、その後何年もの間その建物は建設されなかった。

↑例で出された「もうすぐ超頂上が見えてくるだろう」みたいな「過去の時点からみた未来」になってないよ。
過去の時系列を現在の視点から述べてるだけにしかみえないね。

>>374
存在してないのが問題なら、あたなの解釈もおかしくなるじゃないの? 建物は計画として存在してると考えるベキでしょう。ではければ reserved for the building も成り立たんと思う。
The building(計画)があたかも意思をもっているでもいいと思うけどね。
ただここは受動態だし文章にはでてこない施工主(またはオーナー)の意思と考えてもいいのかもしれん。
まあ、どっちにしても、あなたの反論には無理がアリあるかな。

>>374
お前が自演してるだけだろw English板では、お前みたいなテイストのヤツしか見かけん。 
0381名無しさん@英語勉強中 (ワッチョイ 2e89-++7W)
垢版 |
2020/08/17(月) 17:17:54.32ID:LmJjbsH20
>>379
問題文の前に出てくる文脈をすべて示す。問題文には ★ の印をつけておく。

But most especially I thank my Guru, who is compassion’s very heartbeat, and who so generously
permitted me to study at her Ashram while I was in India. This is also the moment where I would
like to clarify that I write about my experiences in India purely from a personal standpoint and
not as a theological scholar or as anybody’s official spokesperson. This is why I will not be using
my Guru’s name throughout this book ? because I cannot speak for her. Her teachings speak
best for themselves. ★Nor will I reveal either the name or the location of her Ashram, thereby
sparing that fine institution publicity which it may have neither the interest in nor the resources for managing.★
0382名無しさん@英語勉強中 (ワッチョイ 8160-XQXO)
垢版 |
2020/08/17(月) 17:19:06.85ID:IAGHtIYD0
なんや、喧嘩しとったんか? 一旦やめて、379に回答してからにせい。
0383名無しさん@英語勉強中 (ワッチョイ bee3-s47K)
垢版 |
2020/08/17(月) 18:05:50.84ID:9qXnNUEj0
>>381
Nor will I reveal either the name or the location of her Ashram,
ここは倒置が起こってるだけで、とくに問題ないでしょう。
「私は彼女のアシュラムの名称も場所も明らかにするつもりはない」

thereby (そうすることによって=明らかにしないことによって)

sparing that fine institution publicity
分詞構文。≒I will spare that fine institution publicity
ここのニュアンスが難しい。たぶん次のような意味かと。
「あの素晴らしい施設(=アシュラム)を宣伝で煩わせないようにする」

which it may have neither the interest in nor the resources for managing.
which = pubulicity、it=Ashram
(そんなことを言っても)当のアシュラムは、宣伝などには興味を持っていないかもしれないし、その宣伝を行う(managing)するための資金も持っていないかもしれないわけだが。

別の妥当な解釈があったら御教示ください。
0384名無しさん@英語勉強中 (ワッチョイ bee3-s47K)
垢版 |
2020/08/17(月) 18:14:07.28ID:9qXnNUEj0
which it may have neither the interest in nor the resources for managing.
ここの構造ですが、

It may not have the interest in <publicity>.
It may not have the resources for managing <publicity>.

を、neithre A nor Bのパターンで結びつけたものでしょう。
0385名無しさん@英語勉強中 (ワッチョイ 8160-XQXO)
垢版 |
2020/08/17(月) 18:25:07.74ID:IAGHtIYD0
>>383,384
ああ、そういうことか。
スッキリしました。 ありがとう。

翻訳本見るとかなり意訳してる(というか明らかに意訳しすぎというかハッキリ言うと間違ってる)ので、混乱してました。
0386名無しさん@英語勉強中 (ワッチョイ 2eb5-wvIG)
垢版 |
2020/08/17(月) 18:34:33.25ID:p6t70Bz30
>>383
>その宣伝を行う(managing)するための資金も持っていない
ここは、私が宣伝する(名前を言う)ことで、その注目に対処するだけのリソースがないという意味にとった方が繋がりがよいかと
0387名無しさん@英語勉強中 (ワッチョイ 8160-XQXO)
垢版 |
2020/08/17(月) 18:37:26.33ID:IAGHtIYD0
翻訳本の訳
 また、彼女のアシュラムの名や所在地を記すことで、あの素晴らしい施設がそこにふさわしい関心や動機を持たない人々の
 目にさらされるのを避けたいと考えた。

日本語としては綺麗だが間違ってるよね。(いや間違いなのだと断定する自信はないが。w でも間違ってると思う。)
0388名無しさん@英語勉強中 (ワッチョイ bee3-s47K)
垢版 |
2020/08/17(月) 18:50:25.08ID:9qXnNUEj0
>>386
そうですね。ご指摘ありがとうございます。

>>387
誤訳でしょうね。
たぶんpublictyをpublicと読んで、whichの先行詞をpublicとし、which直後のitを無視しているようです。

よろしければ書名を教えていただけませんか?
0389名無しさん@英語勉強中 (ワッチョイ 8160-XQXO)
垢版 |
2020/08/17(月) 19:20:02.20ID:IAGHtIYD0
>>388
EAT, PRAY, LOVE
です。プロだから大急ぎで翻訳してるからそういう間違いはあるでしょうね。
The Hunger Gamesなんかたしかドサッと1ページくらい落ちてるところがありました。 
読める人は英文をそのまま読むし、読めない人は翻訳だけを読む。翻訳と突き合わせて
読んでるのは日本で俺一人くらいだろうからだれも気が付かない。
0390名無しさん@英語勉強中 (ILW 0H52-4de8)
垢版 |
2020/08/17(月) 19:25:44.52ID:Dwm8LTT8H
>>379
上の人の訳の方が正しいような気もするんだけど、
最初に読んだ時のオレの解釈とはちょっと違ったので、
一応ポストしてみる

アシュラムの名前や場所を公開するつもりもない。
彼女にとって興味がない、あるいは、それを捌き切るだけのリソースもないのに、
メディアの注目を集めてしまうことを防ぎたいからだ。
0392名無しさん@英語勉強中 (ワッチョイ c2f0-++7W)
垢版 |
2020/08/17(月) 19:54:33.18ID:JLUqonZv0
neither the interest in nor the resources for
managing publicity

だろ
0395名無しさん@英語勉強中 (ワッチョイ c2b8-QlkZ)
垢版 |
2020/08/18(火) 00:36:18.71ID:wQID9ECJ0
>>379
Nor will I reveal either the name or the location of her Ashram,
また私が彼女のアシュラムの名前や場所を明らかにすることはないでしょう。
thereby sparing that fine institution publicity which it may have neither the interest in nor the resources for managing.
それによって、公開に興味がないかもしれない、また公開に割けるリソースがないかもしれないあの良き施設の公開を避けることができるのです。
whichはpublicity 公開
itはthat fine institution あの良き施設
0396名無しさん@英語勉強中 (ワッチョイ c2b8-QlkZ)
垢版 |
2020/08/18(火) 01:17:00.76ID:wQID9ECJ0
spare publicityは非公開という意味でよく使われるらしい
0397名無しさん@英語勉強中 (ワッチョイW 42e3-If6c)
垢版 |
2020/08/18(火) 13:19:48.42ID:Wb8wMJRY0
When deciding punishments for crimes, criminals’ intentions are important.
(犯罪に対して刑罰を決める際には、犯罪者の意図が重要だ。)

上の文のwhen節についてなのですが、従節における慣用的な主語+be動詞の省略ルールを適用して
whenとdecidingの間にyou[one] are[is](you[one]は総称用法)が省略されているという解釈で大丈夫でしょうか

それとも総称的な意味上の主語が省略された独立分詞構文にwhenがくっついた形?とかもあり得るんでしょうか

文自体が慣用表現っぽくなく(元は英作文参考書の作例です)、主節の主語を従節に当てはめても意味不明な文になるのでモヤモヤしています
0398名無しさん@英語勉強中 (ワッチョイ bee3-s47K)
垢版 |
2020/08/18(火) 17:04:05.27ID:Pp9Tm14m0
>>397
When you are deciding punishments for crimes, とは普通言わず、
When you decide punishments for crimesと言うでしょうから、you areの省略とは考えにくいのでは?

おっしゃる通り、独立分詞構文にwhenが付いた形と見るのが適当かと思います。
文法的に許容範囲にあるかどうかは私には判断できませんが、whenがあるおかげで一読して分かることは分かりますね。
0399名無しさん@英語勉強中 (ワッチョイW 42e3-If6c)
垢版 |
2020/08/18(火) 18:16:13.76ID:Wb8wMJRY0
>>398
回答ありがとうございます。

確かに考えてみるとwhen you are deciding...は不自然な表現ですね。

分詞構文にwhenがついた形を調べてみたところ、あいまいな分詞構文の文意を明確にするために接続詞を併せて使うこともあるそうでした(それなら最初から分詞構文を使わず書けよと思いますが・・・)。
この用法で用いられる接続詞は
while, when, if, thoughなどに限られるようでした。(inspire 総合英語)

ただ、参考書の文例が全て〈接続詞+分詞構文〉の形で、〈接続詞+独立分詞構文〉の文例は一例も見当たらなかったので、これが規範から外れない表現なのか、文法書の裏付けがないのは気になります。
割と有名な参考書の例文ですし、この例の直後にも似たような作例
This must be taken into account when considering punishments for minors.
(This = young people’s brain are not as quick at making good decisions as adults.)
があったので間違いではないとは思うのですが。
0401名無しさん@英語勉強中 (アウアウウー Sa63-kQzz)
垢版 |
2020/08/20(木) 02:38:35.98ID:hfmTc44sa
ibathe in the bloth ってどう訳せばいいですか?
0402名無しさん@英語勉強中 (ワッチョイ 7ff0-DIxo)
垢版 |
2020/08/20(木) 08:33:14.40ID:7gwH/mqU0
次のit is as if~って文法としてどうなってるんですか?
as if ~の形は見たことあるのですが、it is の後ろにあるのがよく分かりません。
なぜit is の後ろにおけるのでしょうか?
When girls put on the platform shoes that are fashionable now, it is as if they are trying to step up to another level, to be someone they are not.
女の子たちが、今はやりの厚底靴をはくときには、まるで、別のレベルへ上がろうとしている、つまり、自分とは違うだれかになろうとしているようである。
0403名無しさん@英語勉強中 (ワッチョイW 9fb8-Is5t)
垢版 |
2020/08/20(木) 11:38:10.65ID:bmjB6s2Q0
when節をitで置き換えているだけかと。
0405名無しさん@英語勉強中 (ワッチョイ ff89-Dag0)
垢版 |
2020/08/20(木) 12:09:12.42ID:oI+ZLxHp0
>>402
(1) It is as if S V.
   (これこれこんな感じだ)
(2) It is like S V.
   (上の表現とだいたい同じような意味)
(3) It is not that S1 V1. It is rather that S2 V2.
   (これこれこうだというわけじゃなくて、あれこれこうというわけなんだよ)
(4) It is not as if S V.
   (これこれこうだというわけじゃあるまいに)
(5) It is not like S V.
   (上の表現と似たような意味)

まあ、こういう表現は(映画などでの)話し言葉や気楽なメールや娯楽小説みたいな文章の中で
頻発するので、理屈を言わずにそのまま覚えることだ。
0409名無しさん@英語勉強中 (スッップ Sd9f-bzhc)
垢版 |
2020/08/21(金) 16:12:47.31ID:DgEzrV9od
いわゆるタフ構文に関する質問です
it is dangerous to swim in this river.は
To swim in this river is dangerous.という普通の名詞的用法のように書いてはいけないのでしょうか。
This river is dangerous to swim in.と2通り可能というわけではないのでしょうか?
0411名無しさん@英語勉強中 (スッップ Sd9f-bzhc)
垢版 |
2020/08/21(金) 17:02:41.83ID:DgEzrV9od
ありがとうございます。
もうひとつ質問なんですが、
To play soccer is fun for meという文のfor meは意味上の主語といえるのでしょうか?
あまり言わないと思いますが、文法的にはFor me to play soccer is fun.が意味上の主語のつけ方になるかなと思うのですがどうでしょう?
0412三年英太郎 ◆3CZBjOt3.Y (ワッチョイ 7f86-Dag0)
垢版 |
2020/08/21(金) 17:20:43.36ID:cEgOw+N70
For .... が to 不定詞の直前にない限り、意味上の主語にはなりません!

>For me to play soccer is fun.

文法的には〇だけど、文末重心の原則であまり好まれないはず。
ウィズダムに次の例文あり

For him to say so, the news might be true.
0414名無しさん@英語勉強中 (ワッチョイW 7f62-bzhc)
垢版 |
2020/08/21(金) 20:06:44.47ID:cCC3CWW/0
ありがとうございました。
よくわかりました。
0415名無しさん@英語勉強中 (スププ Sd9f-R9HW)
垢版 |
2020/08/22(土) 23:15:05.83ID:aNM6/T55d
関係代名詞の主格とbe動詞は短縮できますか? the man who’s driving a car みたいに。
よろしくお願いします。
0421名無しさん@英語勉強中 (ワッチョイ 7fe2-Dv7q)
垢版 |
2020/08/25(火) 16:45:43.53ID:+bCpxPGF0
朝一番の ホームの 冷えた静けさが 好きさ♪
0422名無しさん@英語勉強中 (スップ Sd82-YAr2)
垢版 |
2020/08/26(水) 21:21:14.12ID:a5ZpNX0Td
自分で質問してワッチョイ変えて自分でテキストの文法の説明を読みながら答えている輩に注意しましょう。
0425名無しさん@英語勉強中 (ワッチョイ fee3-JMbW)
垢版 |
2020/08/27(木) 19:47:00.24ID:w2mGzJe30
ジーニアス英和大辞典より "be to do"

2((正式))[be to do]《◆述語動詞の時点から見た未来の動作を表す》

d[運命][通例過去時制で] …する運命になっている‖
After his accident, he was never to get a chance to play in an actual game.
事故の後, 彼は実戦に出場する機会を二度と得られなかった.
0433名無しさん@英語勉強中 (ワッチョイ 27a1-biH0)
垢版 |
2020/09/04(金) 06:25:01.71ID:cwPKy/z60
過去完了を使うには過去の基準点が必要なはずですが、この文章では過去の話でもないのに突然過去完了が登場します
これはなぜですか?

Those behind the niche businesses go to great lengths to perfect the authenticity of their fake assignments.
Some deliberately keep their prose childish to make their book reports — a popular assignment in Japan — look as if
they had been written by 10-year-olds, while others dig up old artworks made by their own children in the past and seek to profit off them, the official said.
(The Japan Times, 2018/8/29)
0435名無しさん@英語勉強中 (ワッチョイW 47b8-Zh5B)
垢版 |
2020/09/04(金) 10:29:40.23ID:35w/1D/X0
Humanity has “entered a pandemic era,” with the worldwide coronavirus outbreak likely the first of accelerating epidemics to come.

(世界的なコロナの大流行で人類はパンデミック時代に突入した)ここで文章が切れてもいいと思うのですが、likely以下の文法の構造が分かりません。付け足しみたいな感じなんでしょうか?
0437名無しさん@英語勉強中 (ワッチョイW 47b8-Zh5B)
垢版 |
2020/09/04(金) 10:52:11.49ID:35w/1D/X0
>>436
with以下が付け足しという事ですね。with以下の和訳を教えてください。
0438名無しさん@英語勉強中 (ワッチョイ c7bd-G4A8)
垢版 |
2020/09/04(金) 10:53:55.90ID:sASHScDX0
>>436
付帯状況のWITH は基本的にいつも
どうやって訳せばいいですか?

ーとしながらしか知らないので教えてください。
0439名無しさん@英語勉強中 (ワッチョイW 5fe6-uYbi)
垢版 |
2020/09/04(金) 12:32:25.94ID:cRQj2HDo0
付帯状況は同時生起
分詞構文が付帯状況かどうかは文脈依存

てれびみながらdsしてた
でも
てれびみつつdsしてた
テレビとdo一緒にしてた
でも同時生起の意味が入ればなんでもオーケーじゃないかな
0441名無しさん@英語勉強中 (ワッチョイ ffe3-ALHH)
垢版 |
2020/09/04(金) 13:15:50.47ID:9sUtHigv0
>>435
Humanity has “entered a pandemic era,” with the worldwide coronavirus outbreak likely the first of accelerating epidemics to come.

>(世界的なコロナの大流行で人類はパンデミック時代に突入した)ここで文章が切れてもいい
>(likely以下) 付け足しみたいな感じ…
同感。

「世界的なコロナの大流行*と共に*人類はパンデミック時代に突入した」ということで、このwithは「付帯状況」というよりもっとシンプルな用法だと思われる。

likely以下は、直前の"the worldwide coronavirus outbreak"に対する補足説明のようなもので、文法的には「同格」ということになろうかと。
"which is"をlikelyの直前に補って考えてもいいかもしれない。

日本語に訳す場合は、一旦切って、後から補足的に説明を加えるようにすれば分かりやすい日本語になる。

世界的なコロナの大流行で人類はパンデミック時代に突入した。
そして、今回のこの大流行は、今後襲い来る加速的に増加しつつある伝染病の最初のものであるかもしれない。
0443名無しさん@英語勉強中 (ワッチョイW 47b8-Zh5B)
垢版 |
2020/09/04(金) 13:55:07.71ID:35w/1D/X0
回答いただきありがとうございます。なるほど、同格なら和訳がしっくりきますね。
0444名無しさん@英語勉強中 (JPW 0H1f-UGDO)
垢版 |
2020/09/04(金) 21:44:43.58ID:w2wK0sFRH
Between the two pine tree stands a cherry tree.
この文のstandsのSはa cherry treeの三人称単数のSらしいのですが、どういう理屈ですか? Between以下の文が副詞なのは分かりますが、主語がstandsの後ろにきているのが違和感です
0446名無しさん@英語勉強中 (ワッチョイ 47f6-biH0)
垢版 |
2020/09/05(土) 02:16:14.52ID:jbKpLTq30
Much as in the 1970s when clubby Keynesianism gave way to Milton Friedman’s
austere monetarism, and in the 1990s when central banks were given their independence,
so the coronavirus marks a new paradigm.

上記の文でMuch as in とso の文法的役割、意味がわかりません。
as in をmuchで強調した形でしょうか。soは単に「だから」という意味なのか、
何かと呼応しているのでしょうか。よろしくお願いします。
前後は以下の通りです。

Our cover this week is about the profound shift taking place in economics as
a result of the covid-19 pandemic. Much as in the 1970s when clubby
Keynesianism gave way to Milton Friedman’s austere monetarism, and
in the 1990s when central banks were given their independence,
so the coronavirus marks a new paradigm. It is characterised by government
borrowing, money-printing and intervention in capital markets—all underpinned by low inflation.
0447名無しさん@英語勉強中 (ワッチョイW ffe3-L5BB)
垢版 |
2020/09/05(土) 08:18:30.74ID:SQA9AaOm0
>>446
以下、ジーニアス英和大辞典より抜粋:

【much】
6 だいたい, およそ‖
〜 the same ほぼ同じ《◆the same より意味が弱い》/
He spoke 〜 as she did. 彼は彼女とほぼ同じような調子で話した

【as A, sò B】
Aと同様にB;Aと同時にB 《◆A, B は節》‖
Just as British people enjoy their beer, so the Japanese enjoy their sake. イギリス人がビールをたしなむように日本人は酒をたしなむ《◆比較を強調するため just を置くこともある》
0451名無しさん@英語勉強中 (ワッチョイ 47f6-biH0)
垢版 |
2020/09/05(土) 23:47:19.42ID:jbKpLTq30
>>447-448
詳しい解説ありがとうございます!
およそという意味のmuchとas A, so Bという表現の組み合わせなんですね。
そしてちょっと変則的にAの部分が句であると。

非常に納得がいきました。自分でも辞書で確認してみようと思います。
助かりました。ありがとうございます!
0453名無しさん@英語勉強中 (スップ Sdff-TRHq)
垢版 |
2020/09/06(日) 11:57:05.35ID:E8w5Hgj/d
Much as S Vの間違い↑
0455名無しさん@英語勉強中 (ワッチョイW ffe3-L5BB)
垢版 |
2020/09/06(日) 12:26:56.49ID:dT5Sf9BT0
>>452
そこで例に挙げられている文。

Much as the car and the clock did in their time, so today the smartphone is poised to enrich lives, reshape entire industries and transform society.

このmuch ad SVは譲歩の副使節ではないし、倒置も生じていないと思いますが
0459名無しさん@英語勉強中 (ワッチョイ ffe3-ALHH)
垢版 |
2020/09/06(日) 13:16:29.20ID:dT5Sf9BT0
譲歩のasの例文をジーニアス英和大辞典から引用。

Much as I admire him as a writer, I do not like him as a man.
作家としては大いに彼には敬服するが, 1人の人間としては好きではない.。

この用法と混同したんでしょう。
0461名無しさん@英語勉強中 (ワッチョイ ffe3-ALHH)
垢版 |
2020/09/06(日) 15:49:33.23ID:dT5Sf9BT0
>>460
結論から言うと、an unusual warm day も可だろうと思います。
長くなりますが、『現代英文法講義』安藤貞雄から適宜抜粋して引用します。(p.483)

23.3 形容詞の配列順序

a. both/all/half --- both
b. 限定詞 --- the
c. 序数詞 --- last
d. 基数詞 --- two
e. 評価 ---nice
f. 寸法 --- big
g. 年齢・温度 --- old
h. 形状 --- round
i. 色彩 --- red
j. 分詞 --- carved
k. 出所 --- French
l. 材料 --- wooden
m. (動)名詞 --- card
n. 主要語 --- tables

【1】 (e)〜(l)の各カテゴリーから一つずつ選んで併置する場合には、通例、andも要らないし、コンマも付けない。
an attractive young Welsh girl. [b (e,g,k) n]
魅力的な、うら若い、ウェールズ人の少女

【2】 これに対して、(e)〜(j)の各カテゴリーから一つの形容詞を選び、合わせて二つ以上の形容詞を併置した場合は、次のようにコンマを付けてもよい。
two large(,) oval mirrors [d (f,h) n]
二つの大きな楕円形の鏡

【3】 同一のカテゴリーから二つ以上の形容詞を選んだ場合は、等位接続になるので、andの機能を示すものとして、義務的にコンマを付ける。
You stupid, unthinking, irresponsible, reckless idiot! [すべて(e)]
このとんまな、考えのない、無責任な、向う見ずなばか者め!
0462名無しさん@英語勉強中 (ワッチョイ ffe3-ALHH)
垢版 |
2020/09/06(日) 15:51:08.68ID:dT5Sf9BT0
【1】の【2】の違いが今一つよく分からないのですが、
an unusual warm dayは、[a (e, g) n]ということで【1】に相当するので、
コンマ無しのこのままの形で文法的に一応正しい形であると思われます。


Google検索でのヒット数は次のようになりました。

an usually warm day 約 504,000 件
an usual warm day 約 4,260 件

文法的には間違いとは言えないけれども、実際に使われることは圧倒的に少ないようです。
0464名無しさん@英語勉強中 (ワッチョイ ffe3-ALHH)
垢版 |
2020/09/06(日) 15:59:32.77ID:dT5Sf9BT0
warmをhotに置き換えると、両者の差は小さくなるようです。

an unusually hot day 約 108,000 件
an unusual hot day 約 10,100 件

hot dayがunusualである頻度は、warm dayがunusualである頻度より高い、ということでしょうか。
0465名無しさん@英語勉強中 (ワッチョイ ffe3-ALHH)
垢版 |
2020/09/06(日) 18:02:05.63ID:dT5Sf9BT0
>>461の【1】と【2】の違いについてですが、
【1】(e)〜(l) = (k),(l)を含む
【2】(e)〜(j) = (k),(l)を含まない
ということなのかも。

とすると、an unusual warm day [a (e, g) n]は、(k),(l)を含まないので【2】に該当し、
an unusual, warm dayとコンマを付けても良いということになる、のかもしれません。

ちなみにこの【1】とか【2】という番号は、原典にはありません。
説明のために私が勝手に付したものです。
0467名無しさん@英語勉強中 (ワッチョイ 5fe6-r3rb)
垢版 |
2020/09/06(日) 20:30:59.58ID:2fVVjM0C0
>>465
e. 評価 ---nice opinion
f. 寸法 --- big shape
g. 年齢・温度 --- old age
h. 形状 --- round
i. 色彩 --- red color
j. 分詞 --- carved
k. 出所 --- French origin
l. 材料 --- wooden material


ということで
opshacom
と覚えましょう
http://www.professorgrammar.com/opshacom3.shtml
0470名無しさん@英語勉強中 (ワッチョイ ffe3-ALHH)
垢版 |
2020/09/07(月) 11:50:19.71ID:HURxSrNE0
日本語でも、「異常な暑い日」という言い方は文法的に間違っているとは言えないだろう。
ただし、その使用頻度は圧倒的に少ない。

"異常に暑い日" 約 174,000件
"異常な暑い日" 約 5.620件

"異常に暖かい日" 約 34,700件
"異常な暖かい日" 約 1,250件
0472名無しさん@英語勉強中 (ワッチョイW 5fe6-uYbi)
垢版 |
2020/09/07(月) 14:39:22.56ID:jY9sCLxR0
>>455
asnナンチャラの最初の副詞句を譲歩と訳すこともできるよね。
状況に応じて。

その投稿って2015年。2012年にmba留学から帰ってきて、そのあとどうしたんだろう。
mba 崩れの英語教師と書いてるね。2020年に。再び留学するつもりらしい。
お金どうするんだろう?
0473名無しさん@英語勉強中 (ワッチョイW 5fe6-uYbi)
垢版 |
2020/09/07(月) 14:46:12.09ID:jY9sCLxR0
>>444
これって仲本浩喜の動画でやってたような気がする。
betweenとあるから、主語ではない。動詞が来たけど、主語がないので、後で主語が来るんだろう。
で、三人称単数が来た。動詞のsはこれを主語とする動詞の意味だったんだな、
こうわかります、と言ってた。
仲本浩喜の動画を、これを見れば駿台英語が分かると書いてる人がいたが仲本は今は代ゼミだったっけ。
0474名無しさん@英語勉強中 (ワッチョイ 5f5d-th+2)
垢版 |
2020/09/08(火) 10:03:14.00ID:QEE73jel0
・I was in the bookstore for ten minutes.
(私はその書店に10分間居た。)という文を
・I had my purse stolen during the ten minutes I was in the bookstore.
(私はその書店に10分間居た間に財布を盗られた。)という文の節として用いると、
期間を表す前置詞のfor は要らないんだよね。理由は知らないが要らないということでOK?
0476名無しさん@英語勉強中 (SGW 0H1f-g0Ir)
垢版 |
2020/09/08(火) 10:57:50.63ID:vYkQXw0jH
シンガポール人にAll your base are belong to us を見せたら、
are はいらない 以外に文法についての指摘はなかった・・・。
シンガポール人の英語力ってそんなもんかね?
0480名無しさん@英語勉強中 (SGW 0H1f-g0Ir)
垢版 |
2020/09/08(火) 12:09:22.31ID:vYkQXw0jH
その人は中華系、英語ネイティブ(自称)のシンガポール人
事あるごとに英語ネイティブにとっては〜って言うから、どれほどのもんなのか気になってた
0481名無しさん@英語勉強中 (キュッキュ a66c-sa+Q)
垢版 |
2020/09/09(水) 10:02:33.69ID:/9y8xtcT00909
シンデレラの台本
The sole and express purpose of such quest to be as followの toの用法
(is) toでいいの?

Grand Duke:
¨It is upon this day decreed that a quest be instituted throughout the length of our domain.
The sole and express purpose of such quest to be as follows:
That every maid in our beloved kingdom, without exception, shall try on her foot this slipper of glass,
and should one be found upon whose foot this slipper shall properly fit
such maiden shall be acclaimed the object of this search and shall be looked upon
as the true love of His Royal Highness, our beloved son and heir, the Noble Prince.
0483名無しさん@英語勉強中 (キュッキュ a66c-sa+Q)
垢版 |
2020/09/09(水) 12:06:35.25ID:/9y8xtcT00909
これ最後のほうのセリフ

>>482だとこうなってる
Grand Duke: Yes, quite so. (reads) It is upon this day decreed that a quest be instituted
throughout the length and breadth of our domain... the sole and express purpose of said quest to be as follows.
音声聞くと
as followsのあとになんかいってるっぽい
0484名無しさん@英語勉強中 (キュッキュ a66c-sa+Q)
垢版 |
2020/09/09(水) 12:23:30.84ID:/9y8xtcT00909
あとA Dream Is A Wish Your Heart Makesの歌詞が出てくるんだが

Your rainbow
Will come smiling through

come smiling throughがいまひとつわからん
虹がsmileの様態でcome through(あらわれる)ってことか? 
0485名無しさん@英語勉強中 (キュッキュ 6689-m1Uj)
垢版 |
2020/09/09(水) 13:00:35.38ID:Wi+438kg00909
>>481
The sole and express purpose of such quest to be as follows:

あなたの言う通り、is to be as follows という意味だと解釈できると思う。


>>484
Your rainbow
Will come smiling through

これも、あなたの言うような意味合いだと思う。
come through は、The sun is coming through.
というような使い方をするけど、これは「日差しが(こっちの方まで)差し込んできている」
というような意味だろう。それと同じく、
Your rainbow will come smiling through.
も、「君の好きな rainbow の光が、微笑みながらこっちまで注ぎ込んでくるだろう」
というような意味だろうと思う。
0487名無しさん@英語勉強中 (キュッキュ 6af0-m1Uj)
垢版 |
2020/09/09(水) 13:13:05.35ID:KtZbdepT00909
>>481
cinderella proclamation で画像検索してみろ
違ってるぞ
0489名無しさん@英語勉強中 (ワッチョイ a532-Fu73)
垢版 |
2020/09/10(木) 00:13:51.78ID:v8aKgaLF0
I read a book an then I was going to sleep.

本を読んでから、眠る予定だった??
0490名無しさん@英語勉強中 (ワッチョイ a532-Fu73)
垢版 |
2020/09/10(木) 00:20:55.90ID:v8aKgaLF0
I read a book an then I was going to sleep.

本を読んでから、眠る予定だった??
0491名無しさん@英語勉強中 (オッペケ Srbd-Wjye)
垢版 |
2020/09/11(金) 01:22:32.21ID:B4j5wSfzr
間接疑問文はどういう時に使いますか?
会話であれば疑問文でよくないですか
私は知らない彼が誰なのか。のように
わざわざ自己の意見を挟む必要性を感じません
主に文章で使うような文法ですか?
0492名無しさん@英語勉強中 (ワッチョイW 2a4b-gY+k)
垢版 |
2020/09/11(金) 07:12:41.60ID:CaAWnnqR0
>>491
日本語の間接疑問文と同じように、日常のあらゆるシーンで使われてる

「何の話してるのかわからん」
「昨日の昼何食べたか覚えてない」
「あいつが本当はどんなやつか知ってる」

“I don’t know what you’re talking about.”
“I can’t remember what I had for lunch yesterday.”
“I know who he really is.”

1〜2個目は「わからん」や「覚えてない」という意見を挟まずに「何の話してるの?」や「昨日の昼何食べたっけ?」と表現できるけど、だからと言って会話で使われないわけではない
3個目みたいなのはそもそも単純な疑問文に変換すらできない
0494名無しさん@英語勉強中 (ワッチョイW deac-annK)
垢版 |
2020/09/12(土) 10:05:22.12ID:W6mgVH4K0
名1 前 名2 関係代名詞
こういう文だと名1に関係代名詞はかかるって見たんだけど
逆に名2にかかることもある?
結局最後は文脈で判断しろってことになるのかな?
0500名無しさん@英語勉強中 (ワッチョイ 6689-m1Uj)
垢版 |
2020/09/12(土) 11:02:47.73ID:rZJO1hnA0
>>494
次の二つの文にある that も、やはりその直前の Mexico にかかっている。
その前の名詞にはかかっていない。なぜだかわからない人はいるだろうか?
わからないなら、あと10年くらい修行する必要がある。俺は答えない。
Mexico の前に the がついているのは typo だなどという人は、論外。

(1) Nash in The American West in the Mexico ●that● they claimed gave birth.

(2) "Under The Volcano" (物語のタイトル) was begun in Hollywood, coloured
by a short stay in the Mexico ●that● it describes, and eventually finished in Dollarton, British Columbia.
0501名無しさん@英語勉強中 (ワッチョイ 6689-m1Uj)
垢版 |
2020/09/12(土) 11:32:03.79ID:rZJO1hnA0
>>495-498 にて、Japan などの固有名詞に a がつくはずがないと
思っている人がいたら、その人も修業が足りない。これは
typo ではない。ちゃんとした理由があって a がついている。
俺はこれについて解説するつもりはない。
0503名無しさん@英語勉強中 (ワッチョイ 6689-m1Uj)
垢版 |
2020/09/12(土) 13:47:40.07ID:rZJO1hnA0
>>494
(1) Beginning with Oscar Wilde's well-known quip that “[a] map of the world
●that● does not include Utopia is not even worth glancing at,” this introduction examines ...

このように、関係代名詞の直前の単語 world に定冠詞 the がついているときには、
普通の「世界」という意味になる場合がほとんどであり、それに対して that という
関係代名詞をつけて修飾して意味を限定する必要がなくなりやすいので、自然と
その前の "a map" に関係代名詞がかかるということになる。

(2) We are the cause of a world ●that's● gone wrong.

ところが、このように関係代名詞の直前の world に the ではなくて a がつくと、
今度は急にそれを関係節によって意味を限定しなくてはならなくなる。したがって、
関係代名詞はその直前の "a world" にかかるのであって、その前の
the cause にはかからないということになる。
0505名無しさん@英語勉強中 (ワッチョイ 6689-m1Uj)
垢版 |
2020/09/12(土) 19:41:04.47ID:rZJO1hnA0
解説はしない、などと俺は言ったけど、気まぐれで解説がしたくなってきた。
>>504 がお礼を言ってくれたからでもある。
たくさんの例文を挙げたけど、主にそれは次のような形をしていた。

(1) 名詞1 + 前置詞 + 冠詞 (a または the) + [通常は冠詞のつかない固有名詞] + 関係節(つまり関係代名詞など)

上のように、Japan, Mexico, Tom などの、通常は冠詞のつかない固有名詞に a とか the という冠詞がつくと、
急にそれには関係節がつき、その固有名詞が限定されるのだ。つまり、関係節は
その直前の "冠詞 + 固有名詞" にかかる。

しかしJapan, Mexico, Tom など、通常は冠詞のつかない固有名詞に通常通り冠詞がついていないときには、
急に関係節がその固有名詞にはかからず、その前の普通の名詞にかかることになる。
具体的な例に沿って解説すると、

>>498 にて
Matthew C. Perry's expeditions in 1853 and 1854 stirred tremendous excitement in a Japan
●that● had been largely closed to Westerners for over two hundred ...

このような例文を挙げたが、関係節である "that had been.." は、もしその直前の Japan に
冠詞がついていなかったら、その前の普通名詞にかかっていたかもしれない。
しかしここでは "a Japan" となっている。このせいで、"a Japan" は急にそのあとに
関係節をつけたくなってくるのだ。

もちろん、"a Japan" になったからと言って、必ずそのあとに関係節がついて、それが
"a Japan" を限定するとは限らない。いろんな例がありうる。でも今回俺が上にたくさん
列挙した数々の例文においては、おそらくすべて関係節がその直前の固有名詞を
限定しているはずだ。
0506名無しさん@英語勉強中 (ワッチョイ 6689-m1Uj)
垢版 |
2020/09/12(土) 19:46:21.07ID:rZJO1hnA0
>>505 で挙げたような用例を Google 検索したければ、

(1) "a Japan that"
(2) "a Mexico which"
(3) "a Mary that"
(4) "a United States that"
(5) "a Russia which"

などのフレーズを Google の穴の中に挿入して
Google を絶叫させればよい。たくさんの用例が
出てくるだろう。

ところで、(4) では通常 "the United States" となるはずのところを
"a United States" となっているので奇妙に感じるかもしれないが、
これも正しい英語だ。その直後に関係節をつければ、途端に
由緒正しい英文になる。
0507名無しさん@英語勉強中 (ワッチョイ 6689-m1Uj)
垢版 |
2020/09/12(土) 20:00:37.46ID:rZJO1hnA0
"a United States that ..." なんて、もともと the United States が複数形なのに、その the を消すだけでなく、
複数形の名詞に a をつけるなんて究極の変態だという感じがしてしまうが、これがまた正しい英語なんだから面白い。
もちろん、この場合は "a United States (that ...)" が複数のものを
指すというよりも、その集合体が一つのように感じられるから a がついているのだろう。
よく似たものを再び探してみる。

(1) UNA-USA creates a powerful national constituency
for ●a United Nations that● advances American interests in a global system.

上の例でもやはり、the United Nations と通常なら書くところを "a United Nations" とし、
そのあとの関係節によって限定している。

(2) ''There's a tremendous potential for ●a Soviet Union that● is in accord
with us on these democratic values,'' he said. (New York Times)

上の例では、"the Soviet Union" と通常ならなるところを "a Soviet Union" として、
そのあとの関係節にて限定している。

ところで、(1) ならば、もしも

(1-a) (*) UNA-USA creates a powerful national constituency
for ●the United Nations that● advances American interests in a global system.

というふうに、.United Nations に the をつけたままで書いたら、間違いとなるだろう。
その直後の関係節は、そのずっと前の "a powerful national constituency"
にかかるかな、というふうに英語ネイティブは考えるだろう。もしも
that 以下の関係節をどうしても "the United Nations" にかけたいのであれば、
次のように「非制限用法」を使わないといけなくなるだろうと思うが、文法的にはこれで正しくはなるけど、
それで文脈的にもきちんとした英文になるかどうかは、俺にはわからない。

(1-b) UNA-USA creates a powerful national constituency
for ●the United Nations, which● advances American interests in a global system.
0508名無しさん@英語勉強中 (ワッチョイW a996-ZIlS)
垢版 |
2020/09/12(土) 21:47:00.69ID:93B/Ai640
he did his best to make her (understand) the problem.
括弧内はunderstandが正解で、understoodだと後ろに目的語となる名詞句がくるから不正解と書かれてあったのですが、どういう意味でしょうか? 
0510名無しさん@英語勉強中 (ワッチョイ b6e3-bL/7)
垢版 |
2020/09/12(土) 22:43:28.98ID:QdLhNcvR0
>>509
>あと、make O understood とゆー言い方は
>O が再帰代名詞じゃなきゃ不可

"make it understood by" site:https://www.gutenberg.org/
でググると、Gutenbergプロジェクトのサイト上に限定して""make it understood by"を検索できます。
結果1,740件ヒットしました。

1件目
Finished, by H. Rider Haggard
Of course her evidence would be in our favour, but to make it understood by a jury she would have to explain a great deal of which she might prefer not to speak.

"make (him OR her) understood by" site:https://www.gutenberg.org/
こうすると、itではなく、him または her が使われている例を検索します。
結果、1件もヒットせず。

"make (him OR her) understood by"
Gutenbergプロジェクトという限定を外して検索すると、
結果、約 41,200 件ヒット

再帰代名詞以外も可能ではないでしょうか?
0511三年英太郎 ◆3CZBjOt3.Y (ワッチョイ ea86-m1Uj)
垢版 |
2020/09/12(土) 23:47:38.52ID:sfmXzRAI0
じゃあ、訂正!

O が再帰代名詞以外のことは《まれ》
(it のぞく)

>make
> 10. b [[make A done]] (過去分詞は以下のような慣用表現で
> 使われるもの以外は《まれ》)
ウィズダム英和より
0512三年英太郎 ◆3CZBjOt3.Y (ワッチョイ ea86-m1Uj)
垢版 |
2020/09/13(日) 00:16:41.51ID:3lRFqEcX0
make this/that/things/my intentions understood 、
その他二三の例があったけど、
数の上では非常に少ないので、《まれ》と結論してよろしかろう
(COCA 調べ)
0513名無しさん@英語勉強中 (ワッチョイW 69af-nSoy)
垢版 |
2020/09/13(日) 13:39:54.56ID:46Xstbze0
すみません、教えてほしいのですが、
「私は、それらをたくさん持っています。」はI have many them.でいいですか?
0514名無しさん@英語勉強中 (JPW 0H92-ZIlS)
垢版 |
2020/09/13(日) 14:03:10.10ID:kobgqY0bH
there is no point our continuing such a argument.
なぜourはその位置なんでしょうか? 
どちらかというとargumentの前にありそうな気もするんですが
0517名無しさん@英語勉強中 (ワッチョイ 6df6-QmsY)
垢版 |
2020/09/14(月) 22:19:59.18ID:ZOzBCPFF0
While Suga is expected to largely follow his predecessor’s Abenomics policies,
recently he given a couple of indications of his personal views on economic
matters. On Thursday, Suga pointed to the need to raise the consumption tax
from the current 10 percent in the future to maintain Japan’s fiscal health,
given the nation’s graying population.

上の文の2行目にgivenとありますが、これはgaveの誤りですよね?
has givenもありでしょうか。
0520名無しさん@英語勉強中 (ワッチョイ 6632-QmsY)
垢版 |
2020/09/15(火) 07:15:44.49ID:1cIQt8OR0
Was your husband okay with you traveling abroad?

というのをみたのですが、何故 your traveling abroad
ではないのですか?これどういう文型なんでしょうか?
0521名無しさん@英語勉強中 (ワッチョイ 6632-QmsY)
垢版 |
2020/09/15(火) 08:55:31.86ID:1cIQt8OR0
I will be attending the meeting tomorrow.

I will attend the meeting tomorrow,

上の文章良くテストででるらしいですが、
明日会議に出席しているでしょう。と
下の文章と日本語だと同じになってしまう
気がするのですが、どう違うか教えて下さい。
0522名無しさん@英語勉強中 (ワッチョイW 2a4b-gY+k)
垢版 |
2020/09/15(火) 09:33:18.44ID:R3QuT4U00
>>520
どっちもOK
意味はほぼ同じだけど、若干のニュアンスの違いはあるらしい
所有格の場合その人物の行動に焦点を当てていて、目的格の場合その行動をする人物そのものに注目しているとか

所有格: 「旦那さん、あなたが海外旅行することには賛成だったの?」
目的格: 「旦那さん、海外旅行するあなたに賛成だったの?」
0523名無しさん@英語勉強中 (ワッチョイW 2a4b-gY+k)
垢版 |
2020/09/15(火) 09:59:46.02ID:R3QuT4U00
>>521
“will attend”: そのイベントに参加します!
“will be attending”: その日のその時間はそのイベントに出席してるだろうなあ
みたいな感じ?

上は、参加することを伝えてる
下は、参加自体はすでに決まってて、その予定を説明してる
0524名無しさん@英語勉強中 (ワッチョイ 6632-QmsY)
垢版 |
2020/09/15(火) 13:00:52.78ID:1cIQt8OR0
>>522

有難うございます。ニュアンスの違いが
あるのですね。もし自分で分を作るなら
Youは間違えだと思ってしまうと思いま
した。

with you traveling abroad?

witn 名詞 動名詞、名詞??

この並びからしてよくわかりません。
所有格ならわかるのですが、これは
具体的にどうなっているのでしょうか?

動名詞が前の名刺を修飾しているの
でしょうか?それとも後?
0526名無しさん@英語勉強中 (ワッチョイ 6632-QmsY)
垢版 |
2020/09/15(火) 14:22:42.60ID:1cIQt8OR0
>>525

有難う御座います。
調べました。
目的語のyou。

あなた「が」海外旅行を
「すること」でいいでしょうか?
0527三年英太郎 ◆3CZBjOt3.Y (ワッチョイ ea86-m1Uj)
垢版 |
2020/09/15(火) 14:29:00.56ID:uYRw1m290
with you traveling abroad
with your traveling abroad

前者の方が口語的
とくに前置詞のあとには目的格がおかれやすく、
所有格だとかために響くらしい

意味の違いはたいしてないだろう
あるという人もいるけど、ないという人もいるから、
まあその程度の差しかないと
0530名無しさん@英語勉強中 (ワッチョイ ff89-b3rt)
垢版 |
2020/09/16(水) 13:29:01.86ID:/Y2pDki90
>>529
drop by の by は、前置詞としても副詞としてもちゃんと前後の意味が通じる。つまり

(1) I'll drop by at this shop.
(2) I'll drop by this shop.

この二つは、ちゃんと意味が通じている。だからこそこの二通りの言い回しが
定着したんだろうよ。しかし

drop in の場合は、
(3) I'll drop in at this shop.
ならばちゃんと意味が通じるが、
(4) (*) I'll drop in this shop.
は、通常の感覚で考えると変に感じられるはずだ。ただし "drop in this shop" を検索すると、
一応はいろいろと例文は出てくる。しかしどうやら、英語ネイティブが書いた英文ではないらしい。
日本人とかフランス人などが書いたものがずらずらと並んでいる。

ただし、英語ネイティブのあいだでも、最近は崩れた英語として若い人のあいだでは通用しているかもしれない。
しかし伝統的にはこれは変だと感じられていたはずだ。その場合には
(5) I'll drop into this shop.
とするのが、少なくとももともと自然な英語だったはずだ。
0532名無しさん@英語勉強中 (JPW 0Hdf-xmxa)
垢版 |
2020/09/16(水) 20:26:05.32ID:M8vfZ9ozH
their parents and teacher were more cautious about risk, with teachers slightly more likely to see high risk.

この文章のwith以降の構造が分かりません。解説を見ると、teachersを主語とすると書いてあったのですが、前置詞の後の名詞は主語になり得ないのではないでしょうか? また、likelyにbe動詞がないのもなぜですか?
0533名無しさん@英語勉強中 (ワッチョイ 7fe3-5MQP)
垢版 |
2020/09/16(水) 21:47:39.77ID:6V0l58VX0
>>532
いわゆる付帯状況のwith

***
ランダムハウス英語辞典から引用

(2)((付帯状況)) ((通例 with… の後に補語を伴って))…した状態で,…して,…しながら
補語は形容詞,副詞,分詞,前置詞句など:
sleep with the window open 窓をあけて眠る
with tears in one's eyes 目に涙を浮かべて
walk with one shoe off and one shoe on 片方の靴は脱ぎ一方の靴は履いたままで
walk (with) (a) pipe in (one's) mouth パイプをくわえて散歩する
***

上の引用中、((通例 with… の後に補語を伴って))の…にあたる部分が、補語に対応する「主語」。

最初の例では with <the window (S)> <open(C)>

あなたの挙げた例では、
with <teachers(S)> <slightly more likely to see high risk(C)>.

ちなみに、withを省くと独立分詞構文になる。
〜, teachers (being) slightly more likely to see high risk.
0534名無しさん@英語勉強中 (ワッチョイ 17f6-lMKa)
垢版 |
2020/09/16(水) 23:30:45.08ID:afbbioEE0
@Maddow discusses HHS spokesman Michael Caputo's comments during a
Facebook Live: "This is really who they've got as the gatekeeper on Covid
information for the public and for the world? Really?"

この文のThis is really who の構文がよくわかりません。意味はわかりますが、
これはどういう文法項目でしょうか。先行詞を含むwho(そういうのがあるとしたら)
でしょうか。「This is who 主語 動詞」で 「これが主語が動詞した人だ」という感じの
決まった形でしょうか。よろしくお願いします。
0536名無しさん@英語勉強中 (ワッチョイ 17f6-lMKa)
垢版 |
2020/09/17(木) 00:33:30.50ID:enEAfH230
>>535
ありがとうございます。見ました。
2b【先行詞を含んで】…する[である]人(はだれでも);〔主格で〕((古))…する[である]人は(だれもが)
He wasn't who he is now.
昔の彼は今とは違った
She can marry who [whoever] she chooses.
彼女は自分の好きな人を選んで結婚できる(◆関係節の動詞はchoose,like,please,want,wishなど)

この用法ということでしょうか。「関係節の動詞はchoose,like,please,want,wishなど」
と説明されています。質問の文では've got=haveですが、それでもOKですかね。
0538名無しさん@英語勉強中 (ワッチョイ ff89-b3rt)
垢版 |
2020/09/17(木) 06:25:53.49ID:/CPMDFRc0
>>534
文法書に載っている文法項目としてしっかり理解しないと気が済まないいなら、
そういう理解の仕方を追求したらよいと思う。でも俺なんかは
英文を読んだり聴いたりしているうちに次のような言い回しがじゃんじゃん出てくるから、
その延長線上で This is who S V. も理解している。文法書とか辞書で確かめたことはない。

(1) This is what you came for. (歌の題名)
(2) This is when Oman will reopen its borders to tourists.
(3) This Is How (We Want You To Get High) (Michael Jackson の歌の題名)
(4) This Is Where I Leave You (映画のタイトル)
(5) This is who I am. (歌の題名)

映画や歌のタイトルになりやすいらしいので、この言い回しはよほどパンチが利いているんだろうな。
0539名無しさん@英語勉強中 (ワッチョイ 579d-lMKa)
垢版 |
2020/09/17(木) 19:36:38.01ID:P1m6R3Ju0
love me be with youってのは正しい英語ですか?
0540名無しさん@英語勉強中 (ワッチョイ 17f6-lMKa)
垢版 |
2020/09/17(木) 22:20:24.65ID:enEAfH230
>>537
そうですね。なぜか勘違いしていました。その例文がドンピシャのようですね。
ありがとうございます!

>>538
>文法書に載っている文法項目としてしっかり理解しないと気が済まないいなら、

なんというか、あいまいな部分を残したくないと思ったので、文法項目で確認したかった
だけです。5番まで挙げていただいた例文のうち特に、This is how, This is when,
This is whereはこのままの形で馴染みがあったんですが、This is whoについては
「ん?」という感だったので、質問させてもらいました。

5については、who am I? が補語の部分に来たので、語順が変わってwho I amに
なったわけではないのですか?つまりwhoは疑問詞ではないのでしょうか。
I don't know who he is.のwho he is は本来who is he?という語順ですが、
目的語になる際に語順が変わりますよね。5もそれと同じではないですか?
それとも、>>534の場合と同じ用法でしょうか。
0546名無しさん@英語勉強中 (ワッチョイ 97f0-8NzO)
垢版 |
2020/09/21(月) 23:52:49.49ID:33SJuZiC0
That area is so empty now that it's scary to walk through at night.
この文の that はどういう用法の that なのでしょうか?
名詞が前にないので接続詞でもない気がしますし、
副詞節でも関係代名詞でもないように見えます。
解説のあるサイトとかないでしょうか?
0549名無しさん@英語勉強中 (ワッチョイW 7fe3-9G3N)
垢版 |
2020/09/22(火) 14:48:26.22ID:Imy4+M530
いや、so thatの構文は今のところ公立中学の指導要領外のはず。近々指導要領が改定されて大幅にレベルアップするらしいから、今後中学レベルに入ってくる可能性は高い。
0551名無しさん@英語勉強中 (ワッチョイ 7fe3-5MQP)
垢版 |
2020/09/22(火) 16:31:30.81ID:Imy4+M530
指導要領について調べて見た。

中学校学習指導要領(平成29年告示)解説
外国語編
https://www.mext.go.jp/component/a_menu/education/micro_detail/__icsFiles/afieldfile/2019/03/18/1387018_010.pdf

「平成33年度(すなわち令和2年度)から全面的に実施することとし」、とあるから既に実施されているのだろうか?
コロナでそれどころではなかったような気もするが。

文構造や文法事項についてはp.36以降に詳しい記述がある。
ざっとみた限り、so that 構文は導入されていない様子。
仮定法が盛り込まれているのがちょっと驚き。

語彙レベルでは、従来の「1200語程度」から、「小学校で学習した語に1600〜1800語程度の新語を加えた語」とされている。(p.33)
これは今どきの平均的中学生には、こなしきれない量なんじゃないかと危ぶまれる。
0553名無しさん@英語勉強中 (ワッチョイW 12e3-sqez)
垢版 |
2020/09/23(水) 22:37:34.09ID:jwN0wb8X0
Increased traffic and security regulations make it hard for companies with delivery trucks.
交通量の増加やセキュリティー上の規制により、企業は配送トラックを利用しにくくなる。

この文のmake it hardが分かりません。前後の文脈にitが指す言葉も見当たりませんので、慣用表現make it(間に合う、成功する)の派生なのかなとも思ったのですが、それではhardが説明できず途方に暮れてます
0557名無しさん@英語勉強中 (ワッチョイW 12e3-sqez)
垢版 |
2020/09/24(木) 06:16:01.96ID:5N8NSDy20
回答ありがとうございます

>>554
ジーニアス引きましたが環境のitでよさそうです。第5文型で〜の状況を難しくするというニュアンスを意訳すれば配送トラックを利用しにくくなるとも訳せそうです

慣用句にしばしば見かけるitって環境のitって名前がついていたんですね。天候や時間などのitと区別がついていませんでした。大変勉強になりました。ありがとうございました。

>>555
itを形式目的語としてみた場合、真目的語が見当たりませんが、真目的語が省略される場合もあるのでしょうか(この場合文末にto use themが省略されているという解釈でしょうか)。
0558名無しさん@英語勉強中 (ワッチョイ 16b5-UzFq)
垢版 |
2020/09/24(木) 09:09:13.56ID:cYsUma+f0
>>553
>>557
横レスだが。

これは、make it hard ( difficult, easy, etc ) for ... to〜 という形式目的語の典型的な形で
ここでは、with delivery trucks が意味上の主語 ( companies ) につくことで、真目的語 to〜
を言わなくても何を指しているのか明らかだから省略しているだけ。
そのため、和訳を見ると、「利用しにくくなる」とその省略を補って訳しているようだ。


「状況のit」は Take it easy. などのように it が何かを指すわけでもなく漠然としたもの。

例えば、
Increased traffic and security regulations make it hard for ABC company. (※)
とだけで終わっており、聞き手に ABC company が何をしている会社かの知識が全くなければ
「何が難しくなるの?」と聞き返すことだろう。

次はある有名な小説の一節だが

He remained silent. She put her arms round his neck.
’Don’t make it difficult for me,’ she pleaded.
’Make what difficult?’
’For me to go to Venice and arrange things.’

ここでは、聞き手(彼)にとって it が何を指すか不明瞭だったから、聞き返している。

と以上書いたが、じゃあ今回の it を「状況のit」のように解釈したら絶対ダメかというと、
そういう訳でもないとは思う。
ただ、その場合は、提示された和訳のようにはならず、そういった会社にとっては、
経営環境も含めて、漠然と状況が悪くなるという意味になるし、今回のように with delivery trucks
がついておらず、(※)のような文だったら、やはり「何が(どうして)難しくなるの?」となってしまう。
0560名無しさん@英語勉強中 (ワッチョイW 12e3-sqez)
垢版 |
2020/09/24(木) 12:41:36.69ID:5N8NSDy20
>>558
詳しい解説ありがとうございます。大変よく分かりました。

日本語訳と照らし合わせると真目的語が省略されている解釈が妥当なのですね…

等位接続や比較構文に見られる定形化された省略を超えた、文脈を根拠とする省略を考え始めると英文の解釈が急に難しく感じます。
588さんの解説を拝読した後ですと、確かに「真目的語to〜を言わなくても何を指しているのか明らか」に感じることが出来るんですが、1人で読んでいるとまずこの「文脈上明らかな省略」が感じ取れないんですよね。根本的に読む力が足りてないんでしょうな……精進いたします…
0563名無しさん@英語勉強中 (ワッチョイW 03a0-BHph)
垢版 |
2020/09/25(金) 17:26:14.85ID:lMgR1bZT0
>>562
たいていの文法書では、動名詞の前につく所有格/目的格は、動名詞の「意味上の主語」と記述されています。
ジーニアス大英和でも、myをひくと、次のように記されています。

[動名詞の前で] 私が《◆動名詞の意味上の主語》‖Would you mind my [((略式)) me] smoking?

「意味上の主語ではなく主語そのものである」という説は、どういう文脈・学説の中で述べられるものなのでしょうか?
0564名無しさん@英語勉強中 (ワッチョイ 1286-mHP9)
垢版 |
2020/09/25(金) 17:50:02.81ID:neRTOw4g0
そーですよ。ロイヤルとかそこらの英和とか1世紀前の文法を
盲目的に引き写してる文法書は、そー書いてますネ。

説明がめんどいので・・・おひまでしたら、たとえば
中村捷 『実例解説英文法』 pp. 164,165 なんかをお読みください。
0565三年英太郎 ◆3CZBjOt3.Y (ワッチョイ 1286-mHP9)
垢版 |
2020/09/25(金) 17:54:03.09ID:neRTOw4g0
トリップ外れてた^^;

チェックしてないけど、現代英文法解説もたぶんそうだと思いますね〜。

どしてかっつーと、生成以後は、動名詞も to 不定詞も、
主語が潜在/明示の違いがあるだけの節(文)と考えるからです。
0570563 (ワッチョイ c6e3-F8K1)
垢版 |
2020/09/26(土) 09:20:43.39ID:hay7uQ990
>>565
なるほど『現代英文法講義』(安藤)でも確かに「意味上の主語」ではなく単に「主語」として扱ってますね。
ありがとうございました。
0572三年英太郎 ◆3CZBjOt3.Y (ワッチョイ 1286-mHP9)
垢版 |
2020/09/26(土) 14:35:10.80ID:zVp3sml90
あ、まじで 意味上の主語(sense subject)って斎藤秀三郎の発案みたいね

> 分詞句内に包摂された名詞と分詞との主述関係を新たに指摘した上で、
> 最終的に上記の知見を「意味上の主語」(=‘Sense-Subject’)として
> 用語・体系化したのは、斎藤秀三郎である(斎藤 1898: 133)
日本の学習英文法史 ―「国産」文法項目を中心に -

おれが最近気になってる「分詞構文」についても、
この論文に出てる。ので、ググってみて
0577名無しさん@英語勉強中 (ワッチョイ c6e3-F8K1)
垢版 |
2020/09/27(日) 09:57:46.57ID:k+/91jkt0
>>574
同感。

「意味上の主語」を「主語」として扱うとすると、
「Would you mind my smoking?」に対して、「 この文の主語は何か?」という問に、
「you と my です」と答えなければならなくなるのではないか?
このyouとmyを同列に扱うのはどう考えてもおかしい。

「この文の主語は何か」という問いの立て方自体に問題があるのだろうか?


>>575
ジーニアス大英和にはSVOOの文型表示あり。
ジーニアス第2版、4版、5版ではSVOのみ。(初版、第3版は未確認)

ランダムハウスには例文あり。
Fry an egg for me.[=Fry me an egg.] 卵を焼いてくれ

Longman 4th, COBUILD, OALD 8th ともに記述無。
0578名無しさん@英語勉強中 (ワッチョイ d2f0-zB+x)
垢版 |
2020/09/27(日) 11:09:54.61ID:sUwpCCpD0
>>577
詳しく調べて頂きまして、ありがとうございます。
fry の第四文型は、大辞典レベルには載ってるけど、ジーニアスの通常版には載っていない用法なんですね。
非常に勉強になりました。
0585名無しさん@英語勉強中 (ワッチョイ 129d-IM3Z)
垢版 |
2020/09/27(日) 12:24:18.64ID:BradrKU10
Merriam-Webster's Advanced Learner's English Dictionaryにも例文が載っている。

They fried us some chicken. = They fried some chicken for us.
0586名無しさん@英語勉強中 (ワッチョイ 1689-mHP9)
垢版 |
2020/09/27(日) 12:58:58.46ID:qi7vs2yh0
SVO の形を取る動詞が、たまには SVOO の形を取ることがある、というのは、
英語ネイティブにとってはおそらくは当たり前のことだから、いちいち
辞書には書かないのかもしれないよ。

たとえば、have という動詞は、俺たちの感覚では SVO の形を取り、たとえば
Have a Merry Christmas. とはいっても、Have yourself a Merry Christmas. とは
言わないんじゃないかと思うと、それは大間違いで、これがじゃんじゃん使われ、
Have yourself a ... という形でいろんなバラエティがある。

さらには、
Austin Rivers Just Had Himself a Night.
CJ had himself a game in OKC.
Le'Afa had himself a half of basketball!
こういう用例はいくらでも見つかる。
0588三年英太郎 ◆3CZBjOt3.Y (ワッチョイ 1286-mHP9)
垢版 |
2020/09/27(日) 13:41:46.01ID:vj4H/RWO0
1. I hope [that you will come to my party].
2. I hope [for you to come to my party].

どちらも埋め込み文だが、2の for you だけ
「意味上の主語」と呼ばなきゃいけない道理はない。

まして、2だけ「この文の主語はなにかという問に答えが二つになる!」
というトンチンカンなイチャモンをつけられる道理もない。
0591名無しさん@英語勉強中 (ワッチョイ e392-8I62)
垢版 |
2020/09/27(日) 14:11:41.74ID:TD6TCccd0
>>588 >>589
青色をみたら渡らなければならないと習った。
今日小学校にくるときに緑色の信号になったからわたったんだ。
でも家に帰ってきてテーブルの上に緑色の羊羹が載っていてとまどっている。
どこをどう渡ればいいんだろう?
0593名無しさん@英語勉強中 (ワッチョイ 1689-mHP9)
垢版 |
2020/09/27(日) 14:25:44.03ID:qi7vs2yh0
SVO の形を取ることの多い動詞が、たまには SVOO の形を取るということについて補足すると、
get も同じような感じだな。

You've got a beautiful girlfriend. (SVO)
この形がおそらくは圧倒的に多いとは思うけど、
You've got yourself a beautiful girlfriend. (SVOO)
こういう形を見聞きすることもある、たとえば Bruce Willis 主演の Diehard の最後の場面で、
悪者を退治したあとに Bruce Willis が奥さんと抱き合っているところを警察官が通りかかり、
警察官が奥さんに対して次のように言う。
You've got yourself a good man. (SVOO)

なお、get が SVOO の形を取ることもあるという点については、すでに辞書に紹介されてはいるけどね。
でも SVOO の形も取ることがあるということをいちいち辞書には書いていないような動詞も、あちこちに
あるだろうとは思う。そういう動詞に関しては、自分で英語を見聞きすることによって、自分で発見して
いくしかないわな。辞書だって、たいして英語のできない自称研究者たちが寄ってたかって作るわけだから、
もちろん不完全な部分も出てくる。辞書のユーザーの方がよく知っている場合だってありうる。
0594名無しさん@英語勉強中 (ワッチョイ c6e3-F8K1)
垢版 |
2020/09/27(日) 14:52:47.09ID:k+/91jkt0
>>587
>顕在・潜在とわず、 常に主語をもつからである。
ということなら、
>566-568の
明示されていれば「主語」、明示されていなければ「意味上の主語」という区分にはどういう意味があるのでしょうか?
0596三年英太郎 ◆3CZBjOt3.Y (ワッチョイ 1286-mHP9)
垢版 |
2020/09/27(日) 15:18:54.47ID:vj4H/RWO0
>>594
566は中村センセの言い方による。
この言い方は、斉藤秀三郎の定義と異なってる。

明示(顕在)・暗示(潜在)にかかわらず、主語は存在するのだよ。
でも明示されてないなら、「意味上の主語は〜」ってことわったほうが
分かりやすいじゃない。

> 595
主語は機能、主格は形態的分類。
主語だから絶対主格とゆーわけでもないし、
主格だから絶対主語とゆーわけでもない。

例1
- Who ate the cake?
- It's he/him!

例2
It was he/him who ate the cake.

例3
He and I/me are good friends.
0597名無しさん@英語勉強中 (ワッチョイ 2f5a-hCCa)
垢版 |
2020/09/27(日) 15:39:07.50ID:sWChsuuH0
文の定動詞の主語は主格になるけど、不定動詞節の主語は目的格になる
exceptional case-markingがあるということらしい。

https://en.wikipedia.org/wiki/Infinitive#Clauses_with_subject_in_the_accusative_case
[T]he subject of the infinitive is in the objective case (them, him)
in contrast to the nominative case that would be used with a finite verb....
Such accusative and infinitive constructions are present in Latin and Ancient Greek,
as well as many modern languages. The unusual case for the subject of an infinitive is
an example of exceptional case-marking, where the infinitive clause's role
being an object of a verb or preposition (want, for) overpowers
the pronoun's subjective role within the clause.

https://en.wikipedia.org/wiki/Gerund#%22Gerund%22_clauses_with_a_specified_subject
In traditional grammars, a grammatical subject has been defined in such a way that
it occurs only in finite clauses, where it is liable to "agree" with the "number"
of the finite verb form. Nevertheless, non-finite clauses imply a "doer" of the verb,
even if that doer is indefinite "someone or something".... Traditional grammarians
may object to the term subject for these "doers". And prescriptive grammarians
go further, objecting to the use of forms more appropriate to the subjects (or objects)
of finite clauses. The argument is that this results in two noun expressions
with no grammatical connection. They prefer to express the "doer" by a possessive form,
such as used with ordinary nouns..... The possessive construction with -ing clauses is
actually very rare in present-day English. Works of fiction show a moderate frequency,
but the construction is highly infrequent in other types of text.
0598名無しさん@英語勉強中 (SGW 0Haa-v5GC)
垢版 |
2020/09/27(日) 15:49:22.47ID:V4jFo73vH
>>552 へ回答くださった方ありがとうございます

She will miss me crying.
このままだと、彼女は私が泣いてるのを恋しく思う って意味ですよね? 彼女は泣きながら私を恋しく思う の意味にもなりますか?
0602三年英太郎 ◆3CZBjOt3.Y (ワッチョイ 1286-mHP9)
垢版 |
2020/09/27(日) 18:53:28.44ID:vj4H/RWO0
便利どころか間違ってるから

いま A Student's Introduction to English Grammar
少しずつ読んでるけど、現代英文法がいかにラディカルな
見直しがされてるか思い知らされるね

一例をあげると、接続詞(conjunction)という概念が消えて、
等位接続詞と呼ばれてたものは coordinator、
従属接続詞と呼ばれてたものは subordinator になってる

まだ詳しい説明のとこまでたどり着けてないけど、
おそらくこちらの方が簡潔に説明できると思われる
(それぞれ coordination と subordination のマーカーだから)
0603三年英太郎 ◆3CZBjOt3.Y (ワッチョイ 1286-mHP9)
垢版 |
2020/09/27(日) 18:55:53.52ID:vj4H/RWO0
ちょうど、in a hurry を副詞句じゃなくて前置詞句にするのと同じ

この句内のどこにも副詞がないのに副詞句認定するより、
主要部が前置詞なのだから、前置詞句にした方が簡潔で美しい
0606名無しさん@英語勉強中 (ワッチョイW c6e3-BHph)
垢版 |
2020/09/27(日) 19:59:28.69ID:k+/91jkt0
現代英文法でラディカルな見直しが行われて新たなパラダイムが生まれたとしても、それのみが真偽を判断する絶対的基準になるわけではないでしょう。
当面旧パラダイムも並行して存続し続けるだろうし、どちらか一方のみに立脚して他方の学説を「間違っている」と非難することは、学問に関わる者の態度として間違っているのではないでしょうか?
0607三年英太郎 ◆3CZBjOt3.Y (ワッチョイ 1286-mHP9)
垢版 |
2020/09/27(日) 20:38:42.41ID:vj4H/RWO0
自分で考えた上で間違ってると思うから、間違ってると言ってるのですよ^^

> Would you mind my/me smoking?

英語には「non-finite clauses 内では、主語が所有格か目的格で表れる」
というルールがあると考えれば、smoking の主語はまぎれもなく
my/me であり、「意味上」の冠をおく必要がない。
仮に「意味上」だとしたら、では機能上 my/me は smoking の何?

> Would you mind [PRO sleeping] with me tonight?

この文では、sleeping の主語が表面上見えないので、
「意味上の主語」という言い方を使える。

中村センセは、「意味上の主語」を生成でいうPROに当ててるのです。
0608名無しさん@英語勉強中 (ワッチョイ 1228-LNaZ)
垢版 |
2020/09/27(日) 22:53:50.67ID:yAvsgbZK0
現行の本で数パターンの用語が使われているなら、何も否定する必要はない
今もその説明のためにスレを消費している状態
この本はこう書いている、あの本ではどうだとか無駄
広く行きわたっている用語を使っておけばいい
別に専門家だけが見るスレでもあるまい
0610三年英太郎 ◆3CZBjOt3.Y (ワッチョイ 1286-mHP9)
垢版 |
2020/09/27(日) 23:55:38.01ID:vj4H/RWO0
ムダじゃないね
自分を絶えずアップデートさせるために5chやってんだもん

スレを消費って、閑古鳥が鳴いてるスレで何いっとんじゃ

お前が有意義なレスつけてから言えっつーの
0611三年英太郎 (ワッチョイW 1286-ZtgG)
垢版 |
2020/09/28(月) 01:34:19.81ID:M4tVFzAp0
現行で使われてるから否定する必要は
ないって、全くおかしいね👎

誰かが現行の慣習の間違いを是正してきたから
世の中が改善されてきたのにね

こーゆー人たちがガリレオを裁いたんだろうね
0613名無しさん@英語勉強中 (ワッチョイ 12e2-dHfp)
垢版 |
2020/09/28(月) 13:46:38.92ID:dCjQUBrc0
自分もA Student's Introduction to English Grammar読みはじめた。
新CGELを頭から読もうとして挫折して放置してたけど、
さすがに2002年出版でもう20年にもなろうとしてるから
放置しておくわけにもいかなくなってきたので。
Student'sの方は学部生・他学部生向けの入門書だから断然読みやすい。
0614名無しさん@英語勉強中 (ワッチョイ d66c-b+lb)
垢版 |
2020/09/28(月) 16:05:53.63ID:1pI9LyBH0
レベルの低い質問で申し訳ないんですが

I was in your store this morning waiting to buy something.

この文の waiting〜が何なのか分かりません
前置詞が省略されてるんでしょうか?
0615名無しさん@英語勉強中 (ワッチョイ d66c-b+lb)
垢版 |
2020/09/28(月) 16:14:54.47ID:1pI9LyBH0
今ぐぐったら分詞の副詞的用法ってあるんですね
忘れてました
すみません
0617名無しさん@英語勉強中 (ワッチョイ 6ff6-/QqT)
垢版 |
2020/09/28(月) 18:27:11.10ID:22UD63zi0
His returns examined at last, the president stands exposed as a tax avoider
and serial debtor.

guardianから。the president 以下の構文がわかりません。standsは立場、などの
名詞でしょうか。分詞構文的なんでしょうか。returns とは納税記録のようなものです。

His returns examined at last, the president stands (being) exposed as a tax avoider
and serial debtor.

よろしくお願いします。
0618名無しさん@英語勉強中 (ワッチョイ c6e3-F8K1)
垢版 |
2020/09/28(月) 18:39:47.89ID:qR/8eIet0
>>617
以下ジーニアス英和大辞典より抜粋

stand

5((正式))[SVC]〈人・物が〉…の状態で立っている;…(の状態・関係・立場)である《◆be 動詞に近い;C は名詞・形容詞(句)・分詞など》
〜 speechless 黙って立っている
The door 〜s open. 戸はあいている
She 〜s in great danger. 彼女はとても危険な状態にある
She 〜s accused of stealing the car. 彼女は自動車を盗んだとして告訴されている
0621名無しさん@英語勉強中 (ワッチョイ 6ff6-/QqT)
垢版 |
2020/09/28(月) 23:09:36.43ID:22UD63zi0
>>620
見出し的に単なるbe動詞の省略かな?と思っていました。そのあとのコンマとの
整合性はありませんが。言われてみればそこが分詞構文ですね。すっきりしました。
ありがとうございます。
0622名無しさん@英語勉強中 (ワッチョイ 1632-/QqT)
垢版 |
2020/09/29(火) 05:03:31.11ID:kglIqIO+0
He made a gesture indicating that it was okay to go ahead.

これはSVOCですか?
0623名無しさん@英語勉強中 (ワッチョイ 9232-r8bz)
垢版 |
2020/09/29(火) 05:46:10.83ID:IbOfcILr0
indicatingはその前の名詞gestureを後置修飾
よってSVOと解釈するのが自然かな
0624名無しさん@英語勉強中 (ワッチョイ 236c-ozis)
垢版 |
2020/09/29(火) 06:10:11.47ID:CMhgCSnh0
ぱっと見こう読んだ

He made a gesture indicating that it was okay to go ahead.

文型 SVO HeがS madeがO a gesture以下がO
He 三人称単数男性 彼は
made makeの過去形 make + a+動詞から派生した名詞の用法 個の場合実質的意
味はgesture
a gesture indicating that・・・ ・・を示すしぐさ
indicating 動詞indicateの現在分詞 (a) gestureにかかる
that 接続詞 動詞indicateの目的節を導く
it たぶん後方照応する形式主語のitだろ(そうでなかったらたぶん先行する名
詞に照応する代名詞) to go ahead が真主語
was be動詞の過去形 三人称単数
okay 形容詞の叙述用法 許可・同意 よろしい、結構な 
go ahead 計画や話を前に進める 

試訳
直訳ぎみ 彼は、進めてよいということを示すしぐさをした。
意訳   彼は、先を進めることに賛成のしぐさをした。
0627名無しさん@英語勉強中 (ワッチョイ 236c-ozis)
垢版 |
2020/09/29(火) 11:57:18.68ID:CMhgCSnh0
a gestureを heのする、あるしぐさ に限定されるならいい気がするけど
どうなんだろな
0633名無しさん@英語勉強中 (ワッチョイ b36c-G8Sl)
垢版 |
2020/09/30(水) 01:18:01.55ID:HQiEWUQ90
少しググった
どこのネイティブか知らんが
makeとleaveで語法がちがうらしい

The story made him feeling sad×
The story made him feel sad〇
He leaves the boy standing all the time〇

ただこういうing形はありらしい
The second example, as Fictional stated is correct: "He makes the boy stand all
the time." Again in this case, if you want to use the form of "to leave"
it can make it "standing" ← こういうing形
: "He leaves the boy standing all the time."

https://forum.wordreference.com/threads/make-somebody-doing-something-make-him-feeling-sad.2759353/
0634名無しさん@英語勉強中 (ワッチョイ b36c-G8Sl)
垢版 |
2020/09/30(水) 05:07:11.59ID:HQiEWUQ90
NHK WORLDのニュース(COVID19)に出てきた表現

takes a wrecking ball to
take (道具など)を・・に用いる、使う
a wrecking ball クレーンでつるした建物解体用の鉄球

Japanese officials say more than 60,000 people have lost
or are about to lose their jobs
- as COVID-19 takes a wrecking ball to a number of industries.

COVID19が諸産業に大きな被害を与えてることの比喩表現だと思うが
直訳だと日本語として通じにくい気がする
0635名無しさん@英語勉強中 (ワッチョイ b36c-G8Sl)
垢版 |
2020/09/30(水) 05:24:02.40ID:HQiEWUQ90
コリンズの英英辞書にいい定義が載ってた そのうち日本の英和辞書にも載るかも
wrecking ball
something that destroys the current situation
https://www.collinsdictionary.com/dictionary/english/wrecking-ball

使用例
Trump denies he'll take a 'wrecking ball' to Obama's legacy
https://video.foxnews.com/v/5244717866001#sp=show-clips

we cannot take a wrecking ball to the economy
https://www.msn.com/en-au/news/other/labor-shouldnt-take-a-wrecking-ball-to-the-economy/ar-AAJCnnx
0636名無しさん@英語勉強中 (ワッチョイ cf89-tG2C)
垢版 |
2020/09/30(水) 05:53:48.57ID:bfzWq3tr0
>>633
>>it can make it "standing" ← こういうing形
: "He leaves the boy standing all the time."

まさかとは思うけど、念のために書くけど、
上のようにネイティブが書いているからと言って、
make something standing
が正しいとは思っていないよね?

ネイティブによる回答文:
Again in this case, if you want to use the form of "to leave"
it can make it "standing": "He leaves the boy standing all the time."

ネイティブが言いたいのは、
「leave を使うときには、そのあとの部分 (it) を
standing にすることができる。つまり
He leaves the boy standing all the time. と書くことができる」
ということだよ。決して (*)make [something] standing
(たとえば <*>make it standing)
と書くことができる、という意味ではないよ。

まあ、すでにそのことが分かったうえで、あなたは別のことが
言いたかったんだろうとは思うけど。
0637名無しさん@英語勉強中 (ワッチョイ b36c-G8Sl)
垢版 |
2020/09/30(水) 06:07:36.01ID:HQiEWUQ90
>>636
そういう書き方は角が立つからやめたほうがいいとおもう
0638名無しさん@英語勉強中 (ワッチョイ cf89-tG2C)
垢版 |
2020/09/30(水) 06:26:53.03ID:bfzWq3tr0
>>637
できる限り角が立たないような書き方をしたつもりだけど、
余計に角が立つわけ?

じゃあ、あなたはどのように書く?あるいは、何も指摘しないで
黙っていた方がいいの?

第一、相手は本当にすべてを承知した上で例のコメントを書いたかもしれない。
それなのに俺が勝手に相手がこのことを理解していないと思い込んでいたとしたら、
それは無礼じゃないか。
0641名無しさん@英語勉強中 (ワッチョイW b392-KqvU)
垢版 |
2020/09/30(水) 06:55:16.80ID:wX/EX93+0
>>631
The speedometer is indicating 80 miles per hour.
というのがあるから現在分詞もあり

This sign indicates where the palace stood.
みたいな感じじゃないの?gesture indicateも。
a gesture indicateはありだと思うけど
0645名無しさん@英語勉強中 (ワッチョイW b392-KqvU)
垢版 |
2020/09/30(水) 07:12:19.80ID:wX/EX93+0
>>635
wrecking ballについては、googleでイメージを検索すれば鉄球が出てくるから、
感覚でどういうこと言ってるかわかるからいちいち
周りのものを壊し回る状況とかしらなくてもオケ
0646名無しさん@英語勉強中 (JPW 0Hdf-S4Yf)
垢版 |
2020/10/03(土) 12:56:46.78ID:7GYyVVfKH
leaveの配置がよく分かりません。
there are many of them left.
何かを残すみたいな文章の時、こういった形をよくみるんですが、このleftは副詞? 何を修飾してるんですか?
0647名無しさん@英語勉強中 (ワッチョイW 8fe3-wWsy)
垢版 |
2020/10/03(土) 14:33:56.86ID:QyVDfCbO0
>>646
ジーニアス英和辞典より
there
2 [there is [are] S] S がある, 〈人が〉いる

[語法]
(3)there is S の後に現在[過去]分詞や一時的な状態を示す形容詞(available, present, free)を伴うことがある:
T〜 are three people working there. そこでは3人が働いている/
T〜 were two prisoners being tortured. 2人の囚人が拷問を受けていた /
There's a credit card gone from my wallet. 私の札入れからクレジットカードがなくなっている /
T〜 were many students present [ill]. 多くの学生が出席していた[病気であった].
0649名無しさん@英語勉強中 (ワッチョイ cf32-tG2C)
垢版 |
2020/10/04(日) 23:35:04.96ID:QXze4Bh20
All amidst an industry which has been decimated by a pandemic which has been keeping people in their homes and out of the air.

CNNのニュース記事中にあったんですけど
動詞がない気がします
なんとなく意味はわかるけどなんか読めません
0652三年英太郎 ◆3CZBjOt3.Y (ワッチョイ ff86-tG2C)
垢版 |
2020/10/05(月) 03:03:22.55ID:l5kMmSj+0
there 構文は、倒置ではないね。
100歩ゆずって倒置としても、その基底構造の骨格は

Many of them are there.

という存在を示す文であって、 .... are left でない。
0654名無しさん@英語勉強中 (ワッチョイW cf32-fOvb)
垢版 |
2020/10/05(月) 14:25:04.40ID:o7cRPBfB0
In the ____ the sales teams do not achieve their objectives, no bonuses will be awarded.
1. case
2. pause
3. event
4. function
答えは3のeventなのですが、ネットで調べるとin the case that SV, 〜で書かれてる文章もあって、どうしてcaseじゃだめなんでしょうか。
0656名無しさん@英語勉強中 (ワッチョイW cf32-fOvb)
垢版 |
2020/10/05(月) 14:41:19.31ID:o7cRPBfB0
>>655
回答ありがとうございます。
in the case of 〜はOKだけど(こちらは英辞郎にも記載がありました)、in the case that SVはNGということでしょうか。
いまいち、eventにはtheがつくのにcaseにはつかない理由がよくわからず、申し訳ないです。
0659三年英太郎 ◆3CZBjOt3.Y (ワッチョイ ff86-tG2C)
垢版 |
2020/10/05(月) 15:35:27.50ID:l5kMmSj+0
あああ>>655は間違い

I will take my umbrella in case it rains.
雨が降る場合に備えて〜

... in the event it rains
雨が降った場合には〜

***********************************

次のアメリカ人は in the case SV が in the event SV と
同じみたいなことを言ってる
https://hinative.com/ja/questions/2293450

が! イギリスンの友だちは in the case = in case だと思ってて
>>654は3以外ダメだと言ってる

ま、テストなんて標準的な言い方に従っておいて方がいいでしょう
0661三年英太郎 ◆3CZBjOt3.Y (ワッチョイ ff86-tG2C)
垢版 |
2020/10/05(月) 15:39:38.22ID:l5kMmSj+0
ん! ウィズダムには

in case .... =if の用法も出てて、
《主に米》、《英》では《非標準》とされる場合もある、
とある。

とゆーわけで>>655は間違いともいえないな。

なんにせよ、テストなら標準的な言い方に従った方がいいでしょう。
0662名無しさん@英語勉強中 (ワッチョイW cf32-fOvb)
垢版 |
2020/10/05(月) 15:40:44.68ID:o7cRPBfB0
>>659
イギリスンに笑ってしまいましたw
ありがとうございます!
ネイティブからすると口語的表現で使う人もいるかもしれないけど、文法的にはin the event that SVが一般的だということですね。
0663三年英太郎 ◆3CZBjOt3.Y (ワッチョイ ff86-tG2C)
垢版 |
2020/10/05(月) 15:58:58.51ID:l5kMmSj+0
ついでにオージーの意見

In case the sales teams.... would be possible, but the rest of the sentence wouldn't really fit.
In the case the sales team.... is grammatically incorrect.

揺れが大きいですな
0665名無しさん@英語勉強中 (ワッチョイ 4ee3-eFyE)
垢版 |
2020/10/07(水) 19:41:34.35ID:nIVRaB2H0
NHKのラジオ講座『入門ビジネス英語』で次のような表現が出てきました。

no more than 〜 〜以内
We can spend no more than $1000 on the new computer..
新しいPCには1,000ドルまで使えます。

(以上、テキストの記述そのままの引用)

これ、「新しいPCには、たった1,000ドルしか使えない」という意味になると思っていたのですが、
この理解の仕方は間違っていて、NHKの解説の方が正しいのでしょうか?
0676名無しさん@英語勉強中 (ワッチョイ de32-Pfvf)
垢版 |
2020/10/08(木) 13:46:09.26ID:w/Ejnguv0
concerning your question, I should have an answer for you next week.

この後の文章がよくわかりません。I should have の後に過去分詞がこなくても
いいのでしょうか?? 訳には答えるべきと思うと書いてあって同じようですが。。
思うって感じはどこからでてくるのでしょうか?答えるべきだろうでもOK?
0683名無しさん@英語勉強中 (ワッチョイ 4ee3-eFyE)
垢版 |
2020/10/08(木) 17:28:15.11ID:kDiycVxk0
Longmanにはこんな風にまとめて記述されてました

4 not/no more than sth
used to emphasize that a particular number, amount, distance etc is not large
・It's a beautiful cottage not more than five minutes from the nearest beach.
・Opinion polls show that no more than 30% of people trust the government.
0689名無しさん@英語勉強中 (ワッチョイ 4ee3-eFyE)
垢版 |
2020/10/08(木) 17:40:47.25ID:kDiycVxk0
NHKのテキストから会話部分を抜粋してみます

Naoya: ...略... I was wondering if you had time to meet.
Kate: Sorry, but I'm almost fully booked that week..
Naoya: I see. But it should take no more than 30 minutes.

直也:...略... 会っていただける時間があるとありがたいのですが。
ケイト:すみませんが、その週はほとんど予定が埋まっています。
直也:そうですか。ですが、30分もかからないはずです。

この会話中でのno more thanとその日本語訳はまったく問題ないと思います。
「多くても30分しかかからない」ということで、30分と長さを、短いものとして否定的に述べてます。
(こういう場合、学校英語では not more than とすべきものとして教えられてきたはず)

で、>>665にもどりますが、

We can spend no more than $1000 on the new computer..
新しいPCには1,000ドルまで使えます。

これは1000ドルを肯定的にとらえてますよね。
やっぱりおかしいんじゃないですか?
0696名無しさん@英語勉強中 (ワッチョイ db5a-kFbH)
垢版 |
2020/10/08(木) 23:44:56.38ID:sjIsZvmU0
この文脈でshouldを「すべき」と取ったらほぼ意味不明。

“What's the point to it other than petty jealousy?”
“I―I gotta go. I'll talk to Tippy sometime today. I should have an answer by then.”

----

“There is one of the party that we're not sure about. He may be who he claims to be,
and maybe not. If it's all right with you, I'd like to drive back to Los Angeles
and send a cable to Scotland Yard. By tonight I should have an answer.”

----

Joseph sat at his desk and penned a letter to the general giving him the definite dates.
He requested a reply. He jogged to the embassy and sent a courier with the letter to Paris.
He should have an answer by midmorning Tuesday.
0703名無しさん@英語勉強中 (ワッチョイW 6f5c-X9Uo)
垢版 |
2020/10/09(金) 00:16:53.21ID:ScAnurE40
>>689
no more than =only 「千ドルだけ買える」>「千ドルまで買える」
これでなんの矛盾もないように思えるけど。

onlyの訳としての日本語の「しかーない」は英語話者が混乱する構文のひとつらしい。
0709三年英太郎 ◆3CZBjOt3.Y (ワッチョイ ca86-NY7j)
垢版 |
2020/10/09(金) 21:26:51.55ID:CAXNc5ql0
morite2先生(笑)のネイチブ腹話術の代わりに
おれがオージーに聞いてやったわ^^

問.
Their friendship lasted [no more than] a year.
上の [ ] は only に置き換えられるか?

答.
Yes. It's not exactly the same, but very close.

*************************************

ソースmorite2禁止します。レベル低すぎィ!
0713名無しさん@英語勉強中 (マグーロ MM13-+B3M)
垢版 |
2020/10/10(土) 12:37:12.19ID:coFYwbz8M1010
絶版先生こと松井先生が2015年のno+比較級+thanのツイートが今RTされて
「なぜ、今頃これがRTされているの?どこかで「トンデモ」の風が吹いた?」って驚いてるけど
このスレの人が見てRTしたんじゃないよね?
0714三年英太郎 ◆3CZBjOt3.Y (マグーロ ca86-NY7j)
垢版 |
2020/10/10(土) 13:01:17.40ID:VtOHa5Yd01010
英国人にも>>709の問題聞きました〜

***********************************


Oui, oui...

Just so you know that there is a subtle difference between the two sentenses.

[no more than]
Is a way of stressing the time line of the friendship.

[only]
Just makes a comment - interjection...


***********************************

>>711
継続時間の for は省略可
0715三年英太郎 ◆3CZBjOt3.Y (マグーロ ca86-NY7j)
垢版 |
2020/10/10(土) 13:09:22.21ID:VtOHa5Yd01010
スタンドと同じで、程度の低いもの同士は惹かれあうんだろう

それ自体は否定しないけども、もうちょっと上のレベルの場に
それを堂々とソースとして持ってこないで欲しいね^^
0716名無しさん@英語勉強中 (マグーロ Sa2f-ikay)
垢版 |
2020/10/10(土) 13:20:02.38ID:esxx63y/a1010
もりてつはthink of A as Bは誤解するネイティブがいるから避けたほうがいいと言っていた
菅のおかげで予備校講師のレベルの低さがよくわかったから信じていいのかわからん
0718名無しさん@英語勉強中 (ワッチョイW 863d-X9Uo)
垢版 |
2020/10/12(月) 06:30:49.80ID:kDPXy3Lz0
no more than もnot more than も「同等もしくはそれ以下=<」っていう本来の文字どおりの意味があると同時に主観的に「とても少ない」という暗示的意味が内在している。no more thanの方がその暗示的意味がより強い。だから結果としてonlyと同じような文脈で使われる。それでもno more than の文には「同等かそれ以下」っていう含意は常にある。
0722名無しさん@英語勉強中 (ワッチョイ 9f33-fMdf)
垢版 |
2020/10/16(金) 13:13:22.86ID:AuAFVfl+0
英語の文法をまともに勉強したことがありません。

We measure the distance from a to the successive points of the sequence, by the sequence of real numbers d(a, a1), d(a, a2), ….

aから点列連続する点たちへの距離を実数列 d(a, a1), d(a, a2), …によって測ることにする。

というような意味だと思います。

質問は、
We measure the distances from a to the successive points of the sequence
のように距離distanceをなぜ複数形にしないかということです。

連続する点が複数個あり複数形なのだから対応する距離も複数形にするのが正しいのではないか?でも実際には複数形にはなっていません。
0723名無しさん@英語勉強中 (ワッチョイ 9f33-fMdf)
垢版 |
2020/10/16(金) 13:13:59.88ID:AuAFVfl+0
訂正します:

英語の文法をまともに勉強したことがありません。

We measure the distance from a to the successive points of the sequence, by the sequence of real numbers d(a, a1), d(a, a2), ….

aから点列を構成する連続する点たちへの距離を実数列 d(a, a1), d(a, a2), …によって測ることにする。

というような意味だと思います。

質問は、
We measure the distances from a to the successive points of the sequence
のように距離distanceをなぜ複数形にしないかということです。

連続する点が複数個あり複数形なのだから対応する距離も複数形にするのが正しいのではないか?でも実際には複数形にはなっていません。
0724名無しさん@英語勉強中 (ワッチョイ 0f89-QI1E)
垢版 |
2020/10/16(金) 13:33:26.26ID:/pFEVQoU0
>>723
俺もいろんな英文を読んでいて、あなたと同じ疑問を抱くことがよくある。
ただ今回の distance については、可算名詞ではなくて不可算名詞なのだ
と考えれば、まったく矛盾がなくなる。ジーニアス英和には少なくとも、
distance には可算名詞と不可算名詞とがあると書いてある。

同じように、たとえば temperature も可算名詞と不可算名詞とがあるからこそ、
次のような temperature が複数になっていないことに納得ができる。

(1) A thermal camera will produce an alluring image of whatever you point it at, color-coded
to show ●the temperature of different objects●.
(Wired という有名な工業技術雑誌より)
https://www.wired.com/story/infrared-cameras-spot-fever-not-slow-covid-19/

(2) ... to monitor ●the temperature of your meats or other foods● as they cook
.
(3) ... you can monitor ★the temperature of different dishes★ all at the same time

https://techcrunch.com/2020/01/06/webers-new-smart-grilling-hub-uses-june-tech-to-make-everyone-a-grillmaster/
0726名無しさん@英語勉強中 (ワッチョイ 4bf6-ntOo)
垢版 |
2020/10/16(金) 16:30:24.33ID:zGL5lIdf0
良い子のみんなは、たくさん英文を読まないと、ただの文法オタクで英語が全然読めない
吾輩は猫先生であーる ◆MLdq8Ig59zES 別名 English geekのようになってしまうぞ。

気を付けよう二ャーーン

(^◇^)  (=^・^=)  (^◇^) (=^・^=)  (^◇^)
0730名無しさん@英語勉強中 (ワッチョイ 9f33-fMdf)
垢版 |
2020/10/17(土) 11:34:43.64ID:sLVT0IE80
死んだ人間に関する記述で、例えば、ニュートンの場合、

「Isaac Newton is a physicist.」ではなく、「Isaac Newton was a physicist.」と書かれるようです。

死んだ人間に対して、例えば、「Gauss is a mathematician.」のように現在形を使うことはないのでしょうか?
0732名無しさん@英語勉強中 (ワッチョイ 0f89-QI1E)
垢版 |
2020/10/18(日) 04:35:45.10ID:hxB2XChP0
>>730
死んだ人間に関しては、"(He, She) was a mathematician, singer, etc." と
必ず was を使うようだね。これを is と言う場合もあるかどうかについては、
俺は知らない。もちろん He is known to have done.... という場合の is は
別の話だけどね。

たとえば死んだ人間について英語ネイティブがうっかり He is [the greatest singer of all time].
などと言ってしまったあとに、慌てて He was... と言いなおしているのを聞いたことがあるような
気もする。Wikipedia の記事で有名人のいろんな略歴を読むことも多いけど、死んだ人については
必ず was になっていて、生きている人については必ず is となっていると俺は感じている。

さらに付け加えると、小説の会話文の中で He was a great guy. とか He was a good father. とか
いう台詞を一人が言ったら、それに対して Was? と相手が尋ね直すときがある。その
Was? というのは、「えっ?Was を使ってるということは、その人はもう死んじゃったの?」という
意味らしい。だからこそ一人目がそのあとに、Yes, he died ten years ago. とか何とか答えている。
0733名無しさん@英語勉強中 (ワッチョイ 0f89-QI1E)
垢版 |
2020/10/18(日) 04:45:39.97ID:hxB2XChP0
死んだ人間について He was a singer. の場合には必ず was だけど、
(He) was the greatest singer of all time.
の場合にだけは、たいていは確かに was を使うけど、次のような
使い方をする人もときどきいる。

(1) Karen Carpenter ●was and is● the greatest female singer of all time.

(2) Michael Jackson ★was and still is★ the greatest entertainer of all time.

(3) Ted Williams ●was and is● the greatest baseball player of all time.

しかしこれは、一つ目の was で、その人が死んだ人だということが明らかだから、
「今でもやはりその人を超える人は出ていないよ」という意味で is とか
still is というふうに現在形を安心して使うことができるらしい。

しかし "was and (still) is" という形ではなく、was なしでいきなり is とだけ
書くのは、死んだ人間についてはおそらくはあり得ないんだろうな、と俺は
今のところ思っている。なお、これについても英語ネイティブに対して確かめたことは
ないので、誰か他の人がこれについてネイティブに尋ねてくれる人がいたら、
もちろんその回答を歓迎する。
0734名無しさん@英語勉強中 (ワッチョイ 4bf6-ntOo)
垢版 |
2020/10/18(日) 09:35:32.90ID:2yViiGMR0
見出し風文法に関する質問です。

https://twitter.com/spectatorindex/status/1317355024612167680
BREAKING: New Zealand's PM Jacinda Ardern set for massive
general election victory

これは、New Zealand's PM Jacinda Ardern set for <is> set for...
のbe動詞が省略されたいわゆる見出し文法だと思います。

https://twitter.com/QuickTake/status/1317397101706444802
BREAKING: Prime Minister Jacinda Ardern swept to a commanding
victory in New Zealand’s general election

これも見出し文法にのっとっているとしたら、Prime Minister Jacinda Ardern
<is> swept to a commanding victory...かと思います。

ざっと調べたところ、sweep to (冠詞のaなし)victoryという定型表現で「圧勝する」という意味
があるのですが、sweep 人 to victoryという表現(人やチームを目的語にする)
はないようです。(Our team swept the series.シリーズ戦に圧勝する
という言い方はありますが)

このツイートは見出し文法にのっとっているのでしょうか。あるいは普通の文法に
のっとって書かれている(sweptは受け身ではなく、単なる過去形)のでしょうか。
また、このツイートでは冠詞のaがついていますが、前述のように定型表現としての
sweep to victoryには冠詞がつかないようですが、aがつくこともあるんでしょうか。

よろしくお願いします。
https://twitter.com/5chan_nel (5ch newer account)
0735名無しさん@英語勉強中 (ワッチョイ 0f89-QI1E)
垢版 |
2020/10/18(日) 12:02:24.60ID:hxB2XChP0
>>734
辞書なんて実際の用例のうちのごく一部しか紹介していないのだから、
俺自身は有名な新聞雑誌や洋書や映画などで英語の使われ方を調べることにしている。
次のような用例は、腐るほど見つかるから、たぶん正しいのだろう。

(1) A broad coalition of left-of-center moderates, conservative evangelicals,
hard-left radicals and traditional Mexican nationalists ●was swept to victory●
with 53 percent of the vote, 32 points above of the runner-up, Ricardo Anaya.
(New York Times, 2018)

(2) Donald Trump ●was swept to victory● by big margins in rural and urban areas.
(Los Angeles Times, 2016)
https://graphics.latimes.com/election-2016-new-hampshire-results/

(3) Tsai ●was swept to victory● by a resilient economy and stock market and protests
against China’s grip in neighboring Hong Kong, which have confronted Taiwanese voters
with the potential perils of closer ties with the mainland.
(Bloomberg, 2020)
https://www.bloomberg.com/news/articles/2020-01-11/tsai-wins-second-term-in-landslide-as-voters-stand-up-to-beijing
0736名無しさん@英語勉強中 (ワッチョイ 9f32-B8Ym)
垢版 |
2020/10/18(日) 18:46:18.40ID:s77ld1+r0
>>734
>sweep 人 to victoryという表現(人やチームを目的語にする) はないようです。

Longmanにまんまドンピシャで出てるが
https://www.ldoceonline.com/dictionary/sweep

9 POLITICS [intransitive, transitive] to win an election easily and in an impressive way
sweep to power/victory

Nixon and Agnew swept to victory with 47 million votes.
Herrera was swept into office on the promise of major reforms.
0737名無しさん@英語勉強中 (ワッチョイ 4bf6-ntOo)
垢版 |
2020/10/19(月) 13:22:11.62ID:Ol2WaHU60
>>735
回答ありがとうございます!なるほど、信頼のおけるメディアの英文なら信用できそう
ですね。受け身としての用例を確認できて納得いたしました。わざわざありがとうございます!

>>736
英英辞典まで確認していませんでしたが、二番目の例文は受け身の用例ですね。
ありがとうございます!
0739名無しさん@英語勉強中 (ワッチョイW efc5-sCmk)
垢版 |
2020/10/20(火) 01:15:59.38ID:K0wqY2jL0
>>723
cobuild にはdistance を複数としてs を付けてる例はなかった。
それにcountable という項目もなかった。だからdistance は
uncountable
a little distance の時は、a little を副詞として考えたら良いのでは?
0741三年英太郎 ◆3CZBjOt3.Y (ワッチョイ 9f86-QI1E)
垢版 |
2020/10/20(火) 02:57:05.03ID:dKc9vfgX0
つーか、ウェブスターの方が有名だしね

〇〇って新語が入った!って毎回話題になってたの
Merriam-Webster's Collegiate Dictionary でしょ

広辞苑と日国の関係に似る
0742名無しさん@英語勉強中 (ワッチョイ ee89-3XbD)
垢版 |
2020/10/21(水) 06:31:07.25ID:uLVG1Qer0
>>740
the world's biggest, arguably the most famous English dictionary

もう少し正確に書くなら、途中にコンマをもう一つ入れて、
the world's biggest, arguably the most famous, English dictionary

さらに(外国人にとってわかりやすく)言い換えると
the world's biggest English dictionary, arguably the most famous one

arguably という言葉の意味は、辞書にあるからわかるよね。

用例(ネットより):
He is arguably the world's greatest living architect.
0743名無しさん@英語勉強中 (ワッチョイ c29d-EtnE)
垢版 |
2020/10/22(木) 13:45:07.91ID:htQtC+7U0
Loving you isn’t the right thing to do

これって文法として正しい?
you の後にisn't ってどういうこと?
0750名無しさん@英語勉強中 (ワッチョイ 7ee3-z4aL)
垢版 |
2020/10/25(日) 21:17:34.32ID:vvL9Nkok0
ちょっと検索してみたところ、たぶん出典は、
『みるみる英語力がアップする音読パッケージトレーニング』という書籍らしい。

https://55english.net/sptr/readingpackage.html
このサイトの情報によると、p.46に出ているとのこと。

http://ash9no.web.fc2.com/miru/miru1-4.html
また、このサイトに出ている文もおそらく同一のものだと思われる。

In this method the learner tries to make use of a language a habit through repetition and correction
このメソッドでは、学習者は、繰り返しと訂正を通じて、ある言語の使用を習慣にしようとします。

これは>>749氏の解釈通りの構造ですね。

>>747は、
前後の文脈を切り離して、the learer を heとし、through repetition and correctionも省略したもので、これだけで意味を取るのは無理だと思います。
0751名無しさん@英語勉強中 (JPW 0Hca-pt/5)
垢版 |
2020/10/26(月) 20:18:04.48ID:0jFFEgPXH
>>749
>>750
ありがとうございます。お察しの通り、みるみる音読パッケージです。
なるほど、VOCなんですね。make use of を一つの動詞としてみてたので、後ろに来る名詞2つがいまいちしっくり来ませんでした。
文章は長かったので、勝手に省略させてもらいました。意味通ってるかなと思ってましたが繋がりが分からなかったですね。申し訳ない。
0754名無しさん@英語勉強中 (ワッチョイ 0233-tMQb)
垢版 |
2020/10/27(火) 18:14:15.82ID:5pp2DxN/0
以下の文が、S + V + OでO = to goであると『表現のための実践ロイヤル英文法』に書いてあります。

I want to go to Italy.

なんとなく「want to」で助動詞のように考えていたので違和感を持ちました。
目的語Oというのは結局のところどういうもののことなのでしょうか?
0755名無しさん@英語勉強中 (ワッチョイ ee89-3XbD)
垢版 |
2020/10/27(火) 18:31:53.69ID:OjJROOLk0
>>754
目的語 --- 「〜を」
to go --- 「行くこと」を
want --- 望む、欲しがる
want to go --- 行くことを望む = 行きたいと考える

だから、want to go も want this book も、.
本質的には同じ構造なんだ。
0757三年英太郎 ◆3CZBjOt3.Y (ワッチョイ 8286-3XbD)
垢版 |
2020/10/27(火) 19:04:41.11ID:A4HYFYXm0
日本語の「極右」「極左」はあんまし聞かなくなってる気がするけど、
英語だと far right/left の方がまだまだ全然多いんよね〜
ポリコレ関係あるのかね?
0759名無しさん@英語勉強中 (ワッチョイ 1333-CXnf)
垢版 |
2020/10/30(金) 08:52:47.38ID:NXyQbAig0
「The set X = {1, 2} × Z_+ in the dictionary order is another example of an ordered set with a smallest element.」

この英文でなぜ「the smallest element」ではなく「a smallest element」となっているのでしょうか?

上の英文を単純な文にすると、例えば、以下のようになるかと思います。

The set X is an example of a set with a smallest element.

集合Xは最小元をもつ集合の例です。
0762名無しさん@英語勉強中 (ワッチョイ 1333-CXnf)
垢版 |
2020/10/30(金) 10:17:09.30ID:NXyQbAig0
以下のような回答が得られました。

回答1
A smallest element is certainly unique, so after establishing its existence you can go on and talk about
"the smallest element". But the sentence "the positive integers form an ordered set with the smallest element"
sounds strange. It makes it sound like there is something out there which you're calling "the smallest element",
and that this set just happens to contain that thing that you're referring to.

回答2
In English, the is a definite article and refers to a specific one (e.g.. that has already been mentioned)

回答3
Another way to think about it is in answer to a question: "Does the set have a smallest element?"
0763名無しさん@英語勉強中 (ワッチョイ 1333-CXnf)
垢版 |
2020/10/30(金) 10:41:34.72ID:NXyQbAig0
確かに、"Does the set have a smallest element?"という文が"Does the set have the smallest element?"と書かれていたらおかしい感じがします。

ところが、例えば、『表現のための実践ロイヤル英文法』をみると、「限定用法の最上級にはtheをつける」となっていて、とにかく機械的にtheをつけると
書いてあるように読めます。文法書の利用の仕方は難しいですね。
0765名無しさん@英語勉強中 (ワッチョイ 1333-CXnf)
垢版 |
2020/10/30(金) 11:00:08.31ID:NXyQbAig0
安藤貞雄著『はじめてわかる英文法』には、「絶対最上級」、「相対最上級」という区別が書かれてあります。
"Does the set have a smallest element?"で使われている最上級は「相対最上級」ではないのだと思います。

文を少し変えて、以下のようにした場合には、おそらく「the smallest element in it」が正しいと推測しますが、どうでしょうか?
"Does the set have a smallest element in it?"
"Does the set have the smallest element in it?"
0766名無しさん@英語勉強中 (ワッチョイ 1333-CXnf)
垢版 |
2020/10/30(金) 11:13:53.14ID:NXyQbAig0
蛇足ですが、杉山忠一著『英文法詳解』には「限定用法の形容詞の最上級は必ずtheをとると思ってよい。」とまで書かれています。
0770名無しさん@英語勉強中 (ワッチョイ 8b89-dNrz)
垢版 |
2020/10/30(金) 13:27:13.19ID:BmxytErj0
>>765
"has a smallest element in it" を検索すると、ずらずらと用例が並ぶ。
その中の一つとして次のものは、ある書物に出てくるそうだ。

Every set of natural numbers that contains at least one element
●has a smallest element in it●.

やはり、smallest element とは言っても、the をつければすでに
話し手や読者にとって特定された element だという意識が働くのだろうと
いう気が俺にはする。

その一方、
ABC has a smallest element (in it).
と言った場合、"ABC has ..." とまで言った段階では、
話し手の頭の中では、その has のあとの目的語は
初めて出てくる物であるから、a にならないといけないと
いうわけなんだろう。もしそれに the がついたら、
それは初出のものではなくなり、すでにその前に言及した
「すでに述べた(あるいは例の、あるいはみんながすでに知っている)
smallest element なのだよ」というニュアンスが醸し出されるのではないかと思う。

それはちょうど、たとえば
Upstrairs, there is a desk.
Upstrairs, there is the house.
という二つの英文の違いと同じだろうと思う。つまり、
a がつけば、「俺にとってもお前にとっても、初めて話題にする desk なんだよ」
というニュアンスであり、the がつくと「すでに承知の例の desk なんだよ」ということになる。
0772名無しさん@英語勉強中 (ワッチョイ 1333-CXnf)
垢版 |
2020/10/30(金) 13:33:14.91ID:NXyQbAig0
>>769
空集合は要素を全く含まない集合なので、全く含まないという意味で0個の要素を含むと言えると思います。
同様に最小の要素が存在しないという意味で0個の最小の要素を含むと言えると思います。

>>770
そうでしたか。
非常に勉強になりました。ありがとうございました。
0774名無しさん@英語勉強中 (ワッチョイ 8b89-dNrz)
垢版 |
2020/10/30(金) 13:54:50.01ID:BmxytErj0
●冠詞の使い方●

(1) On your left is a bathroom.
(2) On your left is the bathroom.

この二つは、両方とも正しい。ただし、文脈によってどちらか
一方だけが正しいということになる。

(1) On your left is a bathroom.

http://www.oocities.org/thomaswar2/goldwalk/goldwalk14.html

上のリンク先には、列車の中を案内している文章が並ぶ。第1車両はこんな感じで、
第二車両にはこれこれこういうものがあり、第三車両はこういうふうになっている、と
解説が続く。そのあとに

On your left is a bathroom. Each car has one like this.

と書いている。つまり、一つの列車に車両が10個つながっているとすると、
それぞれの車両に一つずつ bathroom がついている。だからこそ、
第三車両の話をしているときに、On your left is a bathroom. というふうに
a がついている。つまり、列車には bathroom が一つもないかもしれないし、あるいは
いくつも bathroom があるような列車もあるのだ、と一般人は思っている。だからこそ
列車の中の bathroom には a がつく。一つもないかもしれないし、10個あるかもしれない
bathroom のうちの1つがここにあるから、a bathroom となる。

(続く)
0775名無しさん@英語勉強中 (ワッチョイ 8b89-dNrz)
垢版 |
2020/10/30(金) 13:55:13.99ID:BmxytErj0
(続き)

しかし、
(2) On your left is the bathroom.
https://www.tripadvisor.com/LocationPhotoDirectLink-g1010231-d2479139-i38588211-Sidz_Cottage-Alibaug_Raigad_District_Maharashtra.html

上のリンク先には、一つの部屋の写真がある。その写真には On your left is the bathroom. と書いてある。
話し手も読者もみんな、寄宿舎とか大人数の部屋でない限り、普通は人の住む一つの部屋には一つしかトイレがないと考えている。
だからこそ、ここにある一つの部屋にあるトイレと言えば the bathroom というふうに特定されるのであり、その bathroom
がここでは左にあるのだ、と言っているのだ。
0778名無しさん@英語勉強中 (中止 7bc5-8ci1)
垢版 |
2020/10/31(土) 21:42:54.98ID:KzRCLYZF0HLWN
仲良くなって、夕食食べにいく計画を立てました。
例えば、飯倉片町のキャンティに行ってバジルパスタを食べた後には
どこかのディスコで踊ろう、それが終わったらどこかのバーで飲もう
とかいうことなんでしょうね
0780名無しさん@英語勉強中 (ワッチョイ 8b89-dNrz)
垢版 |
2020/11/01(日) 07:05:01.60ID:7g+UXdmq0
plan が1つかもしれないのに plans と複数になるのはなぜか?英語ネイティブは、
ぼんやりと「私には "何か(たとえば計画とか命令とか) が" がある」と言いたいとき、
特にその計画が単数だと意識していないときで、なんとなくぼんやりと考えている
ときには、まずは複数にしておこうと思っているらしい。そういうことは、たくさんの用例にぶつかっていくうちに
感じられてくる。その例を、思いつくままに挙げてみる。

(1) "Would you like to go to the movies with me this evening?"
と言われて、女が次のように答える。
"I have other plans."
このとき、女は一つの plan しか持っていないかもしれないけど、
とりあえずともかく他のことがしたいのだとか、少なくとも
相手と映画には行きたくないということを言いたいとき、
まずはともかく plans と複数形を使う。

(2) My orders are to search everyone's bag as they come in.
(Cambridge Dictionary のオンライン版の用例より)

上の用例でも、「上司から私は〜するようにと命令を受けている」と
いいたいとき、その命令は一つであるにもかかわらず、my orders と
複数にしている。

(3) My instructions are to (動詞).
こういう例でも、instruction は一つなのに、複数にする例が多い。

(4) イギリスのパブで従業員が客に対して「〜してください」と
指示する。それに対して客が「いや、そんなことはしたくない」と
言うと、従業員は「これは規則ですから」と言い、「規則は規則なんです」
と言う。そのとき、Regulations are regulations. と言っている会話に出会ったことがある。
ここでも今回の規則は単数なのに、ここでは複数を使っている。

まあともかく、こういう用例はいろんな英文に触れているうちにいくらでも出会う。
0781名無しさん@英語勉強中 (ワッチョイ 8b89-dNrz)
垢版 |
2020/11/01(日) 07:09:32.87ID:7g+UXdmq0
次の用例でもやはり、instruction が一つであるにもかかわらず、複数を使っている。

"I beg your pardon, sir," said my friend, the workman-like footman; "but before these gentlemen
say anything for themselves, I wish to explain, as they seem strangers to you, that I only let them in
after I had heard them give the password. ●My instructions are to let anybody in on our side of the door
if they can give the password●.
(The Free Dictionary のサイトより)
0782名無しさん@英語勉強中 (ワッチョイ 8b89-dNrz)
垢版 |
2020/11/01(日) 07:15:58.98ID:7g+UXdmq0
次の用例でも、plans と複数形を使い、そのあとではっきりと、My plans are to (動詞). と言い、
自分の計画を一つしか書いていないのに plans と複数にしている。
このような例を探すには、"My (複数形の名詞) are to (動詞)." の用例を探せばもっとたくさん見つかる。

“If someone pushes,” Ms. Brown said, “I will say, ‘Oh, I’m so sorry, I have ●plans● with my family.’
Because I am my family. And ★my plans are to watch Netflix instead of going to an expensive group dinner★.”
(New York Times より)
0783名無しさん@英語勉強中 (ワッチョイ c1bb-dNrz)
垢版 |
2020/11/01(日) 09:27:36.62ID:Uuv8l36k0
以下のラジオ英会話のダイアローグで、

ラジオ英会話 | おうちで英語学習 | NHKゴガク
https://www2.nhk.or.jp/gogaku/homestudy/kaiwa/137.html

最後に、以下のセリフがあります。

Mirei: But the customer is king, remember!

このtheの意味は何でしょうか?
以下のようにするのは、間違いでしょうか?

But customers are kings, remember!
0784名無しさん@英語勉強中 (ワッチョイ 8b89-dNrz)
垢版 |
2020/11/01(日) 09:56:31.49ID:7g+UXdmq0
"The customer is king." というのは定型句なんだよ。
ことわざみたいなもんだな。Customers are kings. なんて
見聞きしたことがない。

「お客様は神様です」という日本語が定型句になっているけど、
「顧客は神です」なんて普通は言わないだろ?

外国人がいちいち「顧客は神様です」とか
「お客様は一神教の神です」とか、
「お客はブッダです」言ったら間違いか、などと
尋ねてきたら、アホか、超初心者なんだから、まずは
定型句をそのまま覚えろ、と言いたくなるだろ?
0789三年英太郎 ◆3CZBjOt3.Y (ワッチョイ 1386-dNrz)
垢版 |
2020/11/01(日) 22:28:24.54ID:/SRq7xx+0
この the は総称用だから複数になったら落ちる

でも王様は通例(限定されて範囲内に)一人という感覚だろうから
たぶん複数形では言わない

おれも、何かで無難に総称の複数で kings って言ったら
ネイチブにそんな風に言われたことある
0794名無しさん@英語勉強中 (ワッチョイ 8b89-dNrz)
垢版 |
2020/11/02(月) 05:49:55.31ID:9Tc7QtkP0
(1) He is president (または President).
(2) You are Vice President.
(3) He is King.
(4) She is Queen.
(5) Customer is king. (king だけでなく、customer も無冠詞)
(6) The customer is king.
(7) He is captain.
(8) Mother and child are healthy.
(9) Teacher and student are latest arrest at Fivay High School.

これらはすべて正しく、よく使われる。
0795名無しさん@英語勉強中 (ワッチョイ 8b89-dNrz)
垢版 |
2020/11/02(月) 06:01:55.74ID:9Tc7QtkP0
In a time we everyone is only speaking of Customer Experiences,
our idea is that ●Customers are Kings● and we will offer Champagne
on weekends, made to measure in suit measurement and home delivery.
(Forbes という有名な雑誌より)

このように、Customers are kings. というのも使われることは使われるし、
第一これは "(複数名詞) are (複数名詞)" というありきたりの定型的な構文だよね。

ここで customers には定冠詞がないことに注意。もしも the customers と
定冠詞がついていたら、その場所にいる(あるいは話し手の念頭にある)
特定の customers という意味。しかし上の用例では定冠詞がないから、
一般的な customers のこと。
0798名無しさん@英語勉強中 (ワッチョイ b132-GqKp)
垢版 |
2020/11/05(木) 06:34:51.36ID:LcSjHaEP0
Wherever you are, I 'll be with you.

これはなんで後が I am with you や  I would be with you
でも同じような意味になりますか?

あなたがどこへいこうとも、私は忘れないだと
後はI Will not forget you になりますか??
0799名無しさん@英語勉強中 (ワッチョイ b132-GqKp)
垢版 |
2020/11/05(木) 06:37:41.92ID:LcSjHaEP0
>>798

あなたがどこへいこうとも私はあなたの側にいる。

これはなんで後が I am with you や  I would be with you
ではダメなんですか?これでも同じような意味になりますか?
ですすいませんm(__)m
0801名無しさん@英語勉強中 (ワッチョイ 61e2-CJ6T)
垢版 |
2020/11/05(木) 13:57:10.80ID:oCTZXX4g0
マイケル・ダグラスの『Falling Down』のやつは「The customer is always right.」だったか。
0802名無しさん@英語勉強中 (ワッチョイ 61e2-CJ6T)
垢版 |
2020/11/05(木) 14:11:37.29ID:oCTZXX4g0
Richard Marx: “Wherever you go, Whatever you do, I will be right here waiting for you...."
willで意志が伝わる。

「I am with you.」は断言するもっと強い感じ。それもありだけど。
「父さんは何があってもお前の味方だよ」的な。
0803名無しさん@英語勉強中 (ワッチョイ 92f0-j5Tt)
垢版 |
2020/11/05(木) 18:49:31.29ID:M6x0aq/30
マライアの歌にも似たようなのあったな

anytime you need a friend, i will be here
0805名無しさん@英語勉強中 (ワッチョイ b1bb-L1wp)
垢版 |
2020/11/06(金) 17:40:53.03ID:vJEcPo8i0
完全に私の感覚だけど、、、

I am with you.
客観的事実を述べているように感じる。

I 'll be with you.
必ずするという強い意志を感じる。

I would be with you.
そうなるだろうという推測感を感じる。

合っているかどうかは分からない。
0808名無しさん@英語勉強中 (ワッチョイ 61e2-CJ6T)
垢版 |
2020/11/06(金) 20:58:24.91ID:G7obEZ470
>>807
M(前置詞の副詞句):Beneath each sentence (それぞれの文の下に)
S:you
V:will see
O:four words or phrases,
M(過去分詞の形容詞的用法):marked (A), (B), (C), and (D).


  (1) I (    ) apples.

    A. doesn't like   B. likes   C. like   D. am


みたいな感じでは?
0809名無しさん@英語勉強中 (ワッチョイ a932-D/bg)
垢版 |
2020/11/07(土) 10:04:16.14ID:CYsW5XDz0
>>800

有難うございます!
そうなんですね!
0810名無しさん@英語勉強中 (ワッチョイ a932-D/bg)
垢版 |
2020/11/07(土) 10:06:46.36ID:CYsW5XDz0
>>802

有難うございます。
WILL 意志。願望含むかな?

BE動詞だとより強い感じなのですね!
文章付きで理解しやすかったです。
0811名無しさん@英語勉強中 (ワッチョイ a932-D/bg)
垢版 |
2020/11/07(土) 10:07:51.62ID:CYsW5XDz0
>>803

有難うございます。
やっぱりWill
そうしたいの意志なんですね。
0812名無しさん@英語勉強中 (ワッチョイ a932-D/bg)
垢版 |
2020/11/07(土) 10:11:30.94ID:CYsW5XDz0
>>804

有難うございます。
はい。will stand by
またはそのまま入れちゃうか
思い入れによってですね。
0813名無しさん@英語勉強中 (ワッチョイ a932-D/bg)
垢版 |
2020/11/07(土) 10:15:53.34ID:CYsW5XDz0
>>805

would beだとちょっと
距離がある感じなのかな?
有難うございますm(__)m
0815三年英太郎 ◆3CZBjOt3.Y (ワッチョイ 9286-L1wp)
垢版 |
2020/11/07(土) 13:08:39.63ID:xAYvQ14a0
強い意思の will は、多くの場合、強勢をともない、
文字の場合には短縮形 'll で書くことはないです

あんましここのレスを鵜呑みにしないで、
辞書なり文法書なりを確認しましょう
0816名無しさん@英語勉強中 (ワッチョイ a932-D/bg)
垢版 |
2020/11/10(火) 05:39:58.32ID:imYigm8b0
>>815

わかりました。
有難うございます。
0818名無しさん@英語勉強中 (ワッチョイ 2332-LfyP)
垢版 |
2020/11/11(水) 23:13:59.81ID:46EZxqQi0
not only 〜 but (also)で辞書引け
超有名な熟語
0820名無しさん@英語勉強中 (ワッチョイ a3f0-JOl5)
垢版 |
2020/11/12(木) 11:54:14.49ID:cuWX+cJg0
Situated downtown, the hotel is convenient.
中心街に位置していて、そのホテルは便利だ。
downtown は名詞ではなく副詞ですか?
叙述用法の形容詞のsituated の後ろには前置詞句が来ると思ったのですが、なんで直接downtownが置けるのですか?
0822名無しさん@英語勉強中 (ワッチョイW 23b0-sA6t)
垢版 |
2020/11/12(木) 13:26:49.78ID:c2W05mLl0
別スレで聞いたこともあるのですが、文法について説明してもらえなかったので、
こちらで分かる方いたら教えて下さい。


The risk of infection increas the closer you are to another person with the virus and the amount of time you spend in close contact.

この文章の文法について教えて下さい。

@the closer が他にthe 比較級が出てきていないのに「〜する程」と訳して良いのか
Aand以下が名詞になっているが、どの部分とつなげられているか
(それとも名詞and 名詞という形にはなっていないのか)
という2点について特に分からないので教えていただければ助かります。
0823三年英太郎 ◆3CZBjOt3.Y (ワッチョイ 2386-Y0tO)
垢版 |
2020/11/12(木) 18:00:54.39ID:dVbgoA4G0
> The risk of infection increases the closer you are to another person with the virus

the は関係副詞(〜するほど)として機能してるので、
必ずしも呼応する指示副詞 the がなくても問題ないと思われる。

後半は不足してる語句を頭で補えば、まあ読めるじゃん?
0826名無しさん@英語勉強中 (ワッチョイW 2be3-Scn0)
垢版 |
2020/11/13(金) 08:12:58.39ID:Q6RMMqzp0
後半補うとすれば、
[the more] the amount of time you spend in close contact [is]
あるいは、
[directly with] the amount of time you spend in close contact
などでしょうか。
このように補わなければ意味が通じない以上、この文は文法的には誤りであると思います。
0828名無しさん@英語勉強中 (ワッチョイ 4b89-Y0tO)
垢版 |
2020/11/13(金) 14:09:21.37ID:AYzVOv6q0
>>827
そう、それは正しい。俺も最初はそれに違和感を感じたんだけど、
いろんな経済関係の記事を読んでいるうちに、慣れてしまった。一般的に

the number of (形容詞または動詞の過去分詞)

という形が出てきたら、それは

the number of (形容詞または動詞の過去分詞) + 名詞の複数形

という意味だということがわかってくる。検索してみたら、いくらでもそういう
例文が出てくるよ。なお、こういう例文は確か

Michael Swan, Practical English Usage

にも出てきた。
0829名無しさん@英語勉強中 (ワッチョイ 4b89-Y0tO)
垢版 |
2020/11/13(金) 14:33:11.01ID:AYzVOv6q0
>>827
以下は、PEU からの引用。

7. amount and number
"The" is dropped after "the amount/number of."

(1) I was surprised at ★the amount of money collected★. (NOT ... "of the money")

(2) ●The number of unemployed● is rising steadily.

(PEU 142.7)
0830名無しさん@英語勉強中 (ワッチョイ a532-D0pj)
垢版 |
2020/11/13(金) 15:05:21.46ID:pQIF6C/e0
Eating vegetables is good for health.

なんでare じゃないのでしょうか?フルーツもおなじですか?
0834名無しさん@英語勉強中 (ワントンキン MMa3-XkIP)
垢版 |
2020/11/13(金) 17:19:16.73ID:Nr8FR2c0M
I did was make soup today.って
all l did was to make soup todayってことなんやな
書き込む前に調べてたらぼんやり解決した
それにしてもi didだけでもwasってきたら名詞節なんだなってわかるもんなんやな
0835名無しさん@英語勉強中 (ワッチョイ a59d-D0pj)
垢版 |
2020/11/14(土) 00:05:44.37ID:9T5OqV7X0
>>834
さすがにそれは破格、文法的とは言い難い。えてして通じちゃうけど。
0836名無しさん@英語勉強中 (スップ Sd03-9PGX)
垢版 |
2020/11/14(土) 18:23:25.44ID:mMowY3hxd
英検準1のパス単に
This committee will convene every Friday evening at 7 p.m. in the town hall.
とあったんですが、これ普通現在形で書くんじゃないんですか?willを使う理由ってなんなんでしょう
0839名無しさん@英語勉強中 (ワッチョイ 4b89-Y0tO)
垢版 |
2020/11/14(土) 19:42:44.94ID:ouUPkzzs0
>>836
現在形だったら、すでに毎週会合を開いてきて、これからもずっと定期的に開く、という意味。
未来形だったら、まだ会合を開き始めて間がない(たとえば1週間とか2週間くらいしか経っていない)
けど、これからはずっと毎週この日のこの時間に開くよ、という意味になるだろうな。
だから結局は、他の回答者が言ったように、「いま決めたばかり」ということになるけど。
0841名無しさん@英語勉強中 (ワッチョイ a532-D0pj)
垢版 |
2020/11/15(日) 07:30:35.55ID:RvYwIlG60
>>831

有難うございます。
中学英語やってます。
0842名無しさん@英語勉強中 (ワッチョイ 23f0-g/iO)
垢版 |
2020/11/15(日) 11:51:23.41ID:FqyVCfM20
>>839
質問者じゃないけど、なるほどね
こういう簡単なことに
外国語だと気づきにくいよね
0845名無しさん@英語勉強中 (ワッチョイ 9bc5-gFAA)
垢版 |
2020/11/15(日) 22:05:29.94ID:62x5MmhJ0
every weekendとかつくと、openでも動作動詞じゃなくて状態動詞
扱いになるんじゃないの?進行系って動作動詞を状態動詞にする操作でしょ?
動作が定期的に行われるという継続の副詞のevery weekendをつかうことで
毎週末、「ひらいている」、という状態をあらわすことができるんだろうね。
状態だから今あるわけで、未来に生じるわけじゃないから未来を使わない。

でも、
You might know tomorrow whether meetings will be held every morning.
というのってありなんじゃないの?
0846名無しさん@英語勉強中 (ワッチョイ 4b89-Y0tO)
垢版 |
2020/11/16(月) 07:03:49.39ID:HrmQwT9E0
One change that Bellin has made is the creation of an incident command structure.
This is comprised of physician and administrative leaders across the system.
These individuals ●will meet every morning, seven days a week● to discuss the current measures
being taken and any new information available about the current COVID-19 situation in Wisconsin and across the country.

(American Medical Association の記事。文脈を読めばわかるように、こういう組織ができたばかりで、これから
先に毎朝会合を開くと言っている。)
https://www.ama-assn.org/delivering-care/public-health/covid-19-communication-refining-team-based-care-time-crisis

The cardinals ●will meet every morning● to discuss where the church is headed and, over lunches and dinners,
take the measure of one another’s characters, talents and experiences, based on personal relationships and observations.
(The New York Times)
0853名無しさん@英語勉強中 (ワッチョイ f792-BCf+)
垢版 |
2020/11/27(金) 23:40:47.50ID:isE+vYmz0
1: He ran fast.

2: He looks young.
は同じだと思うのに、1のfastは副詞、2のyoungは形容詞という。
なぜ1のfastが形容詞だとだめなんだろう?

He is happy that his daughter passed the examination.
のthat 以下は副詞節ですよね。
that 節だけで副詞節なんかありえるんか、
おもったら、I'm glad that you've got well.
とか
I'm sure that he is coming.
というのもある。こういうのあります、とだけ
説明されてもこまるわな。なんか理屈がないと。
0854名無しさん@英語勉強中 (ワッチョイW d796-2JXS)
垢版 |
2020/11/28(土) 00:30:56.42ID:qfKlTFIQ0
>>853
端的に言えば、あなたの考えている英語における形容詞と副詞の定義が曖昧であることに起因する問題であるように感じられます。疑問が解消されないのであれば形容詞と副詞についてそれぞれ定義したうえで疑問を投げかけてください。
なお、品詞については位置と形でほぼ決まります。
質問趣旨がよくわかりませんが、fastには形容詞も副詞もあります。
He is a fast runner.
He runs (really) fast.
前者は名詞を修飾する形容詞、後者は動詞を修飾する副詞です。
0855名無しさん@英語勉強中 (ワッチョイ 175a-uBav)
垢版 |
2020/11/28(土) 00:31:18.25ID:mZ6UpzI/0
>>853
>1: He ran fast.
>2: He looks young.

1はSVMで、2はSVCで文構造が違う。
修飾語と補語の違いを確認するとよい。

>He is happy
>I'm sure that

形容詞の後のthat節は、その形容詞の感情を引き起こす原因を表すものと、
その感情の対象を表しているものの、2種類がある。

いずれも『ロイヤル英文法』を入手して読んで勉強すると良い。
0856名無しさん@英語勉強中 (ワッチョイ 979d-FP8+)
垢版 |
2020/11/28(土) 00:36:06.35ID:QR/kbfnO0
『ロイヤル英文法』おじさん
0857名無しさん@英語勉強中 (ワッチョイ ff89-P4WK)
垢版 |
2020/11/28(土) 07:38:00.13ID:mBQDVzaG0
>>853
(1) He is happy that his daughter passed the examination.
(2) I'm glad that you've got well.
(3) I'm sure that he is coming.

俺は文法に詳しいわけじゃないから適当に聞き流してくれてもいいが、
上の3つの that は because みたいなもの(ただし、完全に that と because が等しいというわけではない)
だと思っておけばいいんじゃないか?ただ、(3) だけは because という意味よりもむしろ、
"he is coming" ということに関して私は sure である、というような意味のように見えるけど。

400年前の Shakespeare の時代には、"we cry that" が "we cry because" みたいな意味として使われていた。

(1) "King Lear" (リア王) 原文
When we are born, we cry ●that● we are come
To this great stage of fools.

(2) 上の英文の現代英語訳
When we’re born, we cry ★because★ we’ve arrived on the stage of life, like all the other fools.
https://www.sparknotes.com/nofear/shakespeare/lear/page_246/
0858名無しさん@英語勉強中 (ワッチョイ ff89-P4WK)
垢版 |
2020/11/28(土) 07:39:44.18ID:mBQDVzaG0
>>853
>>こういうのあります、とだけ説明されてもこまるわな。なんか理屈がないと。

いや、そういう理屈が欲しい人は、自分で考えないとダメなんだよ。
もしもそういう理屈をすべて解説したら、500ページの「総合英語」とか
800ページの「フォレスト」を1万ページにしないといけなくなる。
0859名無しさん@英語勉強中 (ワッチョイW 1f60-1aTK)
垢版 |
2020/11/28(土) 13:48:16.48ID:SPqUQEfy0
>>853
形容詞や動詞の意味を補足する機能を「副詞なんとか」と呼んで、
名詞の意味を補足する機能を「形容詞なんとか」と呼ぶと
決めてるからで、
形そのもので「形容詞なんとか」「副詞なんとか」と
あらかじめ決まっているわけではないから
> that 節だけで
と考えること自体がおかしい
0860名無しさん@英語勉強中 (ワッチョイ 7729-3gg3)
垢版 |
2020/11/28(土) 16:54:04.40ID:hjs5K22j0
同格のthatで、
for/from the idea/fact that, because of it that みたいな、
判断・感情の根拠・原因、対象に関する名詞節を導くものでthatの前が略された感じ。
0861名無しさん@英語勉強中 (ワッチョイ ff89-P4WK)
垢版 |
2020/11/28(土) 17:20:22.95ID:mBQDVzaG0
OED という辞書の中の that の説明をさらに読んでいると、次のようなものもある。

that (conjunction)
(a) Introducing a clause expressing the cause, ground, or reason of what is stated in the principal clause.

上の定義文では、that という接続詞を、理由や原因や根拠を示すために使っていると書いている。
一言でいうと、ここでの that はしいて言えば because みたいなものだということになる。

1611 Bible (King James) Isa. lxiii. 5
   ●I wondered that● there was none to vphold. View more context for this quotation
   that 以下の理由により驚く、という意味。

1810 G. Crabbe Borough xviii. 244
   Men.●.bless their God that● Time has fenc'd their Heart.
   that 以下のことについて(そのゆえに)神を祝福する、という意味。

1842 Macaulay in Life & Lett. (1876) II. 114
   I should be very ●sorry that● it were known.
   that 以下のことのゆえに残念に思う、という意味。

1859 ‘G. Eliot’ Adam Bede II. iv. xxxv. 372
   Mrs. Poyser was quite agreeably ●surprised that● Hetty wished to go and see Dinah.
   that 以下のことがらのゆえに驚く、という意味。
0864名無しさん@英語勉強中 (ワッチョイ 7fe3-C0hB)
垢版 |
2020/11/28(土) 21:36:29.60ID:TcohGcki0
be sure that 〜については、このthat節は名詞節とも考えられるというような記述をどこかで読んだ記憶があった。
ちょっと確認してみたところ、ロイヤル英文法に以下のような記述があった。

***
I am sure that he is honest.
* that節は前に of the fact が省略されたと考えると名詞節。
(p.38)
***

同p.285には以下のような記述がある。

***
(2) 他動詞の目的語に相当するthat節
<be+形容詞+前置詞>で他動詞と同じような働きをすると考える。
that節の前では前置詞は脱落する。
I am afraid that I may hurt his feelings.
= I am afraid of hurting his feelings.
***
0865名無しさん@英語勉強中 (ワッチョイ 7fe3-C0hB)
垢版 |
2020/11/28(土) 22:13:24.31ID:TcohGcki0
>>863の「このthatは省略も可能なはず」は、混乱をまねきかねない情報なので削除したい。
諸賢にあらせられてはどうかこの一行を無視してくださるように。

実を言うと、
I'm glad that you've got well.
のような副詞節の場合はthatは省略できないと勘違いしていた。(このthatも省略できる)
つまり省略できることが名詞節だと考えられる理由であると言いたかったのだが、これは間違い。
0866名無しさん@英語勉強中 (ワッチョイ f792-BCf+)
垢版 |
2020/11/28(土) 22:50:22.40ID:BZ9JyO+Y0
たくさんの方々、ご回答ありがとうございます。
誰もが悩んでいるんだな、ということや、悩みながらも
辞書を引いたりして調べてbecauseの意味であることを
発見しなければいけないものなのだなと自分自身
反省をしました。
I run fastはS V Cです、ともいえると思います。
そうするとfastは形容詞になる。でもそうしないのは
過去の気まぐれでそうなってしまっているからという
ことなのかもしれません。とりあえず副詞です、と
いうことは間違いありません。だから第一文型という
ことになります。
0870名無しさん@英語勉強中 (ワッチョイ 175a-uBav)
垢版 |
2020/11/29(日) 01:30:57.72ID:l1xqAZu/0
辞書など見なくても、
どう見ても「He runs.」は文が完成してるのに対し
「He seems.」は文が成立していない。
0872名無しさん@英語勉強中 (ニククエW f792-9eSs)
垢版 |
2020/11/29(日) 12:06:03.00ID:8i8cYTif0NIKU
>>846
ありがとう。every morning があると動詞が状態動詞になる、ということはないということが
わかりました。ところでどうやって例文を検索してるんですか?
なにかの本を読んだのですか?できれば教えてもらえるとうれしいです
0873名無しさん@英語勉強中 (ニククエW f792-9eSs)
垢版 |
2020/11/29(日) 12:17:43.72ID:8i8cYTif0NIKU
>>871
英語がめちゃ得意なff89さん、最近よくお世話になってます。
今回もどうもありがとうございます。
その例は、svcなんでしょうか。svでourselves は
強調と考えるのでしょうか。
多分、he runsとhe isは違うという870さんに対する反論なのでしょうね。

he isで完結してるとも言える、と思います。それだけじゃ意味が不十分
なのでhe is tidyとかするだけで。
he runs fastだって似たようなものだと思う。fastがないと意味不明の設定なんて考えられるんだし
0874名無しさん@英語勉強中 (ニククエW f792-9eSs)
垢版 |
2020/11/29(日) 12:21:40.82ID:8i8cYTif0NIKU
それでも形容詞と副詞と分けるんですよね。
辞書に書いてあるからというのもその通りだし、
he runsは意味が取れるけど、he seems は意味が取れない
という説明ももっともだ。
たしかにその通りとも言えるわけだから。
0875名無しさん@英語勉強中 (ニククエ ff89-P4WK)
垢版 |
2020/11/29(日) 12:32:26.27ID:nFsmttCu0NIKU
>>872
>>These individuals ●will meet every morning, seven days a week●

例えば子の用例をどのように検索したかって?俺は裏技なんて駆使できないし、
なんせパソコン音痴なんで、最小限度のことしかできないよ。
上の用例を検索するときにはどのようなキーワードを使ったかというと、

"will * every morning"
というキーワードを Google 検索の白い箱の中にぶち込んだだけだよ。
* の部分にいろんな動詞のついたたくさんの用例が並ぶから、
それを片っ端から読んで、俺にとってもみんなにとってもわかりやすそうな
用例を選んだだけだ。

もしも "will * every morning" でよい用例が見つからなかったら、
"will * every evening" にしたり、いろいろと変えてみるんだ。
"will go * every morning" というふうに、動詞 go を入れたり
wake up を入れたりしてみたら、また別の用例が見つかってくるから、
自分にとって好みのものが見つかりやすくなる。
0876名無しさん@英語勉強中 (ニククエ ff89-P4WK)
垢版 |
2020/11/29(日) 12:42:22.54ID:nFsmttCu0NIKU
>>873
>>★Australia will always be ourselves★

この英文を紹介したのは、今回いろんな人のコメントとはまるで関係なく、
上の用例が、俺がもともと正しいと思っていた英文のどれとも似ていなかったから、
面白いと思ったからだ。普通なら(というか、俺たち外国人学習者から見れば)

(1) Australia will always be itself.
(2) We will always be ourselves.
(3) We Australians will always be ourselves.
(4) Australians will always be themselves.

とかいうふうに書いたりしゃべったりするべきであって、

(5) Australia will always be ourselves.

というのは変だろうと思い込んで、こんな英語は絶対にしゃべったり
書いたりしてはならないって思ってしまいがちだろう?でもオーストラリアの
首相がテレビで堂々と (5) を口にしているから、「ああ、これも正しいんだな。
たぶん書き言葉では許されないけど、くだけた口語では許される形なのかもしれんな」
と俺は思っている。ちなみに、

America will be ourselves.
Britain will be ourselves.
Ireland will be ourselves.

などという単語列を検索してみたら、用例が一つも見つからない。
やっぱり (5) のような用例は、おそらくは「少し砕けすぎた言い回し」であって、
書き言葉や、少し注意してしゃべったときの英語としてはあまりよくない
英語であって、そんなに頻繁に出てくることはないのではないかと思う。
しかし、「くだけた」英語としてはおそらく頻繁に出てくるのだろう。
そんな気がしている。
0877名無しさん@英語勉強中 (ニククエ 175a-uBav)
垢版 |
2020/11/29(日) 13:22:31.44ID:l1xqAZu/0NIKU
>>873
「He is.」で完結するわけがない。
「彼はです」でいったい何が「完結」しているのか。

beやseemのような連結動詞(不完全自動詞)は、
主語の名詞と補語の形容詞/名詞を「連結」するのがその役割。

linking verb: a verb that connects the subject of a sentence
with its complement, for example ‘seem’ in the sentence
‘The house seems big’ SYN copula

日本語であれば「〜は⋯です」の助詞や助動詞に相当する「機能語」であり、
その「機能」を否定したらその存在自体を否定するのも同じこと。

なお「God is.(神は存在する)」のようなbe動詞は完全自動詞であり
実質的な内容を表す「内容語」であり別物。
0879名無しさん@英語勉強中 (ニククエ 175a-uBav)
垢版 |
2020/11/29(日) 14:35:05.58ID:l1xqAZu/0NIKU
文「He is tidy.」の主語はHe、述語(述部)は「is tidy」になる。

自動詞runと異なり、この連結動詞のisは単体では述語を形成しえない。
形容詞tidyを伴って初めて述語が成立する。

形容詞tidyは、主語Heの性質を直接「叙述」する、述語を構成している中心要素であり、
欠落したら述語は成立しなくなる。述語が成立しなければ当然文も成立しない。

「He is a tidy man.」において名詞manを「修飾」する形容詞tidyのような、
あってもなくても文が成立する単なる追加情報である「修飾語」の形容詞と、
述部を形成する「補語」になっている形容詞とでは、本質的に異なる。
0881名無しさん@英語勉強中 (ニククエ 175a-uBav)
垢版 |
2020/11/29(日) 14:42:41.16ID:l1xqAZu/0NIKU
「〜です」を意味する連結動詞のbeと
「存在する」を意味する自動詞のbeは別物であり、
後者が拡張されて前者になっているわけではない。
0882名無しさん@英語勉強中 (ニククエ 175a-uBav)
垢版 |
2020/11/29(日) 14:45:06.36ID:l1xqAZu/0NIKU
be動詞の後に来る「補語」は、
あくまで主語を直接叙述しているものであって、
be動詞を「補足」しているわけではない。
0884名無しさん@英語勉強中 (ニククエ 175a-uBav)
垢版 |
2020/11/29(日) 14:57:34.37ID:l1xqAZu/0NIKU
>>883
連結動詞というのはそういう文法的機能を持つ機能語であり「判定」もクソもない。
足し算の「+」の意味を「判定」するくらいおかしな話。
0888名無しさん@英語勉強中 (ニククエ 175a-uBav)
垢版 |
2020/11/29(日) 15:39:04.44ID:l1xqAZu/0NIKU
>>885
違うものだから、違うんだよ。be動詞に@連結とA存在の意味・用法が2つある。

4 [linking verb] used to DESCRIBE someone or something, or say what group or type they belong to
・The sky was grey.
・Spiders are not really insects.

12 [intransitive] formal to EXIST
・What was once a great and powerful empire has effectively ceased to be.

意味的にも、用例的にも、明らかに違う。連結動詞のbe動詞を自動詞existに置き換えたら明らかにおかしくなる。

これを、一体何を根拠に「同じ」だと思えるんだ?
「違う」から「違う」ものとして定義が辞書に掲載されているし、
実際に誰が読んでも使っても「違う」のだから疑問を挟む余地もない。

それを屁理屈捏ねて「同じ」だと強弁する方が基準でも根拠でも提示するのが筋というもの。
0890名無しさん@英語勉強中 (ニククエ 175a-uBav)
垢版 |
2020/11/29(日) 16:11:39.91ID:l1xqAZu/0NIKU
>>889
君は基本的な文法用語を満足に理解していないから
自分で自分が何を理解していないのかを理解していないだけ。
「補語」も「修飾」も「叙述」も「形容詞」も「副詞」も十分に把握できていない。
謎の電波自論を展開する前に教科書をちゃんと読むことから始めたほうがいい。
0894三年英太郎 ◆3CZBjOt3.Y (ニククエ 9f86-P4WK)
垢版 |
2020/11/29(日) 18:45:55.96ID:w5eDhP1y0NIKU
become も appear も連結動詞やで

連結動詞だから機能語、ってのはムリあるなぁ

意味の弱さとか、弱形があるとか、判定基準あるやろなぁ

God's in his Heaven.
God is.
*God's.
0895名無しさん@英語勉強中 (ニククエ 9f32-0Obg)
垢版 |
2020/11/29(日) 19:07:16.40ID:mxVKWvDK0NIKU
be動詞も存在の意味あるし
というかこれは英語史的には当たり前の話で
もともとbeは二つの意味「である」系統と「存在する」系統が一緒になったもの
活用がぐちゃぐちゃなのもこれら二系統の活用をそれぞれ引きずってるから
0897名無しさん@英語勉強中 (ニククエW 1f60-1aTK)
垢版 |
2020/11/29(日) 19:12:50.83ID:h/kCe6qc0NIKU
> 1: He ran fast.

2: He looks young.
は同じだと思うのに、1のfastは副詞、2のyoungは形容詞という。
なぜ1のfastが形容詞だとだめなんだろう?

そもそもこの最初の質問者ってまだいるの?
0899名無しさん@英語勉強中 (ニククエ 175a-uBav)
垢版 |
2020/11/29(日) 20:15:45.55ID:l1xqAZu/0NIKU
CaGELではbe動詞はSVCの場合も含めて助動詞。

It follows from our syntactic definition that
be is an auxiliary verb not only in examples like
She is working or He was killed
but also in its copula use, as in They are cheap
(cf. They are not cheap and Are they cheap?). (p. 51)
0902名無しさん@英語勉強中 (ワッチョイW f792-9eSs)
垢版 |
2020/11/30(月) 06:09:00.98ID:zYTvv9nF0
>>886
を見て考え込んでいた。
1機能だけが異なるとする英文法
2意味だけが異なり、機能は同じであるか異なるかを問わない英文法
3機能だけが異なり、意味は同じであるか異なるかを問わない英文法
4意味と機能のどちらも同じか、もしくは意味と機能のどちらも異なる、英文法
意味が異なるか、もしくは機能が異なるかのいずれかの英文法
も組み合わせとしてはあるけど
1はありえない、と思うし、
234もありえないかな、、、確かじゃないけど。説明できないけど。
ということは>>886はすべてのケースを言い尽くしてるのか。
よくこんなこと考えるなと思った。
0903名無しさん@英語勉強中 (ワッチョイW f792-9eSs)
垢版 |
2020/11/30(月) 07:07:32.14ID:zYTvv9nF0
883は自然だが、いい指摘で
885と887はそれを受けての自然だが、いいつっこみで、
さらに適切な皮肉も加わっている。
889は888さんの判断の基準をうまく引き出して
それを並べることができている。
つまり、he runs fastのfastとかrunについて、i am stout のstoutとかrun の
扱いと別にしないといけないという根拠として、辞書にそう書いてあるからという
基準が888さんの判断基準だということだった。

僕は辞書と意味と書き換えという、889さんのいう意味がわからない。

891のexist交替可能基準だが、he runs fastもi am stout のいずれも
existで置き換えてもいいと思うし、置き換えてもだめとも言えそう。
だから意味を持たない

894の三年さんからは he runs fast のrun が機能語ではない連結動詞である可能性が
指摘されたと、僕はとらえた。

899さんは文法書の引用があった
0904名無しさん@英語勉強中 (ワッチョイW f792-9eSs)
垢版 |
2020/11/30(月) 07:16:41.94ID:zYTvv9nF0
883さん885さんはどんな人なんだろう?
886さんは短答とか数的処理とかそんなことを勉強した人か数学が好きな人
899さんは英語が専門の人あるいは高校の先生かもしれない
891さんは英語が好きで勉強してる人ですよね。
僕は途中から頭が回らなかったから鈍い人。

he runs fast について、誰もが疑問を抱いている、英文法について本を読むことが
あっても疑問が解決できずにいる、それは勉強が足りないからということに
帰すことで解決するものではなく、he runs fast のfast を副詞だと書くに至った
文法そのものに曖昧さがある、あるいは副詞と分類することが一つの
思い切った選択に過ぎない、そんな分類を組み込んだ、考えることのできる
文法の中のたった一つの可能性にしか過ぎない便宜上のルールとして、辞書に体化されている
英文法ができているという人間の所作としての文法のあり様を示しているのかもしれません。
ともかく、楽しい議論をありがとうございました。


僕の書いた文章日本語になってないことについてはご容赦を。
0906名無しさん@英語勉強中 (ワッチョイW f792-9eSs)
垢版 |
2020/11/30(月) 07:47:18.05ID:zYTvv9nF0
878を見て、この議論についてもう一度考えようと思ったのでした。
892を見てわかるようにこの方は頭が柔らかい。
exist 交替可能基準を提示した方でもありますが、
僕はそれをまとめにおいて、充分考慮していません。
ごめんなさい。
0908三年英太郎 ◆3CZBjOt3.Y (ワッチョイ 9f86-P4WK)
垢版 |
2020/11/30(月) 13:00:52.26ID:uVytj9sd0
>>895
> もともとbeは二つの意味「である」系統と「存在する」系統が一緒になったもの
> 活用がぐちゃぐちゃなのもこれら二系統の活用をそれぞれ引きずってるから

これ眉唾ね

be の活用が形態論的に複雑なのは、確かに
3〜4くらいの動詞がごっちゃになったからだと説明されるけど、
それぞれの元の意味をみてくと、存在とコピュラの二系統には
分けられないわ

それに、そういう語史を持たないいくつかの印欧語でも、
beに当たる動詞は、存在とコピュラの両方の意・役割を
兼ね備えてるじゃないの
0909名無しさん@英語勉強中 (ワッチョイ 7729-3gg3)
垢版 |
2020/11/30(月) 15:01:34.68ID:QilLMqPo0
文法解説は、半可通には手に余るってことね。
質問者の要求が高校英文法+αの枠内で済むならまだしも、
単語の歴史、文法の歴史まで及ぶことが多いので。
0910名無しさん@英語勉強中 (ワッチョイW f792-9eSs)
垢版 |
2020/11/30(月) 15:24:29.36ID:zYTvv9nF0
>>907
he runs fastを
svc と考えてもいいということだと解釈しましたが、、

i am stout.
とか
he runs fast.
のいずれも
i amとかhe runs.
で完結していると言おうとすれば言える。
その上で、fastとかstout が説明のためについている。つまり同じ構造だ
というのが元々の主張です。
実際、i amとかhe runs だけだと、なんのことかと思いますよね。
これを持って完結していないという主張が出てきます。
どちらも完結してないから補足語をつけよう、あるいはどちらも完結しているけど
意味が十分ではないので補足語をつけようということなのだろうと思います。
0911名無しさん@英語勉強中 (ワッチョイ 57e2-cHic)
垢版 |
2020/11/30(月) 15:53:25.35ID:R9W5ULGS0
>>910
>i amとかhe runs.
>で完結していると言おうとすれば言える。
>その上で、fastとかstout が説明のためについている。つまり同じ構造だ
>というのが元々の主張です。

それが仮に「完結」していたとして、fastやstoutは一体何になるわけ?

@「説明のためについている」って、一体何を「説明」してるわけ?
Aその「説明」は一体どんな類の「説明」なの? 修飾なの? 補語なの?
0912名無しさん@英語勉強中 (ワッチョイW f792-9eSs)
垢版 |
2020/11/30(月) 16:23:25.31ID:zYTvv9nF0
説明というのは例えば
おばあさん、昨日の夜 8時にどこにいたんですか。
と問われたおばあさんが
歩いてました。パチンコパーラーエコーの前の路上を。
と答えたとします。
歩いていました、で完結しているのですが、
しかし不十分なのです。茶川警部補の
問いは場所についての問いなのですから。
補足情報の、パチンコパーラーエコーの前の路上があって初めて回答として
意味をなすのです。

それと同じです。
補語だろうが、修飾語であろうが、分詞構文であろうが、好きにご解釈
ください
0913名無しさん@英語勉強中 (ワッチョイ 57e2-cHic)
垢版 |
2020/11/30(月) 16:51:37.35ID:R9W5ULGS0
>>912
>補語だろうが、修飾語であろうが、分詞構文であろうが、好きにご解釈

それは「つまり同じ構造だ」じゃなくて、自分が「(叙述)補語」の機能も「修飾語」の機能も満足に理解してないから「分からない」というだけののことです。

分かっていないものを「同じだ」と言うことはできません。似たような単語が3つ並んでいれば「同じ構造」になるわけではありません。

オリジナルの文法概念を生み出す前に、教科書をちゃんと読み直しましょう。
0916名無しさん@英語勉強中 (スフッ Sdbf-+ooH)
垢版 |
2020/12/01(火) 16:58:57.47ID:hbsJIvZFd
>>910
バカ丸出しw
0918名無しさん@英語勉強中 (ワッチョイW 9b96-vonF)
垢版 |
2020/12/03(木) 18:35:52.06ID:Hqz0oVP+0
I was surprised to discover you a fellow guest.
あなたも客として来ておられることを知って驚きました。
って文章が英文解釈教室という本に載ってたんだけど、discoverにこういう使い方はありますか
0926名無しさん@英語勉強中 (ワッチョイ 0993-DiTS)
垢版 |
2020/12/04(金) 18:48:32.17ID:2Qg54tTT0
英語自慢の上級者さん、↓↓を英作できますか?

時間通りに着かない場合は、ご連絡ください。
0930名無しさん@英語勉強中 (ワッチョイ 0993-DiTS)
垢版 |
2020/12/04(金) 19:58:44.35ID:2Qg54tTT0
流石は上級者のみなさん! チャレンジ精神旺盛ですね。

正解に近い方がおられますが、ちょっとニュアンスが違うかもです。
もう少し経ちましたら、一つの正解例を書き込みたいと思います。
0932名無しさん@英語勉強中 (ワッチョイ 0993-DiTS)
垢版 |
2020/12/04(金) 20:46:18.72ID:2Qg54tTT0
おまたせしました。正解例です。

If you will not arrive on time, please let us know.
時間通りに着かない場合は、ご連絡ください。

解説
通常、条件の副詞節では未来の行為を現在形で表します。
この場合、if節の行為からみて、主節の行為が未来になります。
しかし、主節の行為からみて、if節の行為が未来になる場合もあります。
その場合は、if節でwillを用いる必要があります。

よって、if you are late など現在形で書いた方は不正解です。
『遅れた後、連絡ください』っといったニュアンスになってます。

if you are going to be late.は、始めから遅れる予定のようなニュアンスが感じとれ、
当日の指示と言うよりは、それよりもずっと前の場面を想像してしまいます。
『時間通りに着かない場合』をきっちり英訳しきれてるとはいい難いのですが、
英語としては奇妙に聞こえません。よって三角とさせてもらいます。
0933名無しさん@英語勉強中 (ワッチョイ 0993-DiTS)
垢版 |
2020/12/04(金) 20:51:38.15ID:2Qg54tTT0
>>931
すいません。
私の力量不足で何が言いたいのか
この文からは解りません。
0935名無しさん@英語勉強中 (ワッチョイ 2ae3-lq0O)
垢版 |
2020/12/04(金) 21:34:43.21ID:mfAqTbqn0
2箇所訂正
→on time
→ならないかな?

931をDeepLに訳させたらこうなったよ。
「お時間に正確に到着することが難しい場合は、お手数ですが事前にご連絡いただければ幸いです。」
0937名無しさん@英語勉強中 (ワッチョイ 0993-DiTS)
垢版 |
2020/12/04(金) 21:47:10.59ID:2Qg54tTT0
>>934
ここでは、willの『意思』を解釈に持ち出すほどの、
コンテックスは含まれてないと思いますよ。
素直に単純未来とるのが自然です。

If you do not arrive on timeにすると、
『もし時間通りつけなかったら、その後に連絡ください』
と言う意味になります。

『もし時間通りにつけない事となるとわかったら、連絡ください』とするためには、
If you will not arrive on timeとwillをつける必要があります。
0938三年英太郎 ◆3CZBjOt3.Y (ワッチョイ f686-jrSk)
垢版 |
2020/12/04(金) 21:54:22.36ID:PhyY0ajB0
面白い話だと思ったけど、設問に無理があるわね。

If you will not arrive on time, please let us know.

時間通りに着かない場合は、ご連絡ください。

この英→和は良いけども、和→英は無理な話よ。
0939名無しさん@英語勉強中 (ワッチョイ 0993-DiTS)
垢版 |
2020/12/04(金) 21:56:27.53ID:2Qg54tTT0
訂正
『もし時間通りにつけない(ことになる)とわかったら、連絡ください』
0940三年英太郎 ◆3CZBjOt3.Y (ワッチョイ f686-jrSk)
垢版 |
2020/12/04(金) 21:58:44.88ID:PhyY0ajB0
> 主節の行為からみて、if節の行為が未来になる場合もあります。
> その場合は、if節でwillを用いる必要があります。

まずこの解説が間違いだと思うわ。たぶん誤読してるわ。
出典は何かしら?
0942名無しさん@英語勉強中 (ワッチョイ 0993-DiTS)
垢版 |
2020/12/04(金) 22:06:28.35ID:2Qg54tTT0
>>938
そんな事ないとおもいますよ。
意図しない英文になることもあるでしょうが、
いまのところこの問題のキーポイントが
やはり議論になってます。

まあ正解不正解にあまりこだわらないで、
解説の部分で疑問があれば掘り下げて質問してもらえたほうが、
楽しめるとおもいます
0943三年英太郎 ◆3CZBjOt3.Y (ワッチョイ f686-jrSk)
垢版 |
2020/12/04(金) 22:12:54.96ID:PhyY0ajB0
if と will の共起について、PEU4は5種に分類しているわ。(§243)

そのうち、932の用法は2にあたるわね。

> We use "will" with "if" when we are saying ‘if it is true now that …’ or ‘if we know now that …’.

例: If Anna won’t be here on Thursday, we’d better cancel the meeting.

これは、「アナが木曜にここに来ない」ことがいま真であるなら・・・
ということであって、cancel the meeting → Anna won't be here
の時系列を示しているわけではないわ。

If you will not arrive on time, please let us know.

「あなたが時間通りにつかない」ことが、いま、真であるなら・・・
ということね。発話時が中心軸なのであって、
"let me know"する時間を中心としてるわけではないでしょう。
0944名無しさん@英語勉強中 (ワッチョイ 0993-DiTS)
垢版 |
2020/12/04(金) 22:14:38.85ID:2Qg54tTT0
>>941
予め知っていなければ、
アクシデントで遅れてしまう場合は
連絡しなくてもいいと取られてしまいます。
0946三年英太郎 ◆3CZBjOt3.Y (ワッチョイ f686-jrSk)
垢版 |
2020/12/04(金) 22:23:56.86ID:PhyY0ajB0
次は日本語の問題に移るわ。

a. 時間通りに着かない場合には、連絡してください。
b. 時間通りに着かなかった場合には、連絡してください。

行為を過去→未来の順にしてみると

a. 連絡→(時間遅れで)着く
b. 連絡→(時間遅れで)着く、(時間遅れで)着く→連絡

b. はどっちにも解釈しうるように思うけども、まあここでは問題ではないわ。

このように、日本語では「連絡する」ことが時間の中心になってるわね。

***************************************

つまり、日本語と英語ではアスペクト解釈に違いがあるのよ。
だから英→和にした文を、また和→英に戻すことは難しいわ。
0947名無しさん@英語勉強中 (ワッチョイ 0993-DiTS)
垢版 |
2020/12/04(金) 22:29:57.71ID:2Qg54tTT0
>>936
文法的にはクリアーしてるかも? ただ
I’m running late.と言うと、遅刻するかどうかは分からない状態です。
『遅れそう』『急げば間に合うかもしれない』というようなニュアンスです。

If you are running late,
『もし遅れそうだが急げば間に合うかもしれない場合は、ご連絡ください』になってしまいます。
わざわざこの様な言い回しを選ぶかなーって思いますね。
0949名無しさん@英語勉強中 (ワッチョイ 8492-OYD7)
垢版 |
2020/12/04(金) 22:41:54.22ID:GyeDGlQr0
>>943
じゃあ、
If Anna isn't be here on Thursday, we'd be better cancel the meeting.
の場合は、未来を表す副詞節だから現在という言い方をすることに
ついて、
このときの副詞節は条件を表しているから、条件には未来というものは
ない。これこれの条件のもとで、ということだから現在
と説明している本を読んだ。
willは法の助動詞なので、条件というものをあらわすというのでは
なくて、判断が入っている、それが現在真実と思っているときには
willをつかう、というふうになるのかな?
0950三年英太郎 ◆3CZBjOt3.Y (ワッチョイ f686-jrSk)
垢版 |
2020/12/04(金) 22:54:07.26ID:PhyY0ajB0
>>948
日本語に仮定法はないわ。

>>949

> 条件文にwillが生起しうる理由は,条件文がnon-predictive (非予測的),
あるいはepistemic (認識的) ,speech-act (言語行為的) である場合に
willが生起可能であるからと論じている.つまり,文脈的に既知(contextually
given) である内容を条件文にした場合,willが生起可能ということである
http://www.yonago-k.ac.jp/tosho/tosho/research_rep/45/pdf/03_Will_occurrence.pdf

If *Anna won’t be here on Thursday*, we’d better cancel the meeting.

アスタリスクで囲んだ箇所は、既知である内容というわけね。
だから、「アナが木曜に来ないことは真である」ということなら、
ミーティングはキャンセルしよう、ということになるわけね。
0952名無しさん@英語勉強中 (ワントンキン MM3e-UqLj)
垢版 |
2020/12/04(金) 23:05:50.17ID:kpsqJU5JM
やっぱ文法のこういうニュアンスって海外ドラマとかで身につけていくしかないんかな
文法書のルールから英文作ろうとするからこうなる海外ドラマとかで「遅れそうなら連絡して」ってシーンがあればそこで言われてるのがそのまま正解やし
0953名無しさん@英語勉強中 (ワッチョイ 5cb5-zYXO)
垢版 |
2020/12/04(金) 23:11:54.53ID:Fs+5lYOU0
>>947
あなたは running late という表現をご存知なくて、
ググって最初に出てくるこのページを見て感想を書いたんですね。

https://nic-english.com/phrase/im-running-late/
なにせ、「遅れそう」「急げば間に合うかもしれない」など全く表現が一緒ですから・・

If you are running late, let us know.
というネイティブがよく使う定型化した表現を、
「わざわざこの様な言い回しを選ぶかなー」など言ってる時点でもうね・・
0955名無しさん@英語勉強中 (ワッチョイ 0993-DiTS)
垢版 |
2020/12/04(金) 23:52:01.05ID:2Qg54tTT0
>>945
その場合もありますが、私が書いた解釈もできます。

物事うまく言ってる時はいいのですが、
トラブル発生した時にどうとでも言い訳されてしまいそうな予感がします。

if you knowではなく、If you will not arriveと、
判断基準をダイレクトに書くことをオススメするかな―。
0956名無しさん@英語勉強中 (ワッチョイ 0993-DiTS)
垢版 |
2020/12/04(金) 23:58:44.77ID:2Qg54tTT0
>>943
そちらの引用は、use "will" with "if"の形の時の
意味を説明してるだけにしか見えないですね。
willがあれば真だよと書いてるだけじゃないのかな?
真である場合、全てにwillが着くと理解してるのかな? ではないよね?w
ちょっと何言ってるか、よく理解できませんでした。また明日ですね―
0957名無しさん@英語勉強中 (ワッチョイ 0993-DiTS)
垢版 |
2020/12/05(土) 00:12:13.61ID:UY/bkGjc0
>>953
別に引用してもいいでしょ。君も自信がなくて確認しに行ったんでしょ?w

あなたの文は、ネイティブがよく使う定型化した表現かもしれませんが、
時間通りに着かない場合は、ご連絡ください。って
文章からイメージされる場面からは、かけ離れてると思いますよ。

あまりにもフランクだし、かなりの親しい関係を想像するし、
友達でもちょっと失礼かもなって思いました。
0958名無しさん@英語勉強中 (ワッチョイ 5cb5-zYXO)
垢版 |
2020/12/05(土) 00:31:18.77ID:5rDISafN0
>>957

>別に引用してもいいでしょ。君も自信がなくて確認しに行ったんでしょ?w

貴方のレス内容から、英語ができない方とお見受けしたので、
どこかの受け売りだろうとググってみたところ、ドンピシャってわけです。


>あなたの文は、ネイティブがよく使う定型化した表現かもしれませんが、

あらら、前言をすぐに撤回されるのですね(笑)


これ以上、貴方と議論する価値は全くありませんので、これで私は最後にします。
0959名無しさん@英語勉強中 (ワッチョイ 0993-DiTS)
垢版 |
2020/12/05(土) 00:42:06.81ID:UY/bkGjc0
>>958
撤回はしてませんよ。
切り取りしないで最後まで読んでくださいね。

時間通りに着かない場合は、ご連絡ください。
If you are running late, let us know.
ぜひこの英語力でこれからも頑張ってください。

私もあなたにこれ以上アドバイスすることはないでしょう
0961名無しさん@英語勉強中 (ワッチョイ baf0-jrSk)
垢版 |
2020/12/05(土) 01:53:15.35ID:8gAefJJP0
>>926
遅れたら なのか 遅れると分かってるなら
なのか、その文だけじゃ判断できない
0963名無しさん@英語勉強中 (ワッチョイ faf0-XXx1)
垢版 |
2020/12/05(土) 07:20:08.93ID:GkAS/dJJ0
snow has accumulated as tall as 1mは文法(語法)的にダメですか?
tallは副詞ではなく形容詞ですが、この文は個人的に語呂的な違和感がなく
副詞的に使え得る例外ルールがあったりするのかな?とか考えました
0964名無しさん@英語勉強中 (ワッチョイ 2c32-1fWK)
垢版 |
2020/12/05(土) 08:17:31.64ID:hep7WcxN0
>>963
× accumulated as tall as

〇 accumulated as much as
0965名無しさん@英語勉強中 (ワッチョイ 8492-OYD7)
垢版 |
2020/12/05(土) 09:40:13.35ID:y4B5rkbq0
これ、1m as tallとか1m as much
とかできるのかな?
1. snow has accumulated 1m as tall.
みたいなふうに。

2. snow has accumulated as tall as 1m
はOKだとおもうけど。つまり
3. snow has accumulated as tall as it is 1m.
が2になったみたいな感じか。

だけどどうなんだろう?
0966名無しさん@英語勉強中 (ワッチョイ 0993-DiTS)
垢版 |
2020/12/05(土) 11:23:19.80ID:UY/bkGjc0
>>961
まあ捉え方の個人差はありますよね。
ただ時間通りに着かない場合は、ご連絡ください。っと日本語でいわれて、
そこまで混乱するほどの事でもないと思いますけどね。
0967名無しさん@英語勉強中 (ワッチョイ 0993-DiTS)
垢版 |
2020/12/05(土) 11:53:25.32ID:UY/bkGjc0
>>943
俺がかいた説明で、文作れるし、意味取りも間違わないと思うよ。
君の解説である真かどうかだけで、ちゃんと成立するのか疑問だな。

真であることを根拠に、
ただしく英作できるの?
あと下の2つの意味の違いを説明できるのかな?

If you do not arrive on time, please let us know.
If you will not arrive on time, please let us know.

発話時が中心軸がlet me knowって命令文に反映されてますよね?w
時間の差は考慮されてると思うよ。
中心軸がウンたらとか、君は何が言いたいのか分からんな。
0968名無しさん@英語勉強中 (ワッチョイ 0993-DiTS)
垢版 |
2020/12/05(土) 12:02:05.30ID:UY/bkGjc0
>>960
俺は英語の間違いをズバリ指摘して、
片や『コピペだろ』と罵倒しか出来ない人の肩を持つのかな?

相当悔しかったんだろうねーw
0970名無しさん@英語勉強中 (ワッチョイ 0993-DiTS)
垢版 |
2020/12/05(土) 12:44:31.59ID:UY/bkGjc0
>>969
ENGLISH EX 副詞節 K-15ですよ
この問題は、TOEFLでも出てこないレベルで、
教養の有るネイティブと同等の英語力が必要だそうです。
受験で出ないので、普通の参考書には載ってないと思います。

まあ英語の形を説明するのに御託は並べましたが、
あの説明で正しく形が成り立つなら問題ないでしょう。

成り立ってないと証明してもらえたなら有り難いんですが、
『俺の持ってる文法書と書いてることが違う』とだけ言われても、
そらいろんな考え方があるだろうなってなるだけ。
困惑でしかない。

まあ―種明かししたので、ここら辺でお開きですかね。

皆さんお疲れさまでした。
0971名無しさん@英語勉強中 (ワッチョイ d7e2-19N2)
垢版 |
2020/12/05(土) 12:51:30.25ID:WJPZaGGN0
何これ、ゴミじゃん(笑)。

高山英士
大学卒業後、通訳及び翻訳の仕事を始める。
インタースクールやサイマルアカデミー等で
通訳者養成クラスや英検上級講座の講師を務める。
また、長年に渡り英和辞典の執筆と校閲を行うなど、
英語の実務と教育に関して豊富な知識と経験を有する。

森 一泰
大学在学時から翻訳の仕事を始める。
放送英語の翻訳,学術論文の翻訳,国際会議の英文スピーチ作成,
国立大学の和文英訳の解答・模擬試験の作成等,
英語の実務と英語教育に関して豊富な知識と経験を有する
0972三年英太郎 ◆3CZBjOt3.Y (ワッチョイ f686-jrSk)
垢版 |
2020/12/05(土) 13:20:03.37ID:2Sg89mYs0
>>967
If you do not arrive on time, please let us know.
もし遅れるなら

If you will not arrive on time, please let us know.
もし遅れると今わかってるなら

あたしが言いたいのは、あーたの「正解」の英文と和文では
中心となってる時間軸が違うから、和→英を想像するのは
ムリってこと。その参考本は、英→和でしょう?
0973三年英太郎 ◆3CZBjOt3.Y (ワッチョイ f686-jrSk)
垢版 |
2020/12/05(土) 13:25:23.47ID:2Sg89mYs0
PEUの例

If Anna won’t be here on Thursday, we’d better cancel the meeting.
アナが木曜に来ないとわかってるなら、ミーティングはキャンセルした方がいいね

If prices will really come down in a few months, I’m not going to buy one now.
物価が数か月後に下がるっていうのなら、今は買わないことにするよ

あーたは、この will は時間が「主節→ if節」に進むというマーカーだというけど、
そうでないのは例文から明らかじゃない。どっちが先とは言い難いわ。
そうではなく、PEUでは‘if it is true now that …’ or ‘if we know now that …’.
の意であると言ってるのよ。
0974名無しさん@英語勉強中 (ワッチョイ 0993-DiTS)
垢版 |
2020/12/05(土) 13:29:44.18ID:UY/bkGjc0
>>971
君は一生ゴミだろうねw
0975三年英太郎 ◆3CZBjOt3.Y (ワッチョイ f686-jrSk)
垢版 |
2020/12/05(土) 13:45:24.08ID:2Sg89mYs0
> 条件節内でwillが生起しうる場合に共通するのは,
> 図に示されているように,二重破線矢印が表すモダリティの力を
> 法助動詞willを使って命題を査定しているのが話し手Sではなく,
> それ以外の概念化主体Cであるという点である.
http://www.yonago-k.ac.jp/tosho/tosho/research_rep/45/pdf/03_Will_occurrence.pdf

If [Anna won’t be here on Thursday], we’d better cancel the meeting.

話し手以外の概念化主体Cが [ ] 内の命題を査定している、と言ってるわね。

「アナが木曜に来ない」という命題が真ならば

アナが木曜に来れないということなら・・・

********************************************

最初も問題も、「時間通りに着かないということなら・・・」だったら
だいぶもとの英文を想像しやすくなるはずだわ。
0977名無しさん@英語勉強中 (スップ Sd02-O6iA)
垢版 |
2020/12/05(土) 14:34:43.95ID:RjujqD2id
亀レスだけどI'm with youはあなたに同意する、あなたの味方と言う意味。
I'm with you meaningで検索するとたくさん同じ回答が出てくる
0978名無しさん@英語勉強中 (スップ Sd00-O6iA)
垢版 |
2020/12/05(土) 15:27:30.87ID:hZVcewz+d
>>947
I’m running late for today’s meeting, hopefully by only five minutes. So sorry, my apologies.
running lateはビジネスシーンで普通に使うよ。ここ英太郎と英太郎の別ワッチョイしかいないから。
0979名無しさん@英語勉強中 (ワッチョイ 0993-DiTS)
垢版 |
2020/12/05(土) 16:23:22.29ID:UY/bkGjc0
>>978
普通に使うかって事ではなく、
時間通りに着かない場合は、ご連絡ください。
って意味として使わんよなって事。

あーもう面倒くさいな〜
0980名無しさん@英語勉強中 (ワッチョイ 0993-DiTS)
垢版 |
2020/12/05(土) 16:37:49.77ID:UY/bkGjc0
>>976
全部そうだよ。どっちか先はわからん君には何言っても無駄だか?w

>If Anna won’t be here on Thursday, we’d better cancel the meeting.
>If Anna is not here on Thursday, we’d better cancel the meeting.
それぞれ の意味、いつnot hereが確認されて、いつcancel the meetingされるの?

君の参考書もホントのこと書いてると思うけど
君が中途半端に一部引用してるだけにしか思えん
0986名無しさん@英語勉強中 (ワッチョイ 5c89-jrSk)
垢版 |
2020/12/05(土) 19:53:55.88ID:hPkj+lYb0
>>963
snow と tall とを組み合わせるのは、たぶん変なんだろうと思う。
(もちろん、snowman には tall という言葉が似あうだろうけど。)
次のような用例なら見つかる。

(1) Call us to get the snow removed from your driveway or sidewalk,
when ★snow has accumulated to a height of two inches★ and more

(2) Hand shoveling services will begin when ●snow has accumulated to two inches●.

(3) Whenever ◆snow has accumulated to a depth of two inches◆ (2″) or more on any curbed street or ...
0988名無しさん@英語勉強中 (ワッチョイ 5c89-jrSk)
垢版 |
2020/12/05(土) 20:02:46.59ID:hPkj+lYb0
>>963
次のような言い回しもある。

(1) As ●snow piled up to more than 60 inches● in Erie, Pennsylvania, forecasters
warn of frigid, sub-zero arctic air and dangerously cold wind chills in ...

(2) A dispatch received today from Tonopah said that while
★snow piled up to a depth of six inches★ in town, the depth was as great as

(3) ◆Snow piled up to a height of 10 cm (4 inches)◆ in the Golan Heights at week's end,
bringing numerous tourists to Israel's north-eastern plateau.

(4) Soft and flaky, ★the snow piled up to a depth of six or seven inches★ in this territory.
0991名無しさん@英語勉強中 (ワッチョイ 5c89-jrSk)
垢版 |
2020/12/06(日) 06:40:32.51ID:4gqHftiH0
>>990 が、いいことを言ってくれた。

一言でいうと、tall は「細長いもの、やせた感じのするもの」に使う。
山なんかは太った感じがするから、tall は似合わない。

high は、「高い」感じのするものならどんなものにでも使える
万能の言葉だけど、tall が使える場面は限られているらしい。

wall はどうかというと、tall も high も両方とも使えるみたいだけど、
やっぱり3階建て以上の細長い wall が tall であり、平屋の壁みたいに
太った感じのする wall は high と言わねばならないんだろうな、と俺は思っている。


tall
★high and narrow or long★ -- use this about trees and plants
or about buildings and parts of buildings

- tall marble columns
- tall grass

Source: Longman Language Activator, Second Edition
0992名無しさん@英語勉強中 (ワッチョイ 5c89-jrSk)
垢版 |
2020/12/06(日) 06:47:18.68ID:4gqHftiH0
>>963
tall については、全20巻の OED には、次のようにさらに厳密な定義がある。
もちろん、簡単に言えばすでに示した Longman の辞書通りなんだけど。

tall
7. a. Of things, as ships (spec. square-riggers), trees, mountains:
High, lofty; ★esp. of things high in proportion to their width★, as tall chimney, column, house, mast, spire.
0993名無しさん@英語勉強中 (ワッチョイ 5cb5-zYXO)
垢版 |
2020/12/06(日) 06:53:49.71ID:MaB3rOpN0
>>989
ビジネスシーンどころか、お客様相手でも普通に使う。
一応証拠を示しておくと
"If you are running late, let us know" で検索すると山のように用例が出てくるが
たった1、2ページだけでもこんな感じ(これでも抜粋)

If you are running late, let us know and we will set up a place for you.(ヨガ教室 アメリカ)
If you are running late let us know and we will do our best to accommodate you. (ヘアサロン アメリカ)
If you are running late, let us know(クリニック アイルランド)
If you are running late, let us know as soon as possible and we will try to accommodate you without disrupting other client appointments (in some cases, we may need to reschedule your appointment) (エステサロン アメリカ)
If you are running late, let us know and we will be happy to accommodate you. (ヘアサロン アメリカ)
if you are running late, let us know by calling. (ヘアサロン アメリカ)
If you are running late, let us know. We may be able to accommodate you. (健康サロン アメリカ)
If you are running late, let us know.(動物病院 アメリカ)
If you are running late, let us know. (食品販売サイト アメリカ)
0999名無しさん@英語勉強中 (スププ Sd70-45xP)
垢版 |
2020/12/06(日) 20:10:02.55ID:BJsWJiwBd
>>993
使われてるからどうかでなくて、日本語訳の気持ちがはいってないんだろ。そんだけ大味に取り組んでるから、if you are lateと書く奴が沢山出てくる。 ここで踏ん反り替えてる英太郎の日本語訳もひどいもんだ。
なんでこんな日本語が通じないんやろね。
1000名無しさん@英語勉強中 (ワッチョイ 125a-bout)
垢版 |
2020/12/06(日) 20:20:25.91ID:c5q93Xqt0
1000
10011001
垢版 |
Over 1000Thread
このスレッドは1000を超えました。
新しいスレッドを立ててください。
life time: 174日 7時間 5分 30秒
10021002
垢版 |
Over 1000Thread
5ちゃんねるの運営はプレミアム会員の皆さまに支えられています。
運営にご協力お願いいたします。


───────────────────
《プレミアム会員の主な特典》
★ 5ちゃんねる専用ブラウザからの広告除去
★ 5ちゃんねるの過去ログを取得
★ 書き込み規制の緩和
───────────────────

会員登録には個人情報は一切必要ありません。
月300円から匿名でご購入いただけます。

▼ プレミアム会員登録はこちら ▼
https://premium.5ch.net/

▼ 浪人ログインはこちら ▼
https://login.5ch.net/login.php
レス数が1000を超えています。これ以上書き込みはできません。

ニューススポーツなんでも実況